Exam 2

Lakukan tugas rumah & ujian kamu dengan baik sekarang menggunakan Quizwiz!

A young adult patient who denies any history of smoking is seen in the clinic with a new diagnosis of chronic obstructive pulmonary disease (COPD). The nurse should plan to teach the patient about a. a1-antitrypsin testing. b. leukotriene modifiers. c. use of the nicotine patch. d. continuous pulse oximetry.

A When COPD occurs in young patients, especially without a smoking history, a genetic deficiency in a1-antitrypsin should be suspected. Because the patient does not smoke, a nicotine patch would not be ordered. There is no indication that the patient requires continuous pulse oximetry. Leukotriene modifiers would be used in patients with asthma, not with COPD.

A patient has acute bronchitis with a nonproductive cough and wheezes. Which topic should the nurse plan to include in the teaching plan? a. Purpose of antibiotic therapy b. Ways to limit oral fluid intake c. Appropriate use of cough suppressants d. Safety concerns with home O2 therapy

ANS: C Cough suppressants are frequently prescribed for acute bronchitis. Because most acute bronchitis is viral in origin, antibiotics are not prescribed unless there are systemic symptoms. Fluid intake is encouraged. Home O2 is not prescribed for acute bronchitis, although it may be used for chronic bronchitis. DIF: Cognitive Level: Apply (application) REF: 500 TOP: Nursing Process: Planning MSC: NCLEX: Physiological Integrity

The nurse should expect to assess which causative agent in a child with warts? a. Bacteria b. Fungus c. Parasite d. Virus

ANS: D Human warts are caused by the human papillomavirus. Infection with bacteria, fungus, and parasites does not result in warts.

Which finding in a patient hospitalized with bronchiectasis is most important to report to the health care provider? a. Cough productive of bloody, purulent mucus b. Scattered crackles and wheezes heard bilaterally c. Complaint of sharp chest pain with deep breathing d. Respiratory rate 28 breaths/minute while ambulating

A Hemoptysis may indicate life-threatening hemorrhage, and should be reported immediately to the health care provider. The other findings are frequently noted in patients with bronchiectasis and may need further assessment but are not indicators of life-threatening complications.

10. Which intervention will the nurse include in the plan of care for a patient who has cardiogenic shock? a. Check temperature every 2 hours. b. Monitor breath sounds frequently. c. Maintain patient in supine position. d. Assess skin for flushing and itching.

ANS: B Because pulmonary congestion and dyspnea are characteristics of cardiogenic shock, the nurse should assess the breath sounds frequently. The head of the bed is usually elevated to decrease dyspnea in patients with cardiogenic shock. Elevated temperature and flushing or itching of the skin are not typical of cardiogenic shock. DIF: Cognitive Level: Apply (application) REF: 1591 TOP: Nursing Process: Implementation MSC: NCLEX: Physiological Integrity

6. A patient who has experienced blunt abdominal trauma during a motor vehicle collision is complaining of increasing abdominal pain. The nurse will plan to teach the patient about the purpose of a. peritoneal lavage. b. abdominal ultrasonography. c. nasogastric (NG) tube placement. d. magnetic resonance imaging (MRI).

ANS: B For patients who are at risk for intraabdominal bleeding, focused abdominal ultrasonography is the preferred method to assess for intraperitoneal bleeding. An MRI would not be used. Peritoneal lavage is an alternative, but it is more invasive. An NG tube would not be helpful in the diagnosis of intraabdominal bleeding.

Which is the causative agent of scarlet fever? a. Enteroviruses b. Corynebacterium organisms c. Scarlet fever virus d. Group A β-hemolytic streptococci (GABHS)

ANS: D GABHS infection causes scarlet fever. Enteroviruses do not cause the same complications. Corynebacterium organisms cause diphtheria. Scarlet fever is not caused by a virus.

A patient's vital signs are pulse 87, respirations 24, and BP of 128/64 mm Hg and cardiac output is 4.7 L/min. The patient's stroke volume is _____ mL. (Round to the nearest whole number.)

ANS: 54 Stroke volume = cardiac output/heart rate

A patient with asthma has a personal best peak expiratory flow rate (PEFR) of 400 L/min. When explaining the asthma action plan, the nurse will teach the patient that a change in therapy is needed when the PEFR is less than ___ L/minute

320 A PEFR less than 80% of the personal best indicates that the patient is in the yellow zone where changes in therapy are needed to prevent progression of the airway narrowing.

An infant's parents ask the nurse about preventing OM. Which should be recommended? a. Avoid tobacco smoke. b. Use nasal decongestant. c. Avoid children with OM. d. Bottle-feed or breastfeed in supine position

A

In providing nourishment for a child with cystic fibrosis (CF), which factor should the nurse keep in mind? a. Diet should be high in carbohydrates and protein. b. Diet should be high in easily digested carbohydrates and fats. c. Most fruits and vegetables are not well tolerated. d. Fats and proteins must be greatly curtailed.

A

Parents of a child with cystic fibrosis ask the nurse about genetic implications of the disorder. Which statement, made by the nurse, expresses accurately the genetic implications? a. If it is present in a child, both parents are carriers of this defective gene. b. It is inherited as an autosomal dominant trait. c. It is a genetic defect found primarily in non-Caucasian population groups. d. There is a 50% chance that siblings of an affected child also will be affected.

A

Which nursing action for a patient with chronic obstructive pulmonary disease (COPD)could the nurse delegate to experienced unlicensed assistive personnel (UAP)? a. Obtain O2 saturation using pulse oximetry. b. Monitor for increased O2 need with exercise. c. Teach the patient about safe use of O2 at home. d. Adjust O2 to keep saturation in prescribed parameters.

A UAP can obtain O2 saturation (after being trained and evaluated in the skill). The other actions require more education and a scope of practice that licensed practical/vocational nurses (LPN/LVNs) or registered nurses (RNs) would have.

A nurse is conducting an in-service on asthma. Which statement is the most descriptive of bronchial asthma? a. There is heightened airway reactivity. b. There is decreased resistance in the airway. c. The single cause of asthma is an allergic hypersensitivity. d. It is inherited.

A In bronchial asthma, spasm of the smooth muscle of the bronchi and bronchioles causes constriction, producing impaired respiratory function. In bronchial asthma, there is increased resistance in the airway. There are multiple causes of asthma, including allergens, irritants, exercise, cold air, infections, medications, medical conditions, and endocrine factors. Atopy or development of an immunoglobulin E (IgE)-mediated response is inherited but is not the only cause of asthma.

Effective lone-rescuer CPR on a 5-year-old child should include a. two breaths to every 30 chest compressions. b. two breaths to every 15 chest compressions. c. reassessment of child after 50 cycles of compression and ventilation. d. reassessment of child every 10 minutes that CPR continues.

A Lone-rescuer CPR is two breaths to 30 compressions for all ages until signs of recovery occur. Reassessment of the child should take place after 20 cycles or 1 minute.

The Heimlich maneuver is recommended for airway obstruction in children older than _____ year(s). a. 1 b. 4 c. 8 d. 12

A The Heimlich maneuver is recommended for airway obstruction in children older than 1 year. Younger than 1 year, back blows and chest thrusts are administered. The Heimlich maneuver can be used in children older than 1 year.

The nurse in the emergency department receives arterial blood gas results for four recently admitted patients with obstructive pulmonary disease. The results for which patient will require the most rapid action by the nurse? a. pH 7.28, PaCO2 50 mm Hg, and PaO2 58 mm Hg b. pH 7.48, PaCO2 30 mm Hg, and PaO2 65 mm Hg c. pH 7.34, PaCO2 33 mm Hg, and PaO2 80 mm Hg d. pH 7.31, PaCO2 58 mm Hg, and PaO2 64 mm Hg

A The pH, PaCO2, and PaO2 indicate that the patient has severe uncompensated respiratory acidosis and hypoxemia. Rapid action will be required to prevent increasing hypoxemia and correct the acidosis.

A patient who has been experiencing an asthma attack develops bradycardia and a decrease in wheezing. Which action should the nurse take first? a. Notify the health care provider. b. Document changes in respiratory status. c. Encourage the patient to cough and deep breathe. d. Administer IV methylprednisolone (Solu-Medrol).

A The patient's assessment indicates impending respiratory failure, and the nurse should prepare to assist with intubation and mechanical ventilation after notifying the health care provider. IV corticosteroids require several hours before having any effect on respiratory status. The patient will not be able to cough or deep breathe effectively. Documentation is not a priority at this time.

Which information about a patient who is receiving cisatracurium (Nimbex) to prevent asynchronous breathing with the positive pressure ventilator requires action by the nurse? a. No sedative has been ordered for the patient. b. The patient does not respond to verbal stimulation. c. There is no cough or gag reflex when the patient is suctioned. d. The patient's oxygen saturation remains between 90% to 93%.

ANS: A Because neuromuscular blockade is extremely anxiety provoking, it is essential that patients who are receiving neuromuscular blockade receive concurrent sedation and analgesia. Absence of response to stimuli is expected in patients receiving neuromuscular blockade. The O2 saturation is adequate. DIF: Cognitive Level: Apply (application)

23. The urgent care center protocol for tick bites includes the following actions. Which action will the nurse take first when caring for a patient with a tick bite? a. Use tweezers to remove any remaining ticks. b. Check the vital signs, including temperature. c. Give doxycycline (Vibramycin) 100 mg orally. d. Obtain information about recent outdoor activities.

ANS: A Because neurotoxic venom is released as long as the tick is attached to the patient, the initial action should be to remove any ticks using tweezers or forceps. The other actions are also appropriate, but the priority is to minimize venom release.

Which assessment finding obtained by the nurse when caring for a patient with a right radial arterial line indicates a need for the nurse to take immediate action? a. The right hand is cooler than the left hand. b. The mean arterial pressure (MAP) is 77 mm Hg. c. The system is delivering 3 mL of flush solution per hour. d. The flush bag and tubing were last changed 3 days previously.

ANS: A The change in temperature of the left hand suggests that blood flow to the left hand is impaired. The flush system needs to be changed every 96 hours. A mean arterial pressure (MAP) of 75 mm Hg is normal. Flush systems for hemodynamic monitoring are set up to deliver 3 to 6 mL/hour of flush solution.

19. During change-of-shift report, the nurse is told that a patient has been admitted with dehydration and hypotension after having vomiting and diarrhea for 4 days. Which finding is most important for the nurse to report to the health care provider? a. New onset of confusion b. Decreased bowel sounds c. Heart rate 112 beats/min d. Pale, cool, and dry extremities

ANS: A The changes in mental status are indicative that the patient is in the progressive stage of shock and that rapid intervention is needed to prevent further deterioration. The other information is consistent with compensatory shock. DIF: Cognitive Level: Analyze (analysis) REF: 1597 OBJ: Special Questions: Prioritization TOP: Nursing Process: Assessment MSC: NCLEX: Physiological Integrity

The nurse is caring for a newborn whose mother is diabetic. Which clinical manifestations should the nurse expect to see? a. Hypoglycemic, large for gestational age b. Hyperglycemic, large for gestational age c. Hypoglycemic, small for gestational age d. Hyperglycemic, small for gestational age

ANS: A The clinical manifestations of a newborn born to a mother with diabetes include being large for gestational age, being plump and full-faced, having abundant vernix caseosa, being listless and lethargic, and having hypoglycemia. These manifestations appear a short time after birth. The newborn is hypoglycemic from increased fetal production of insulin and large for gestational age.

A nurse is admitting a child to the hospital with a diagnosis of giardiasis. Which medication should the nurse expect to be prescribed? a. Metronidazole (Flagyl) b. Amoxicillin clavulanate (Augmentin) c. Clarithromycin (Biaxin) d. Prednisone (Orapred)

ANS: A The drugs of choice for treatment of giardiasis are metronidazole (Flagyl), tinidazole (Tindamax), and nitazoxanide (Alinia). These are classified as antifungals. Amoxicillin and clarithromycin are antibiotics that treat bacterial infections. Prednisone is a steroid and is used as an anti-inflammatory medication.

When caring for a patient who has an arterial catheter in the left radial artery for arterial pressure-based cardiac output (APCO) monitoring, which information obtained by the nurse is most important to report to the health care provider? a. The patient has a positive Allen test. b. There is redness at the catheter insertion site. c. The mean arterial pressure (MAP) is 86 mm Hg. d. The dicrotic notch is visible in the arterial waveform.

ANS: B Redness at the catheter insertion site indicates possible infection. The Allen test is performed before arterial line insertion, and a positive test indicates normal ulnar artery perfusion. A MAP of 86 is normal and the dicrotic notch is normally present on the arterial waveform.

14. Which data collected by the nurse caring for a patient who has cardiogenic shock indicate that the patient may be developing multiple organ dysfunction syndrome (MODS)? a. The patient's serum creatinine level is elevated. b. The patient complains of intermittent chest pressure. c. The patient's extremities are cool and pulses are weak. d. The patient has bilateral crackles throughout lung fields.

ANS: A The elevated serum creatinine level indicates that the patient has renal failure as well as heart failure. The crackles, chest pressure, and cool extremities are all symptoms consistent with the patient's diagnosis of cardiogenic shock. DIF: Cognitive Level: Apply (application) REF: 1591 TOP: Nursing Process: Assessment MSC: NCLEX: Physiological Integrity

A patient with respiratory failure has arterial pressure-based cardiac output (APCO) monitoring and is receiving mechanical ventilation with peak end-expiratory pressure (PEEP) of 12 cm H2O. Which information indicates that a change in the ventilator settings may be required? a. The arterial pressure is 90/46. b. The heart rate is 58 beats/minute. c. The stroke volume is increased. d. The stroke volume variation is 12%.

ANS: A The hypotension suggests that the high intrathoracic pressure caused by the PEEP may be decreasing venous return and (potentially) cardiac output. The other assessment data would not be a direct result of PEEP and mechanical ventilation.

A patient who is orally intubated and receiving mechanical ventilation is anxious and is "fighting" the ventilator. Which action should the nurse take next? a. Verbally coach the patient to breathe with the ventilator. b. Sedate the patient with the ordered PRN lorazepam (Ativan). c. Manually ventilate the patient with a bag-valve-mask device. d. Increase the rate for the ordered propofol (Diprivan) infusion.

ANS: A The initial response by the nurse should be to try to decrease the patient's anxiety by coaching the patient about how to coordinate respirations with the ventilator. The other actions may also be helpful if the verbal coaching is ineffective in reducing the patient's anxiety.

Which is the most common cause of anemia in preterm newborns? a. Frequent blood sampling b. Respiratory distress syndrome c. Meconium aspiration syndrome d. Persistent pulmonary hypertension

ANS: A The most common cause of anemia in preterm newborns is frequent blood-sample withdrawal and inadequate erythropoiesis in acutely ill newborns. Microsamples should be used for blood tests, and the amount of blood drawn should be monitored. Respiratory distress syndrome, meconium aspiration syndrome, and persistent pulmonary hypertension are not causes of anemia. They may require frequent blood sampling, which will contribute to the problem of decreased erythropoiesis and anemia.

While family members are visiting, a patient has a respiratory arrest and is being resuscitated. Which action by the nurse is best? a. Tell the family members that watching the resuscitation will be very stressful. b. Ask family members if they wish to remain in the room during the resuscitation. c. Take the family members quickly out of the patient room and remain with them. d. Assign a staff member to wait with family members just outside the patient room.

ANS: B Research indicates that family members want the option of remaining in the room during procedures such as cardiopulmonary resuscitation (CPR) and that this decreases anxiety and facilitates grieving. The other options may be appropriate if the family decides not to remain with the patient.

What causes tinea capitis (ringworm)? a. Virus b. Fungus c. Allergic reaction d. Bacterial infection

ANS: B Ringworm is caused by a group of closely related filamentous fungi that invade primarily the stratum corneum, hair, and nails. They are superficial infections that live on, not in, the skin. Virus and bacterial infection are not the causative organisms for ringworm. Ringworm is not an allergic response.

Which bite causes Rocky Mountain spotted fever? a. Flea b. Tick c. Mosquito d. Mouse or rat

ANS: B Rocky Mountain spotted fever is caused by a tick. The tick must attach and feed for at least 1 to 2 hours to transmit the disease. The usual habitat of the tick is in heavily wooded areas. Fleas, mosquitoes, and mice or rats do not transmit Rocky Mountain spotted fever.

. Which nursing action could the registered nurse (RN) working in a skilled care hospital unit delegate to an experienced licensed practical/vocational nurse (LPN/LVN) caring for a patient with a permanent tracheostomy? a. Assess the patient's risk for aspiration. b. Suction the tracheostomy when directed. c. Teach the patient to provide tracheostomy self-care. d. Determine the need for tracheostomy tube replacement.

ANS: B Suctioning of a stable patient can be delegated to LPNs/LVNs. Patient assessment and patient teaching should be done by the RN. DIF: Cognitive Level: Apply (application) REF: 490 OBJ: Special Questions: Delegation TOP: Nursing Process: Planning MSC: NCLEX: Safe and Effective Care Environment

Which hemodynamic parameter is most appropriate for the nurse to monitor to determine the effectiveness of medications given to a patient to reduce left ventricular afterload? a. Mean arterial pressure (MAP) b. Systemic vascular resistance (SVR) c. Pulmonary vascular resistance (PVR) d. Pulmonary artery wedge pressure (PAWP)

ANS: B Systemic vascular resistance reflects the resistance to ventricular ejection, or afterload. The other parameters will be monitored, but do not reflect afterload as directly.

4. A patient who is unconscious after a fall from a ladder is transported to the emergency department by emergency medical personnel. During the primary survey of the patient, the nurse should a. obtain a complete set of vital signs. b. obtain a Glasgow Coma Scale score. c. attach an electrocardiogram monitor. d. ask about chronic medical conditions.

ANS: B The Glasgow Coma Scale is included when assessing for disability during the primary survey. The other information is part of the secondary survey.

During change-of-shift report on a medical unit, the nurse learns that a patient with aspiration pneumonia who was admitted with respiratory distress has become increasingly agitated. Which action should the nurse take first? a. Give the prescribed PRN sedative drug. b. Offer reassurance and reorient the patient. c. Use pulse oximetry to check the oxygen saturation. d. Notify the health care provider about the patient's status.

ANS: C Agitation may be an early indicator of hypoxemia. The other actions may also be appropriate, depending on the findings about O2 saturation. DIF: Cognitive Level: Analyze (analysis)

Which action is a priority for the nurse to take when the low pressure alarm sounds for a patient who has an arterial line in the left radial artery? a. Fast flush the arterial line. b. Check the left hand for pallor. c. Assess for cardiac dysrhythmias. d. Rezero the monitoring equipment.

ANS: C The low pressure alarm indicates a drop in the patient's blood pressure, which may be caused by cardiac dysrhythmias. There is no indication to rezero the equipment. Pallor of the left hand would be caused by occlusion of the radial artery by the arterial catheter, not by low pressure. There is no indication of a need for flushing the line.

Nursing care of the newborn with oral candidiasis (thrush) includes: a. avoiding use of pacifier. b. removing characteristic white patches with a soft cloth. c. continuing medication for a prescribed number of days. d. applying medication to oral mucosa, being careful that none is ingested.

ANS: C The medication must be continued for the prescribed number of days. To prevent relapse, therapy should continue for at least 2 days after the lesions disappear. Pacifiers can be used. The pacifier should be replaced with a new one or boiled for 20 minutes once daily. One of the characteristics of thrush is that the white patches cannot be removed. The medication is applied to the oral mucosa and then swallowed to treat Candida organisms in the gastrointestinal tract.

Which refers to a newborn whose rate of intrauterine growth was slowed and whose birth weight falls below the 10th percentile on intrauterine growth charts? a. Postterm b. Postmature c. Low birth weight d. Small for gestational age

ANS: D A small-for-gestational-age (or small-for-date) newborn is any child whose rate of intrauterine growth was slowed and whose birth weight falls below the 10th percentile on intrauterine growth curves. A postterm or postmature newborn is any child born after 42 weeks of gestational age, regardless of birth weight. A low-birth-weight newborn is a child whose birth weight is less than 2500 g, regardless of gestational age.

The nurse is teaching nursing students about childhood skin lesions. Which is an elevated, circumscribed skin lesion that is less than 1 cm in diameter and filled with serous fluid? a. Cyst b. Papule c. Pustule d. Vesicle

ANS: D A vesicle is elevated, circumscribed, superficial, less than 1 cm in diameter, and filled with serous fluid. A cyst is elevated, circumscribed, palpable, encapsulated, and filled with liquid or semisolid material. A papule is elevated, palpable, firm, circumscribed, less than 1 cm in diameter, and brown, red, pink, tan, or bluish red. A pustule is elevated, superficial, and similar to a vesicle but filled with purulent fluid.

Which is an important nursing consideration in preventing the complications of congenital hypothyroidism (CH)? a. Assess for family history of CH. b. Assess mother for signs of hypothyroidism. c. Be certain appropriate screening is done prenatally. d. Be certain appropriate screening is done on newborn.

ANS: D Early diagnosis and treatment are essential to prevent the complications of CH. Neonatal screening is mandatory in all 50 United States and territories and is usually obtained in the first 24 to 48 hours of birth. A number of different etiologies exist for CH; family history will identify a small percentage only. The screening can be done postnatally on blood obtained via heel stick.

A patient who has just been admitted with community-acquired pneumococcal pneumonia has a temperature of 101.6° F with a frequent cough and is complaining of severe pleuritic chest pain. Which prescribed medication should the nurse give first? a. Codeine c. Acetaminophen (Tylenol) b. Guaifenesin d. Piperacillin/tazobactam (Zosyn)

ANS: D Early initiation of antibiotic therapy has been demonstrated to reduce mortality. The other medications are also appropriate and should be given as soon as possible, but the priority is to start antibiotic therapy. DIF: Cognitive Level: Analyze (analysis) REF: 501 OBJ: Special Questions: Prioritization TOP: Nursing Process: Implementation MSC: NCLEX: Physiological Integrity

To verify the correct placement of an oral endotracheal tube (ET) after insertion, the best initial action by the nurse is to a. auscultate for the presence of bilateral breath sounds. b. obtain a portable chest x-ray to check tube placement. c. observe the chest for symmetric chest movement with ventilation. d. use an end-tidal CO2 monitor to check for placement in the trachea.

ANS: D End-tidal CO2 monitors are currently recommended for rapid verification of ET placement. Auscultation for bilateral breath sounds and checking chest expansion are also used, but they are not as accurate as end-tidal CO2 monitoring. A chest x-ray confirms the placement but is done after the tube is secured.

5. A 19-yr-old patient is brought to the emergency department (ED) with multiple lacerations and tissue avulsion of the left hand. When asked about tetanus immunization, the patient denies having any previous vaccinations. The nurse will anticipate giving a. tetanus immunoglobulin (TIG) only. b. TIG and tetanus-diphtheria toxoid (Td). c. tetanus-diphtheria toxoid and pertussis vaccine (Tdap) only. d. TIG and tetanus-diphtheria toxoid and pertussis vaccine (Tdap).

ANS: D For an adult with no previous tetanus immunizations, TIG and Tdap are recommended. The other immunizations are not sufficient for this patient.

The screening test for PKU is most reliable if the blood sample is: a. from cord blood. b. taken 14 days after birth. c. taken before oral feedings are initiated. d. fresh blood from the heel.

ANS: D Fresh heel-stick blood is the preferred source for the test. Fresh heel-stick blood, not cord blood, must be used. The test must be performed soon after birth so that a low-phenylalanine diet can be instituted if required. The newborn should ingest breast milk or formula before the test is performed.

The nurse is caring for a patient with a subarachnoid hemorrhage who is intubated and placed on a mechanical ventilator with 10 cm H2O of peak end-expiratory pressure (PEEP). When monitoring the patient, the nurse will need to notify the health care provider immediately if the patient develops a. oxygen saturation of 93%. b. respirations of 20 breaths/minute. c. green nasogastric tube drainage. d. increased jugular venous distention.

ANS: D Increases in jugular venous distention in a patient with a subarachnoid hemorrhage may indicate an increase in intracranial pressure (ICP) and that the PEEP setting is too high for this patient. A respiratory rate of 20, O2 saturation of 93%, and green nasogastric tube drainage are within normal limits.

The nurse is admitting a patient diagnosed with an acute exacerbation of chronic obstructive pulmonary disease (COPD). How should the nurse determine the appropriate O2 flow rate? a. Minimize O2 use to avoid O2 dependency. b. Maintain the pulse oximetry level at 90% or greater. c. Administer O2 according to the patient's level of dyspnea. d. Avoid administration of O2 at a rate of more than 2 L/min.

B The best way to determine the appropriate O2 flow rate is by monitoring the patient's oxygenation either by arterial blood gases (ABGs) or pulse oximetry. An O2 saturation of 90% indicates adequate blood O2 level without the danger of suppressing the respiratory drive. For patients with an exacerbation of COPD, an O2 flow rate of 2 L/min may not be adequate. Because O2 use improves survival rate in patients with COPD, there is no concern about O2 dependency. The patient's perceived dyspnea level may be affected by other factors (e.g., anxiety) besides blood O2 level.

The nurse receives a change-of-shift report on the following patients with chronic obstructive pulmonary disease (COPD). Which patient should the nurse assess first? a. A patient with loud expiratory wheezes b. A patient with a respiratory rate of 38 breaths/min c. A patient who has a cough productive of thick, green mucus d. A patient with jugular venous distention and peripheral edema

B A respiratory rate of 38/min indicates severe respiratory distress, and the patient needs immediate assessment and intervention to prevent possible respiratory arrest. The other patients also need assessment as soon as possible, but they do not need to be assessed as urgently as the patient with tachypnea.

A patient with chronic obstructive pulmonary disease (COPD) has coarse crackles throughout the lung fields and a chronic, nonproductive cough. Which nursing intervention will be most effective? a. Change the O2 flow rate to the highest prescribed rate. b. Teach the patient to use the Flutter airway clearance device. c. Reinforce the ongoing use of pursed-lip breathing techniques. d. Teach the patient about consistent use of inhaled corticosteroids.

B Airway clearance devices assist with moving mucus into larger airways, where it can more easily be expectorated. The other actions may be appropriate for some patients with COPD, but they are not indicated for this patient's problem of thick mucus secretions.

The nurse is caring for a child with carbon monoxide poisoning associated with smoke inhalation. Which is essential in this child's care? a. Monitor pulse oximetry. b. Monitor arterial blood gases. c. Administer oxygen if respiratory distress develops. d. Administer oxygen if child's lips become bright, cherry red.

B Arterial blood gases are the best way to monitor carbon monoxide poisoning. Pulse oximetry is contraindicated in the case of carbon monoxide poisoning because the PaO2 may be normal. The child should receive 100% oxygen as quickly as possible, not only if respiratory distress or other symptoms develop.

A patient with cystic fibrosis (CF) has blood glucose levels that are consistently between 180 to 250 mg/dL. Which nursing action will the nurse plan to implement? a. Discuss the role of diet in blood glucose control. b. Evaluate the patient's use of pancreatic enzymes. c. Teach the patient about administration of insulin. d. Give oral hypoglycemic medications before meals.

C The glucose levels indicate that the patient has developed CF-related diabetes, and insulin therapy is required. Because the etiology of diabetes in CF is inadequate insulin production, oral hypoglycemic agents are not effective. Patients with CF need a high-calorie diet. Inappropriate use of pancreatic enzymes would not be a cause of hyperglycemia in a patient with CF.

It is now recommended that children with asthma who are taking long-term inhaled steroids should be assessed frequently because which disease or assessment findings may develop? a. Cough b. Osteoporosis c. Slowed growth d. Cushing syndrome

C The growth of children on long-term inhaled steroids should be assessed frequently to assess for systemic effects of these drugs. Cough is prevented by inhaled steroids. No evidence exists that inhaled steroids cause osteoporosis. Cushing syndrome is caused by long-term systemic steroids.

A young adult patient with cystic fibrosis (CF) is admitted to the hospital with increased dyspnea. Which intervention should the nurse include in the plan of care? a. Schedule a sweat chloride test. b. Arrange for a hospice nurse visit. c. Place the patient on a low-sodium diet. d. Perform chest physiotherapy every 4 hours.

D Routine scheduling of airway clearance techniques is an essential intervention for patients with CF. A sweat chloride test is used to diagnose CF, but it does not provide any information about the effectiveness of therapy. There is no indication that the patient is terminally ill. Patients with CF lose excessive sodium in their sweat and require high amounts of dietary sodium.

Which information will the nurse include in the asthma teaching plan for a patient being discharged? a. Use the inhaled corticosteroid when shortness of breath occurs. b. Inhale slowly and deeply when using the dry powder inhaler (DPI). c. Hold your breath for 5 seconds after using the bronchodilator inhaler. d. Tremors are an expected side effect of rapidly acting bronchodilators.

D Tremors are a common side effect of short-acting b2-adrenergic (SABA) medications and not a reason to avoid using the SABA inhaler. Inhaled corticosteroids do not act rapidly to reduce dyspnea. Rapid inhalation is needed when using a DPI. The patient should hold the breath for 10 seconds after using inhalers.

Which consideration is the most important in managing tuberculosis (TB) in children? a. Skin testing annually b. Pharmacotherapy c. Adequate nutrition d. Adequate hydration

B Drug therapy for TB includes isoniazid, rifampin, and pyrazinamide daily for 2 months and two or three times a week for the remaining 4 months. Pharmacotherapy is the most important intervention for TB.

Which assessment finding in a patient who has received omalizumab (Xolair) is most important to report immediately to the health care provider? a. Pain at injection site b. Flushing and dizziness c. Peak flow reading 75% of normal d. Respiratory rate 24 breaths/minute

B Flushing and dizziness may indicate that the patient is experiencing an anaphylactic reaction, and immediate intervention is needed. The other information should also be reported, but do not indicate possibly life-threatening complications of omalizumab therapy.

Which finding by the nurse for a patient with a nursing diagnosis of impaired gas exchange will be most useful in evaluating the effectiveness of treatment? a. Even, unlabored respirations b. Pulse oximetry reading of 92% c. Absence of wheezes or crackles d. Respiratory rate of 18 breaths/min

B For the nursing diagnosis of impaired gas exchange, the best data for evaluation are arterial blood gases (ABGs) or pulse oximetry. The other data may indicate either improvement or impending respiratory failure caused by fatigue.

The nurse is assessing a child with acute epiglottitis. Examining the child's throat by using a tongue depressor might precipitate which symptom or condition? a. Inspiratory stridor b. Complete obstruction c. Sore throat d. Respiratory tract infection

B If a child has acute epiglottitis, examination of the throat may cause complete obstruction and should be performed only when immediate intubation can take place. Stridor is aggravated when a child with epiglottitis is supine. Sore throat and pain on swallowing are early signs of epiglottitis. Epiglottitis is caused by H. influenzae in the respiratory tract.

A patient with chronic obstructive pulmonary disease (COPD) has poor gas exchange. Which action by the nurse would support the patient's ventilation? a. Have the patient rest in bed with the head elevated to 15 to 20 degrees. b. Encourage the patient to sit up at the bedside in a chair and lean forward. c. Ask the patient to rest in bed in a high-Fowler's position with the knees flexed. d. Place the patient in the Trendelenburg position with pillows behind the head.

B Patients with COPD improve the mechanics of breathing by sitting up in the "tripod" position. Resting in bed with the head elevated in a semi-Fowler's position would be an alternative position if the patient was confined to bed, but sitting in a chair allows better ventilation. The Trendelenburg position or sitting upright in bed with the knees flexed would decrease the patient's ability to ventilate well.

The home health nurse is visiting a patient with chronic obstructive pulmonary disease (COPD). Which nursing action is appropriate to implement for a nursing diagnosis of impaired breathing pattern related to anxiety? a. Titrate O2 to keep saturation at least 90%. b. Teach the patient how to use pursed-lip breathing. c. Discuss a high-protein, high-calorie diet with the patient. d. Suggest the use of over-the-counter sedative medications.

B Pursed-lip breathing techniques assist in prolonging the expiratory phase of respiration and decrease air trapping. There is no indication that the patient requires O2 therapy or an improved diet. Sedative medications should be avoided because they decrease respiratory drive.

A patient is receiving 35% O2 via a Venturi mask. To ensure the correct amount of O2 delivery, which action by the nurse is important? a. Teach the patient to keep the mask on during meals. b. Keep the air entrainment ports clean and unobstructed. c. Give a high enough flow rate to keep the bag from collapsing. d. Drain moisture condensation from the corrugated tubing every hour.

B The air entrainment ports regulate the O2 percentage delivered to the patient, so they must be unobstructed. The other options refer to other types of O2 devices. A high O2 flow rate is needed when giving O2by partial rebreather or nonrebreather masks. Draining O2 tubing is necessary when caring for a patient receiving mechanical ventilation. The mask can be removed or changed to a nasal cannula at a prescribed setting when the patient eats.

The nurse reviews the medication administration record (MAR) for a patient having an acute asthma attack. Which medication should the nurse administer first? a. Methylprednisolone (Solu-Medrol) 60 mg IV b. Albuterol (Ventolin HFA) 2.5 mg per nebulizer c. Salmeterol (Serevent) 50 mcg per dry-powder inhaler (DPI) d. Ipratropium (Atrovent) 2 puffs per metered-dose inhaler (MDI)

A Albuterol is a rapidly acting bronchodilator and is the first-line medication to reverse airway narrowing in acute asthma attacks. The other medications work more slowly.

A patient who is experiencing an acute asthma attack is admitted to the emergency department. Which assessment should the nurse complete first? a. Listen to the patient's breath sounds. b. Ask about inhaled corticosteroid use. c. Determine when the dyspnea started. d. Obtain the forced expiratory volume (FEV) flow rate.

A Assessment of the patient's breath sounds will help determine how effectively the patient is ventilating and whether rapid intubation may be necessary. The length of time the attack has persisted is not as important as determining the patient's status at present. Most patients having an acute attack will be unable to cooperate with an FEV measurement. It is important to know about the medications the patient is using but not as important as assessing the breath sounds.

1. The nurse notes new onset confusion in an older patient who is normally alert and oriented. In which order should the nurse take the following actions? (Put a comma and a space between each answer choice [A, B, C, D].) a. Obtain the O2 saturation. b. Check the patient's pulse rate. c. Document the change in status. d. Notify the health care provider.

ANS: A, B, D, C Assessment for physiologic causes of new onset confusion such as pneumonia, infection, or perfusion problems should be the first action by the nurse. Airway and oxygenation should be assessed first, then circulation. After assessing the patient, the nurse should notify the health care provider. Finally, documentation of the assessments and care should be done. DIF: Cognitive Level: Analyze (analysis) REF: 502 OBJ: Special Questions: Prioritization TOP: Nursing Process: Implementation MSC: NCLEX: Physiological Integrity

The nurse is caring for a 5-year-old child who is scheduled for a tonsillectomy in 2 hours. Which action should the nurse include in the child's postoperative care plan? (Select all that apply.) a. Notify the surgeon if the child swallows frequently. b. Apply a heat collar to the child for pain relief. c. Place the child on the abdomen until fully wake. d. Allow the child to have diluted juice after the procedure. e. Encourage the child to cough frequently.

A, C, D Frequent swallowing is a sign of bleeding in children after a tonsillectomy. The child should be placed on the abdomen or the side to facilitate drainage. The child can drink diluted juice, cool water, or popsicles after the procedure. An ice collar should be used after surgery. Frequent coughing and nose blowing should be avoided.

1. The clinic nurse is teaching a patient with acute sinusitis. Which interventions should the nurse plan to include in the teaching session (select all that apply)? a. Decongestants can be used to relieve swelling. b. Blowing the nose should be avoided to decrease the nosebleed risk. c. Taking a hot shower will increase sinus drainage and decrease pain. d. Saline nasal spray can be made at home and used to wash out secretions. e. You will be more comfortable if you keep your head in an upright position.

ACDE The steam and heat from a shower will help thin secretions and improve drainage. Decongestants can be used to relieve swelling. Patients can use either over-the-counter sterile saline solutions or home-prepared saline solutions to thin and remove secretions. Maintaining an upright posture decreases sinus pressure and the resulting pain. Blowing the nose after a hot shower or using the saline spray is recommended to expel secretions. DIF: Cognitive Level: Apply (application) REF: 483 TOP: Nursing Process: Implementation MSC: NCLEX: Physiological Integrity

1. The following four patients arrive in the emergency department (ED) after a motor vehicle collision. In which order should the nurse assess them? (Put a comma and a space between each answer choice [A, B, C, D, E].) a. A 74-yr-old patient with palpitations and chest pain b. A 43-yr-old patient complaining of 7/10 abdominal pain c. A 21-yr-old patient with multiple fractures of the face and jaw d. A 37-yr-old patient with a misaligned lower left leg with intact pulses

ANS: C, A, B, D The highest priority is to assess the 21-yr-old patient for airway obstruction, which is the most life- threatening injury. The 74-yr-old patient may have chest pain from cardiac ischemia and should be assessed and have diagnostic testing for this pain. The 43-yr-old patient may have abdominal trauma or bleeding and should be seen next to assess circulatory status. The 37-yr-old patient appears to have a possible fracture of the left leg and should be seen soon, but this patient has the least life-threatening injury.

1. A 198-lb patient is to receive a dobutamine infusion at 5 mcg/kg/min. The label on the infusion bag states: dobutamine 250 mg in 250 mL of normal saline. When setting the infusion pump, the nurse will set the infusion rate at how many milliliters per hour?

ANS: 27 To administer the dobutamine at the prescribed rate of 5 mcg/kg/min from a concentration of 250 mg in 250 mL, the nurse will need to infuse 27 mL/hr. DIF: Cognitive Level: Apply (application) REF: 1599 TOP: Nursing Process: Implementation MSC: NCLEX: Physiological Integrity

The nurse assumes care of a patient who just returned from surgery for a total laryngectomy and radical neck dissection and notes the following problems. In which order should the nurse address the problems? (Put a comma and a space between each answer choice [A, B, C, D].) a. The patient is in a side-lying position with the head of the bed flat. b. The patient is coughing blood-tinged secretions from the tracheostomy. c. The nasogastric (NG) tube is disconnected from suction and clamped off. d. The wound drain in the neck incision contains 200 mL of bloody drainage.

ANS: A, B, D, C The patient should first be placed in a semi-Fowler's position to maintain the airway and reduce incisional swelling. The blood-tinged secretions may obstruct the airway, so suctioning is the next appropriate action. Then the wound drain should be drained because the 200 mL of drainage will decrease the amount of suction in the wound drain and could lead to incisional swelling and poor healing. Finally, the NG tube should be reconnected to suction to prevent gastric dilation, nausea, and vomiting. DIF: Cognitive Level: Analyze (analysis) REF: 494 OBJ: Special Questions: Prioritization TOP: Nursing Process: Implementation MSC: NCLEX: Physiological Integrity

The nurse is caring for a patient who has an intraortic balloon pump (IABP) following a massive heart attack. When assessing the patient, the nurse notices blood backing up into the IABP catheter. In which order should the nurse take the following actions? (Put a comma and a space between each answer choice [A, B, C, D].) a. Ensure that the IABP console has turned off. b. Assess the patient's vital signs and orientation. c. Obtain supplies for insertion of a new IABP catheter. d. Notify the health care provider of the IABP malfunction.

ANS: A, B, D, C Blood in the IABP catheter indicates a possible tear in the balloon. The console will shut off automatically to prevent complications such as air embolism. Next, the nurse will assess the patient and communicate with the health care provider about the patient's assessment and the IABP problem. Finally, supplies for insertion of a new IABP catheter may be needed, based on the patient assessment and the decision of the health care provider.

When assisting with oral intubation of a patient who is having respiratory distress, in which order will the nurse take these actions? (Put a comma and a space between each answer choice [A, B, C, D, E].) a. Obtain a portable chest-x-ray. b. Position the patient in the supine position. c. Inflate the cuff of the endotracheal tube after insertion. d. Attach an end-tidal CO2 detector to the endotracheal tube. e. Oxygenate the patient with a bag-valve-mask device for several minutes.

ANS: E, B, C, D, A The patient is pre-oxygenated with a bag-valve-mask system for 3 to 5 minutes before intubation and then placed in a supine position. Following the intubation, the cuff on the endotracheal tube is inflated to occlude and protect the airway. Tube placement is assessed first with an end-tidal CO2 sensor, then with a chest x-ray.

1. The health care provider orders the following interventions for a 67-kg patient who has septic shock with a blood pressure of 70/42 mm Hg and O2 saturation of 90% on room air. In which order will the nurse implement the actions? (Put a comma and a space between each answer choice [A, B, C, D, E].) a. Give vancomycin 1 g IV. b. Obtain blood and urine cultures c. Start norepinephrine 0.5 mcg/min. d. Infuse normal saline 2000 mL over 30 minutes. e. Titrate oxygen administration to keep O2 saturation above 95%.

ANS: E, D, C, B, A The initial action for this hypotensive and hypoxemic patient should be to improve the O2 saturation, followed by infusion of IV fluids and vasopressors to improve perfusion. Cultures should be obtained before giving antibiotics. DIF: Cognitive Level: Analyze (analysis) REF: 1600 OBJ: Special Questions: Prioritization TOP: Nursing Process: Implementation MSC: NCLEX: Physiological Integrity

18. An unresponsive 79-yr-old patient is admitted to the emergency department (ED) during a summer heat wave. The patient's core temperature is 105.4° F (40.8° C), blood pressure (BP) is 88/50 mm Hg, and pulse is 112 beats/min. The nurse will plan to a. apply wet sheets and a fan to the patient. b. provide O2 at 2 L/min with a nasal cannula. c. start lactated Ringer's solution at 1000 mL/hr. d. give acetaminophen (Tylenol) rectal suppository.

ANS: A The priority intervention is to cool the patient. Antipyretics are not effective in decreasing temperature in heat stroke and 100% O2 should be given, which requires a high flow rate through a non-rebreather mask. An older patient would be at risk for developing complications such as pulmonary edema if given fluids at 1000 mL/hr.

The nurse is taking care of a 7-year-old child with a skin rash called a papule. Which clinical finding should the nurse expect to assess with this type of skin rash? a. A lesion that is elevated, palpable, firm, and circumscribed; less than 1 cm in diameter b. A lesion that is elevated, flat-topped, firm, rough, and superficial; greater than 1 cm in diameter c. An elevated lesion, firm, circumscribed, palpable; 1 to 2 cm in diameter d. An elevated lesion, circumscribed, filled with serous fluid; less than 1 cm in diameter

ANS: A A papule is elevated; palpable; firm; circumscribed; less than 1 cm in diameter; and brown, red, pink, tan, or bluish red. A plaque is an elevated, flat-topped, firm, rough, superficial papule greater than 1 cm in diameter. It may be coalesced papules. A nodule is elevated, 1 to 2 cm in diameter, firm, circumscribed, palpable, and deeper in the dermis than a papule. A vesicle is elevated, circumscribed, superficial, less than 1 cm in diameter, and filled with serous fluid.

The nurse is caring for a patient who arrived in the emergency department with acute respiratory distress. Which assessment finding by the nurse requires the most rapid action? a. The patient's PaO2 is 45 mm Hg. b. The patient's PaCO2 is 33 mm Hg. c. The patient's respirations are shallow. d. The patient's respiratory rate is 32 breaths/min.

ANS: A The PaO2 indicates severe hypoxemia and respiratory failure. Rapid action is needed to prevent further deterioration of the patient. Although the shallow breathing, rapid respiratory rate, and low PaCO2 also need to be addressed, the most urgent problem is the patient's poor oxygenation. DIF: Cognitive Level: Analyze (analysis)

15. A patient with septic shock has a BP of 70/46 mm Hg, pulse of 136 beats/min, respirations of 32 breaths/min, temperature of 104°F, and blood glucose of 246 mg/dL. Which intervention ordered by the health care provider should the nurse implement first? a. Give normal saline IV at 500 mL/hr. b. Give acetaminophen (Tylenol) 650 mg rectally. c. Start insulin drip to maintain blood glucose at 110 to 150 mg/dL. d. Start norepinephrine to keep systolic blood pressure above 90 mm Hg.

ANS: A Because of the decreased preload associated with septic shock, fluid resuscitation is the initial therapy. The other actions also are appropriate, and should be initiated quickly as well. DIF: Cognitive Level: Analyze (analysis) REF: 1600 OBJ: Special Questions: Prioritization TOP: Nursing Process: Implementation MSC: NCLEX: Physiological Integrity

17. The nurse is caring for a patient who has septic shock. Which assessment finding is most important for the nurse to report to the health care provider? a. Skin cool and clammy b. Heart rate of 118 beats/min c. Blood pressure of 92/56 mm Hg d. O2 saturation of 93% on room air

ANS: A Because patients in the early stage of septic shock have warm and dry skin, the patient's cool and clammy skin indicates that shock is progressing. The other information will also be reported, but does not indicate deterioration of the patient's status. DIF: Cognitive Level: Analyze (analysis) REF: 1594 OBJ: Special Questions: Prioritization TOP: Nursing Process: Assessment MSC: NCLEX: Physiological Integrity

After being hit by a baseball, a patient arrives in the emergency department with a possible nasal fracture. Which finding by the nurse is most important to report to the health care provider? a. Clear nasal drainage b. Complaint of nasal pain c. Bilateral nose swelling and bruising d. Inability to breathe through the nose

ANS: A Clear nasal drainage may indicate a meningeal tear with leakage of cerebrospinal fluid. This would place the patient at risk for complications such as meningitis. The other findings are typical with a nasal fracture and do not indicate any complications. DIF: Cognitive Level: Analyze (analysis) REF: 476 OBJ: Special Questions: Prioritization TOP: Nursing Process: Assessment MSC: NCLEX: Physiological Integrity

A nurse is caring for a patient who is orally intubated and receiving mechanical ventilation. To decrease the risk for ventilator-associated pneumonia, which action will the nurse include in the plan of care? a. Elevate head of bed to 30 to 45 degrees. b. Give enteral feedings at no more than 10 mL/hr. c. Suction the endotracheal tube every 2 to 4 hours. d. Limit the use of positive end-expiratory pressure.

ANS: A Elevation of the head decreases the risk for aspiration. Positive end-expiratory pressure is frequently needed to improve oxygenation in patients receiving mechanical ventilation. Suctioning should be done only when the patient assessment indicates that it is necessary. Enteral feedings should provide adequate calories for the patient's high energy needs. DIF: Cognitive Level: Apply (application)

22. The following interventions are ordered by the health care provider for a patient who has respiratory distress and syncope after eating strawberries. Which will the nurse complete first? a. Give epinephrine. b. Administer diphenhydramine. c. Start continuous ECG monitoring. d. Draw blood for complete blood count (CBC)

ANS: A Epinephrine rapidly causes peripheral vasoconstriction, dilates the bronchi, and blocks the effects of histamine and reverses the vasodilation, bronchoconstriction, and histamine release that cause the symptoms of anaphylaxis. The other interventions are also appropriate but would not be the first ones completed. DIF: Cognitive Level: Analyze (analysis) REF: 1599 OBJ: Special Questions: Prioritization TOP: Nursing Process: Implementation MSC: NCLEX: Physiological Integrity

1. A 78-kg patient with septic shock has a pulse rate of 120 beats/min with low central venous pressure and pulmonary artery wedge pressure. Urine output has been 30 mL/hr for the past 3 hours. Which order by the health care provider should the nurse question? a. Administer furosemide (Lasix) 40 mg IV. b. Increase normal saline infusion to 250 mL/hr. c. Give hydrocortisone (Solu-Cortef) 100 mg IV. d. Titrate norepinephrine to keep systolic blood pressure (BP) above 90 mm Hg.

ANS: A Furosemide will lower the filling pressures and renal perfusion further for the patient with septic shock. Patients in septic shock require large amounts of fluid replacement. If the patient remains hypotensive after initial volume resuscitation with minimally 30 mL/kg, vasopressors such as norepinephrine may be added. IV corticosteroids may be considered for patients in septic shock who cannot maintain an adequate BP with vasopressor therapy despite fluid resuscitation. DIF: Cognitive Level: Apply (application) REF: 1600 TOP: Nursing Process: Implementation MSC: NCLEX: Physiological Integrity

A patient with acute respiratory distress syndrome (ARDS) and acute kidney injury has the following drugs ordered. Which drug should the nurse discuss with the health care provider before giving? a. gentamicin 60 mg IV b. pantoprazole (Protonix) 40 mg IV c. sucralfate (Carafate) 1 g per nasogastric tube d. methylprednisolone (Solu-Medrol) 60 mg IV

ANS: A Gentamicin, which is one of the aminoglycoside antibiotics, is potentially nephrotoxic, and the nurse should clarify the drug and dosage with the health care provider before administration. The other drugs are appropriate for the patient with ARDS. DIF: Cognitive Level: Apply (application)

The nurse is caring for a 5-year-old child with impetigo contagiosa. The parents ask the nurse what will happen to their child's skin after the infection has subsided and healed. Which answer should the nurse give? a. There will be no scarring. b. There may be some pigmented spots. c. It is likely there will be some slightly depressed scars. d. There will be some atrophic white scars.

ANS: A Impetigo contagiosa tends to heal without scarring unless a secondary infection occurs.

When admitting a patient with possible respiratory failure and a high PaCO2, which assessment information should be immediately reported to the health care provider? a. The patient is very somnolent. b. The patient complains of weakness. c. The patient's blood pressure is 164/98. d. The patient's oxygen saturation is 90%.

ANS: A Increasing somnolence will decrease the patient's respiratory rate and further increase the PaCO2 and respiratory failure. Rapid action is needed to prevent respiratory arrest. An SpO2 of 90%, weakness, and elevated blood pressure all require ongoing monitoring but are not indicators of possible impending respiratory arrest. DIF: Cognitive Level: Analyze (analysis)

Which is usually the only symptom of pediculosis capitis (head lice)? a. Itching b. Vesicles c. Scalp rash d. Localized inflammatory response

ANS: A Itching is generally the only manifestation of pediculosis capitis (head lice). Diagnosis is made by observation of the white eggs (nits) on the hair shaft. Vesicles, scalp rash, and localized inflammatory response are not symptoms of head lice.

The nurse is conducting a staff in-service on appearance of childhood skin conditions. Lymphangitis ("streaking") is frequently seen in which condition? a. Cellulitis b. Folliculitis c. Impetigo contagiosa d. Staphylococcal scalded skin

ANS: A Lymphangitis is frequently seen in cellulitis. If it is present, hospitalization is usually required for parenteral antibiotics. Lymphangitis is not associated with folliculitis, impetigo, or staphylococcal scalded skin.

The nurse is caring for a mechanically ventilated patient with a cuffed tracheostomy tube. Which action by the nurse would determine if the cuff has been properly inflated? a. Use a hand-held manometer to measure cuff pressure. b. Review the health record for the prescribed cuff pressure. c. Suction the patient through a fenestrated inner cannula to clear secretions. d. Insert the decannulation plug before removing the nonfenestrated inner cannula.

ANS: A Measurement of cuff pressure using a manometer to ensure that cuff pressure is 20 mm Hg or lower will avoid compression of the tracheal wall and capillaries. Never insert the decannulation plug in a tracheostomy tube until the cuff is deflated and the nonfenestrated inner cannula is removed. Otherwise, the patient's airway is occluded. A health care provider's order is not required to determine safe cuff pressure. A nonfenestrated inner cannula must be used to suction a patient to prevent tracheal damage occurring from the suction catheter passing through the fenestrated openings. DIF: Cognitive Level: Apply (application) REF: 487 TOP: Nursing Process: Implementation MSC: NCLEX: Physiological Integrity

A patient who is receiving positive pressure ventilation is scheduled for a spontaneous breathing trial (SBT). Which finding by the nurse is most important to discuss with the health care provider before starting the SBT? a. New ST segment elevation is noted on the cardiac monitor. b. Enteral feedings are being given through an orogastric tube. c. Scattered rhonchi are heard when auscultating breath sounds. d. HYDROmorphone (Dilaudid) is being used to treat postoperative pain.

ANS: A Myocardial ischemia is a contraindication for ventilator weaning. The ST segment elevation is an indication that weaning should be postponed until further investigation and/or treatment for myocardial ischemia can be done. The other information will also be shared with the health care provider, but ventilator weaning can proceed when opioids are used for pain management, abnormal lung sounds are present, or enteral feedings are being used.

12. A patient arrives in the emergency department (ED) several hours after taking "25 to 30" acetaminophen (Tylenol) tablets. Which action will the nurse plan to take? a. Give N-acetylcysteine. b. Discuss the use of chelation therapy. c. Start oxygen using a non-rebreather mask. d. Have the patient drink large amounts of water.

ANS: A N-acetylcysteine is the recommended treatment to prevent liver damage after acetaminophen overdose. The other actions might be used for other types of poisoning, but they will not be appropriate for a patient with acetaminophen poisoning.

The nurse is caring for a patient who has just had a thoracentesis. Which assessment information obtained by the nurse is a priority to communicate to the health care provider? a. O2 saturation is 88%. b. Blood pressure is 155/90 mm Hg. c. Pain level is 5 (on 0 to 10 scale) with a deep breath. d. Respiratory rate is 24 breaths/minute when lying flat.

ANS: A O2 saturation would be expected to improve after a thoracentesis. A saturation of 88% indicates that a complication such as pneumothorax may be occurring. The other assessment data also indicate a need for ongoing assessment or intervention, but the low O2 saturation is the priority. DIF: Cognitive Level: Analyze (analysis) REF: 527 OBJ: Special Questions: Prioritization TOP: Nursing Process: Assessment MSC: NCLEX: Physiological Integrity

The nurse needs to obtain blood for ongoing assessment of a high-risk newborn's progress. Which tests should the nurse monitor? (Select all that apply.) a. Blood glucose b. Complete blood count (CBC) c. Calcium d. Serum electrolytes e. Neonatal prothrombin time (PTT)

ANS: A, C, D The most common blood tests done on high-risk newborns are blood glucose, bilirubin, calcium, hematocrit, serum electrolytes, and blood gases. Hematocrits rather than CBCs are performed. This will monitor the red cell volume. Neonatal prothrombin time (PTT) is not a test.

. The nurse is caring for a hospitalized older patient who has nasal packing in place after a nosebleed. Which assessment finding will require the most immediate action by the nurse? a. The oxygen saturation is 89%. b. The nose appears red and swollen. c. The patient reports level 8 (0 to 10 scale) pain. d. The patient's temperature is 100.1° F (37.8° C).

ANS: A Older patients with nasal packing are at risk of aspiration or airway obstruction. An O2 saturation of 89% should alert the nurse to further assess for these complications. The other assessment data also indicate a need for nursing action but not as immediately as the low O2 saturation. DIF: Cognitive Level: Analyze (analysis) REF: 476 OBJ: Special Questions: Prioritization TOP: Nursing Process: Assessment MSC: NCLEX: Physiological Integrity

8. A nurse obtains a health history from a patient who has a 35 pack-year smoking history. The patient complains of hoarseness and tightness in the throat and difficulty swallowing. Which question is important for the nurse to ask? a. "How much alcohol do you drink in an average week?" b. "Do you have a family history of head or neck cancer?" c. "Have you had frequent streptococcal throat infections?" d. "Do you use antihistamines for upper airway congestion?"

ANS: A Prolonged alcohol use and smoking are associated with the development of laryngeal cancer, which the patient's symptoms and history suggest. Family history is not a risk factor for head or neck cancer. Frequent antihistamine use would be asked about if the nurse suspected allergic rhinitis, but the patient's symptoms are not suggestive of this diagnosis. Patients with streptococcal throat infections will also have pain and a fever. DIF: Cognitive Level: Apply (application) REF: 491 TOP: Nursing Process: Assessment MSC: NCLEX: Health Promotion and Maintenance

9. When planning the response to the potential use of smallpox as a biological weapon, the emergency department (ED) nurse manager will plan to obtain adequate quantities of a. vaccine. c. antibiotics. b. atropine. d. whole blood.

ANS: A Smallpox infection can be prevented or ameliorated by the administration of vaccine given rapidly after exposure. The other interventions would be helpful for other agents of terrorism but not for smallpox.

A nurse is caring for a patient with acute respiratory distress syndrome (ARDS) who is receiving mechanical ventilation using synchronized intermittent mandatory ventilation (SIMV). The settings include fraction of inspired oxygen (FIO2) of 80%, tidal volume of 450, rate of 16/minute, and positive end-expiratory pressure (PEEP) of 5 cm. Which assessment finding is most important for the nurse to report to the health care provider? a. O2 saturation of 99% b. Heart rate 106 beats/minute c. Crackles audible at lung bases d. Respiratory rate 22 breaths/minute

ANS: A The FIO2 of 80% increases the risk for O2 toxicity. Because the patient's O2 saturation is 99%, a decrease in FIO2 is indicated to avoid toxicity. The other patient data would be typical for a patient with ARDS and would not be the most important data to report to the health care provider. DIF: Cognitive Level: Analyze (analysis)

The nurse is caring for a patient who is intubated and receiving positive pressure ventilation to treat acute respiratory distress syndrome (ARDS). Which finding is most important to report to the health care provider? a. Red-brown drainage from nasogastric tube b. Blood urea nitrogen (BUN) level 32 mg/dL c. Scattered coarse crackles heard throughout lungs d. Arterial blood gases: pH of 7.31, PaCO2 of 50, and PaO2 of 68

ANS: A The nasogastric drainage indicates possible gastrointestinal bleeding or stress ulcer and should be reported. The pH and PaCO2 are slightly abnormal, but current guidelines advocating for permissive hypercapnia indicate that these would not indicate an immediate need for a change in therapy. The BUN is slightly elevated but does not indicate an immediate need for action. Adventitious breath sounds are commonly heard in patients with ARDS. DIF: Cognitive Level: Analyze (analysis)

A nurse is caring for an obese patient with right lower lobe pneumonia. Which position will be best to improve gas exchange? a. On the left side b. On the right side c. In the tripod position d. In the high-Fowler's position

ANS: A The patient should be positioned with the "good" lung in the dependent position to improve the match between ventilation and perfusion. The obese patient's abdomen will limit respiratory excursion when sitting in the high-Fowler's or tripod positions. DIF: Cognitive Level: Apply (application)

Which patient in the ear, nose, and throat (ENT) clinic should the nurse assess first? a. A patient who is complaining of a sore throat and has a muffled voice b. A patient who has a "scratchy throat" and a positive rapid strep antigen test c. A patient who is receiving radiation for throat cancer and has severe fatigue d. A patient with a history of a total laryngectomy whose stoma is red and inflamed

ANS: A The patient's clinical manifestation of a muffled voice suggests a possible peritonsillar abscess that could lead to an airway obstruction requiring rapid assessment and potential treatment. The other patients do not have diagnoses or symptoms that indicate any life-threatening problems. DIF: Cognitive Level: Analyze (analysis) REF: 484 OBJ: Special Questions: Prioritization | Special Questions: Multiple Patients TOP: Nursing Process: Assessment MSC: NCLEX: Physiological Integrity

13. A triage nurse in a busy emergency department (ED) assesses a patient who complains of 7/10 abdominal pain and states, "I had a temperature of 103.9° F (39.9° C) at home." The nurse's first action should be to a. assess the patient's current vital signs. b. give acetaminophen (Tylenol) per agency protocol. c. ask the patient to provide a clean-catch urine for urinalysis. d. tell the patient that it will be 1 to 2 hours before seeing a health care provider.

ANS: A The patient's pain and statement about an elevated temperature indicate that the nurse should obtain vital signs before deciding how rapidly the patient should be seen by the health care provider. A urinalysis may be appropriate, but this would be done after the vital signs are taken. The nurse will not give acetaminophen before confirming a current temperature elevation.

15. The following interventions are part of the emergency department (ED) protocol for a patient who has been admitted with multiple bee stings to the hands. Which action should the nurse take first? a. Remove the patient's rings. b. Apply ice packs to both hands. c. Apply calamine lotion to itching areas. d. Give diphenhydramine (Benadryl) 50 mg PO.

ANS: A The patient's rings should be removed first because it might not be possible to remove them if swelling develops. The other orders should also be implemented as rapidly as possible after the nurse has removed the jewelry.

. A patient who was admitted the previous day with pneumonia complains of a sharp pain of 7 (on 0 to 10 scale) "whenever I take a deep breath." Which action will the nurse take next? a. Auscultate for breath sounds. b. Administer the PRN morphine. c. Have the patient cough forcefully. d. Notify the patient's health care provider.

ANS: A The patient's statement indicates that pleurisy or a pleural effusion may have developed and the nurse will need to listen for a pleural friction rub and decreased breath sounds. Assessment should occur before administration of pain medications. The patient is unlikely to be able to cough forcefully until pain medication has been administered. The nurse will want to obtain more assessment data before calling the health care provider. DIF: Cognitive Level: Analyze (analysis) REF: 528 OBJ: Special Questions: Prioritization TOP: Nursing Process: Assessment MSC: NCLEX: Physiological Integrity

19. An unresponsive patient is admitted to the emergency department (ED) after falling through the ice while ice skating. Which assessment will the nurse obtain first? a. Pulse c. Breath sounds b. Heart rhythm d. Body temperature

ANS: A The priority assessment in an unresponsive patient relates to CAB (circulation, airway, breathing) so a pulse check should be performed first. While assessing the pulse, the nurse should look for signs of breathing. The other data will also be collected rapidly but are not as essential as determining if there is a pulse.

When prone positioning is used for a patient with acute respiratory distress syndrome (ARDS), which information obtained by the nurse indicates that the positioning is effective? a. The patient's PaO2 is 89 mm Hg, and the SaO2 is 91%. b. Endotracheal suctioning results in clear mucous return. c. Sputum and blood cultures show no growth after 48 hours. d. The skin on the patient's back is intact and without redness.

ANS: A The purpose of prone positioning is to improve the patient's oxygenation as indicated by the PaO2 and SaO2. The other information will be collected but does not indicate whether prone positioning has been effective. DIF: Cognitive Level: Apply (application)

20. Following an earthquake, patients are triaged by emergency medical personnel and transported to the emergency department (ED). Which patient will the nurse need to assess first? a. A patient with a red tag c. A patient with a black tag b. A patient with a blue tag d. A patient with a yellow tag

ANS: A The red tag indicates a patient with a life-threatening injury requiring rapid treatment. The other tags indicate patients with less urgent injuries or those who are likely to die.

The nurse completes discharge instructions for a patient with a total laryngectomy. Which statement by the patient indicates that additional instruction is needed? a. "I must keep the stoma covered with an occlusive dressing." b. "I need to have smoke and carbon monoxide detectors installed." c. "I can participate in my prior fitness activities except swimming." d. "I should wear a Medic-Alert bracelet to identify me as a neck breather."

ANS: A The stoma may be covered with clothing or a loose dressing, but this is not essential. An occlusive dressing will completely block the patient's airway. The other patient comments are all accurate and indicate that the teaching has been effective. DIF: Cognitive Level: Apply (application) REF: 495 TOP: Nursing Process: Evaluation MSC: NCLEX: Physiological Integrity

Which is an important nursing consideration in the care of the newborn with PKU? a. Suggest ways to make formula more palatable. b. Teach proper administration of phenylalanine hydroxylase. c. Encourage the breastfeeding mother to adhere to low-phenylalanine diet. d. Give reassurance that dietary restrictions are a temporary inconvenience.

ANS: A To achieve optimal metabolic control, a restricted phenylalanine diet will probably be required for virtually all individuals with classic PKU throughout life. The nurse and nutritionist should work with families to make the formula more palatable for the newborn. Phenylalanine hydroxylase is not effective because it cannot act within the cell where phenylalanine is metabolized. Partial breastfeeding may be possible, but only with extremely careful monitoring of the newborn's blood levels. According to the latest research, lifelong dietary restriction may be necessary.

Which is an important nursing consideration when caring for a child with herpetic gingivostomatitis (HGS)? a. Apply topical anesthetics before eating. b. Drink from a cup, not a straw. c. Wait to brush teeth until lesions are sufficiently healed. d. Explain to parents how this is sexually transmitted.

ANS: A Treatment for HGS is aimed at relief of pain. Drinking bland fluids through a straw helps avoid painful lesions. Mouth care is encouraged with a soft toothbrush. HGS is usually caused by herpes simplex virus type 1, which is not associated with sexual transmission.

The nurse is caring for a school-age child with a tinea capitis (ringworm) infection. What should the nurse expect the therapeutic management of this child to include? a. Administering oral griseofulvin b. Administering topical or oral antibiotics c. Applying topical sulfonamides d. Applying Burow solution compresses to affected area

ANS: A Treatment with the antifungal agent griseofulvin is part of the treatment for the fungal disease ringworm. Oral griseofulvin therapy frequently continues for weeks or months. Antibiotics, sulfonamides, and Burow solution are not effective in fungal infections.

48. The nurse is caring for a patient who has a right-sided chest tube after a right lower lobectomy. Which nursing action can the nurse delegate to the unlicensed assistive personnel (UAP)? a. Document the amount of drainage every 8 hours. b. Obtain samples of drainage for culture from the system. c. Assess patient pain level associated with the chest tube. d. Check the water-seal chamber for the correct fluid level.

ANS: A UAP education includes documentation of intake and output. The other actions are within the scope of practice and education of licensed nursing personnel. DIF: Cognitive Level: Apply (application) REF: 526 OBJ: Special Questions: Delegation TOP: Nursing Process: Planning MSC: NCLEX: Safe and Effective Care Environment

3. After the return of spontaneous circulation following the resuscitation of a patient who had a cardiac arrest, therapeutic hypothermia is ordered. Which action will the nurse include in the plan of care? a. Initiate cooling per protocol. b. Avoid the use of sedative drugs. c. Check mental status every 15 minutes. d. Rewarm if temperature is below 91° F (32.8° C).

ANS: A When therapeutic hypothermia is used postresuscitation, external cooling devices or cold normal saline infusions are used to rapidly lower body temperature to 89.6° F to 93.2° F (32° C to 34° C). Because hypothermia will decrease brain activity, assessing mental status every 15 minutes is not done at this stage. Sedative drugs are given during therapeutic hypothermia.

Which statement by the patient indicates that teaching has been effective for a patient scheduled for radiation therapy of the larynx? a. "I will need to buy a water bottle to carry with me." b. "I should not use any lotions on my neck and throat." c. "Until the radiation is complete, I may have diarrhea." d. "Alcohol-based mouthwashes will help clean my mouth."

ANS: A Xerostomia can be partially alleviated by drinking fluids at frequent intervals. Radiation will damage tissues at the site being radiated but should not affect the abdominal organs, so loose stools are not a usual complication of head and neck radiation therapy. Frequent oral rinsing with non-alcohol-based rinses is recommended. Prescribed lotions and sunscreen may be used on radiated skin, although they should not be used just before the radiation therapy. DIF: Cognitive Level: Apply (application) REF: 495 TOP: Nursing Process: Evaluation MSC: NCLEX: Physiological Integrity

1. Which interventions will the nurse plan for a comatose patient who is to begin therapeutic hypothermia (select all that apply)? a. Assist with endotracheal intubation. b. Insert an indwelling urinary catheter. c. Begin continuous cardiac monitoring. d. Obtain an order to restrain the patient. e. Prepare to give sympathomimetic drugs.

ANS: A, B, C Cooling can produce dysrhythmias, so the patient's heart rhythm should be continuously monitored and dysrhythmias treated if necessary. Bladder catheterization and endotracheal intubation are needed during cooling. Sympathomimetic drugs tend to stimulate the heart and increase the risk for fatal dysrhythmias such as ventricular fibrillation. Patients receiving therapeutic hypothermia are comatose or do not follow commands so restraints are not indicated.

2. Which preventive actions by the nurse will help limit the development of systemic inflammatory response syndrome (SIRS) in patients admitted to the hospital (select all that apply)? a. Ambulate postoperative patients as soon as possible after surgery. b. Use aseptic technique when manipulating invasive lines or devices. c. Remove indwelling urinary catheters as soon as possible after surgery. d. Administer prescribed antibiotics within 1 hour for patients with possible sepsis. e. Advocate for parenteral nutrition for patients who cannot take in adequate calories.

ANS: A, B, C, D Because sepsis is the most frequent etiology for SIRS, measures to avoid infection such as removing indwelling urinary catheters as soon as possible, use of aseptic technique, and early ambulation should be included in the plan of care. Adequate nutrition is important in preventing SIRS. Enteral, rather than parenteral, nutrition is preferred when patients are unable to take oral feedings because enteral nutrition helps maintain the integrity of the intestine, thus decreasing infection risk. Antibiotics should be given within 1 hour after being prescribed to decrease the risk of sepsis progressing to SIRS. DIF: Cognitive Level: Apply (application) REF: 1606 TOP: Nursing Process: Planning MSC: NCLEX: Physiological Integrity

1. Which factors will the nurse consider when calculating the CURB-65 score for a patient with pneumonia (select all that apply)? a. Age d. O2 saturation b. Blood pressure e. Presence of confusion c. Respiratory rate f. Blood urea nitrogen (BUN) level

ANS: A, B, C, E, F Data collected for the CURB-65 are mental status (confusion), BUN (elevated), blood pressure (decreased), respiratory rate (increased), and age (65 years and older). The other information is also essential to assess, but are not used for CURB-65 scoring. DIF: Cognitive Level: Apply (application) REF: 501 TOP: Nursing Process: Assessment MSC: NCLEX: Physiological Integrity

The nurse is reviewing the medical records for five patients who are scheduled for their yearly physical examinations in September. Which patients should receive the inactivated influenza vaccination (select all that apply)? a. A 76-yr-old nursing home resident b. A 36-yr-old female patient who is pregnant c. A 42-yr-old patient who has a 15 pack-year smoking history d. A 30-yr-old patient who takes corticosteroids for rheumatoid arthritis e. A 24-yr-old patient who has allergies to penicillin and cephalosporins

ANS: A, B, D Current guidelines suggest that healthy individuals between 6 months and age 49 years receive intranasal immunization with live, attenuated influenza vaccine. Individuals who are pregnant, residents of nursing homes, or are immunocompromised or who have chronic medical conditions should receive inactivated vaccine by injection. The corticosteroid use by the 30-yr-old patient increases the risk for infection. DIF: Cognitive Level: Apply (application) REF: 481 OBJ: Special Questions: Multiple Patients TOP: Nursing Process: Planning MSC: NCLEX: Health Promotion and Maintenance

1. A patient with suspected neurogenic shock after a diving accident has arrived in the emergency department. A cervical collar is in place. Which actions should the nurse take (select all that apply)? a. Prepare to administer atropine IV. b. Obtain baseline body temperature. c. Infuse large volumes of lactated Ringer's solution. d. Provide high-flow O2 (100%) by nonrebreather mask. e. Prepare for emergent intubation and mechanical ventilation.

ANS: A, B, D, E All of the actions are appropriate except to give large volumes of lactated Ringer's solution. The patient with neurogenic shock usually has a normal blood volume, and it is important not to volume overload the patient. In addition, lactated Ringer's solution is used cautiously in all shock situations because an ischemic liver cannot convert lactate to bicarbonate. DIF: Cognitive Level: Apply (application) REF: 1600 TOP: Nursing Process: Implementation MSC: NCLEX: Physiological Integrity

A nurse is assessing a premature newborn. Which assessment findings are consistent with prematurity? (Select all that apply.) a. Abundant lanugo over the body b. Ear cartilage soft and pliable c. Flexed body posture d. Deep creases on the sole of the foot e. Skin is bright pink, smooth, and shiny.

ANS: A, B, E The premature newborn has fine lanugo hair that is abundant over the body. The ear cartilage is soft and pliable, and the soles and palms have minimal creases, resulting in a smooth appearance. The premature newborn's skin is bright pink (often translucent, depending on the degree of immaturity), smooth, and shiny, with small blood vessels clearly visible underneath the thin epidermis. In contrast to full-term infants' overall attitude of flexion and continuous activity, preterm infants may be inactive and listless. The extremities maintain an attitude of extension and remain in any position in which they are placed.

A nurse is admitting a premature newborn to the NICU. Which interventions should the nurse implement to prevent retinopathy? (Select all that apply.) a. Place on pulse oximetry. b. Decrease exposure to bright, direct lighting. c. Place on a cardiac monitor. d. Cover eyes with an eye shield at night. e. Use supplemental oxygen only when needed.

ANS: A, B, E To prevent retinopathy, the nurse should provide preventive care by closely monitoring blood oxygen levels, responding promptly to saturation alarms, and preventing fluctuations in blood oxygen levels. Pulse oximetry is recommended to monitor the infant's oxygenation status during resuscitation and to prevent excessive use of oxygen in both term and preterm infants. Decrease exposure to bright, direct lighting; although exposure to bright light has not been proven to contribute to retinopathy of prematurity, such exposure is undesirable from a neurobehavioral developmental perspective. Use supplemental oxygen judiciously and monitor oxygen blood levels carefully; prevent wide fluctuations in oxygen blood levels (hyperoxia and hypoxia). Placing the newborn on a cardiac monitor will not prevent retinopathy. Covering the eyes with eye shields is not a preventive measure for retinopathy.

A nurA nurse is planning care for a premature newborn. Which interventions should the nurse implement for skin care? (Select all that apply.) a. Use cleaning agents with neutral pH. b. Rub skin during drying. c. Use adhesive remover solvent when removing tape. d. Avoid removing adhesives for at least 24 hours. e. Consider pectin barriers beneath adhesives

ANS: A, D, E The skin care for a premature newborn should include use of pH-neutral cleanser or soaps no more than two or three times a week. Adhesives should not be removed for at least 24 hours after application. Pectin barriers should be used beneath adhesives to protect skin. Avoid rubbing skin during bathing or drying. Do not use adhesive remover, solvents, or bonding agents. Adhesive removal can be facilitated using water, mineral oil, or petrolatum.

The community health nurse is teaching parents about prevention of the spread and reoccurrence of pediculosis (head lice). Which should the nurse include in the teaching session? (Select all that apply.) a. Dryclean nonwashable items. b. Spray the environment with an insecticide. c. Seal nonwashable items in a plastic bag for 5 days. d. Boil combs and brushes for 10 minutes. e. Discourage sharing of personal items.

ANS: A, D, E To prevent the spread and reoccurrence of pediculosis the nurse should teach the parents to: dryclean nonwashable items, boil combs and brushes for 10 minutes or soak for 1 hour in a pediculicide, and discourage the sharing of personal items, such as combs, hats, scarves and other headgear. Spraying with insecticide is not recommended because of the danger to children and animals. Nonwashable items should be sealed for 14 days in a plastic bag.

Following surgery for an abdominal aortic aneurysm, a patient's central venous pressure (CVP) monitor indicates low pressures. Which action is a priority for the nurse to take? a. Administer IV diuretic medications. b. Increase the IV fluid infusion per protocol. c. Document the CVP and continue to monitor. d. Elevate the head of the patient's bed to 45 degrees.

ANS: B A low CVP indicates hypovolemia and a need for an increase in the infusion rate. Diuretic administration will contribute to hypovolemia and elevation of the head may decrease cerebral perfusion. Documentation and continued monitoring is an inadequate response to the low CVP.

Acyclovir (Zovirax) is given to children with chickenpox to: a. minimize scarring. b. decrease the number of lesions. c. prevent aplastic anemia. d. prevent spread of the disease.

ANS: B Acyclovir decreases the number of lesions; shortens duration of fever; and decreases itching, lethargy, and anorexia. Treating pruritus and discouraging itching minimize scarring. Aplastic anemia is not a complication of chickenpox. Strict isolation until vesicles are dried prevents spread of disease.

11. Norepinephrine has been prescribed for a patient who was admitted with dehydration and hypotension. Which patient data indicate that the nurse should consult with the health care provider before starting the norepinephrine? a. The patient is receiving low dose dopamine. b. The patient's central venous pressure is 3 mm Hg. c. The patient is in sinus tachycardia at 120 beats/min. d. The patient has had no urine output since being admitted.

ANS: B Adequate fluid administration is essential before giving vasopressors to patients with hypovolemic shock. The patient's low central venous pressure indicates a need for more volume replacement. The other patient data are not contraindications to norepinephrine administration. DIF: Cognitive Level: Apply (application) REF: 1598 TOP: Nursing Process: Implementation MSC: NCLEX: Physiological Integrity

24. After change-of-shift report in the progressive care unit, who should the nurse care for first? a. Patient who had an inferior myocardial infarction 2 days ago and has crackles in the lung bases b. Patient with suspected urosepsis who has new orders for urine and blood cultures and antibiotics c. Patient who had a T5 spinal cord injury 1 week ago and currently has a heart rate of 54 beats/minute d. Patient admitted with anaphylaxis 3 hours ago who now has clear lung sounds and a blood pressure of 108/58 mm Hg

ANS: B Antibiotics should be given within the first hour for patients who have sepsis or suspected sepsis in order to prevent progression to systemic inflammatory response syndrome and septic shock. The data on the other patients indicate that they are more stable. Crackles heard only at the lung bases do not require immediate intervention in a patient who has had a myocardial infarction. Mild bradycardia does not usually require atropine in patients who have a spinal cord injury. The findings for the patient admitted with anaphylaxis indicate resolution of bronchospasm and hypotension. DIF: Cognitive Level: Analyze (analysis) REF: 1601 OBJ: Special Questions: Prioritization | Special Questions: Multiple Patients TOP: Nursing Process: Assessment MSC: NCLEX: Safe and Effective Care Environment

9. Which finding is the best indicator that the fluid resuscitation for a 90-kg patient with hypovolemic shock has been effective? a. Hemoglobin is within normal limits. b. Urine output is 65 mL over the past hour. c. Central venous pressure (CVP) is normal. d. Mean arterial pressure (MAP) is 72 mm Hg.

ANS: B Assessment of end organ perfusion, such as an adequate urine output, is the best indicator that fluid resuscitation has been successful. Urine output should be equal to or more than 0.5 mL/kg/hr. The hemoglobin level, CVP, and MAP are useful in determining the effects of fluid administration, but they are not as useful as data indicating good organ perfusion. DIF: Cognitive Level: Analyze (analysis) REF: 1589 TOP: Nursing Process: Evaluation MSC: NCLEX: Physiological Integrity

A patient admitted with acute respiratory failure has ineffective airway clearance related to thick secretions. Which nursing intervention would specifically address this patient problem? a. Encourage use of the incentive spirometer. b. Offer the patient fluids at frequent intervals. c. Teach the patient the importance of ambulation. d. Titrate oxygen level to keep O2 saturation above 93%.

ANS: B Because the reason for the poor airway clearance is the thick secretions, the best action will be to encourage the patient to improve oral fluid intake. Patients should be instructed to use the incentive spirometer on a regular basis (e.g., every hour) to facilitate the clearance of the secretions. The other actions may also be helpful in improving the patient's gas exchange, but they do not address the thick secretions that are causing the poor airway clearance. DIF: Cognitive Level: Apply (application)

23. Which finding about a patient who is receiving vasopressin to treat septic shock indicates an immediate need for the nurse to report the finding to the health care provider? a. The patient's urine output is 18 mL/hr. b. The patient is complaining of chest pain. c. The patient's peripheral pulses are weak. d. The patient's heart rate is 110 beats/minute.

ANS: B Because vasopressin is a potent vasoconstrictor, it may decrease coronary artery perfusion. The other information is consistent with the patient's diagnosis, and should be reported to the health care provider but does not indicate an immediate need for a change in therapy. DIF: Cognitive Level: Apply (application) REF: 1599 TOP: Nursing Process: Assessment MSC: NCLEX: Physiological Integrity

The nurse is examining 12-month-old Amy, who was brought to the clinic for persistent diaper rash. The nurse finds perianal inflammation with satellite lesions that cross the inguinal folds. What is most likely the cause of the diaper rash? a. Impetigo b. Candida albicans c. Urine and feces d. Infrequent diapering

ANS: B C. albicans infection produces perianal inflammation and a maculopapular rash with satellite lesions that may cross the inguinal folds. Impetigo is a bacterial infection that spreads peripherally in sharply marginated, irregular outlines. Eruptions involving the skin in contact with the diaper, but sparing the folds, are likely to be caused by chemical irritation, especially urine and feces.

Airborne isolation is required for a child who is hospitalized with: a. mumps. b. chickenpox. c. exanthema subitum (roseola). d. erythema infectiosum (fifth disease).

ANS: B Chickenpox is communicable through direct contact, droplet spread, and contaminated objects. Mumps is transmitted from direct contact with saliva of infected person and is most communicable before onset of swelling. The transmission and source of the viral infection exanthema subitum (roseola) is unknown. Erythema infectiosum (fifth disease) is communicable before onset of symptoms.

A 68-year-old patient has been in the intensive care unit for 4 days and has a nursing diagnosis of disturbed sensory perception related to sleep deprivation. Which action should the nurse include in the plan of care? a. Administer prescribed sedatives or opioids at bedtime to promote sleep. b. Cluster nursing activities so that the patient has uninterrupted rest periods. c. Silence the alarms on the cardiac monitors to allow 30- to 40-minute naps. d. Eliminate assessments between 0100 and 0600 to allow uninterrupted sleep.

ANS: B Clustering nursing activities and providing uninterrupted rest periods will minimize sleep-cycle disruption. Sedative and opioid medications tend to decrease the amount of rapid eye movement (REM) sleep and can contribute to sleep disturbance and disturbed sensory perception. Silencing the alarms on the cardiac monitors would be unsafe in a critically ill patient, as would discontinuing assessments during the night.

The central venous oxygen saturation (ScvO2) is decreasing in a patient who has severe pancreatitis. To determine the possible cause of the decreased ScvO2, the nurse assesses the patient's a. lipase. b. temperature. c. urinary output. d. body mass index.

ANS: B Elevated temperature increases metabolic demands and oxygen use by tissues, resulting in a drop in oxygen saturation of central venous blood. Information about the patient's body mass index, urinary output, and lipase will not help in determining the cause of the patient's drop in ScvO2.

Which term best describes the identification of the distribution and causes of disease, injury, or illness? a. Nursing process b. Epidemiologic process c. Community-based statistics d. Mortality and morbidity statistics

ANS: B Epidemiology is the science of population health applied to the detection of morbidity and mortality in a population. It identifies the distribution and causes of diseases across a population. Nursing process is a systematic problem-solving approach for the delivery of nursing care. Morbidity and mortality statistics, along with natal rates, may provide an objective picture of a community's health status.

1. During the primary assessment of a victim of a motor vehicle collision, the nurse determines that the patient has an unobstructed airway. Which action should the nurse take next? a. Palpate extremities for bilateral pulses. b. Observe the patient's respiratory effort. c. Check the patient's level of consciousness. d. Examine the patient for any external bleeding.

ANS: B Even with a patent airway, patients can have other problems that compromise ventilation, so the next action is to assess the patient's breathing. The other actions are also part of the initial survey but assessment of breathing should be done immediately after assessing for airway patency.

The intensive care unit (ICU) nurse educator will determine that teaching about arterial pressure monitoring for a new staff nurse has been effective when the nurse a. balances and calibrates the monitoring equipment every 2 hours. b. positions the zero-reference stopcock line level with the phlebostatic axis. c. ensures that the patient is supine with the head of the bed flat for all readings. d. rechecks the location of the phlebostatic axis when changing the patient's position.

ANS: B For accurate measurement of pressures, the zero-reference level should be at the phlebostatic axis. There is no need to rebalance and recalibrate monitoring equipment hourly. Accurate hemodynamic readings are possible with the patient's head raised to 45 degrees or in the prone position. The anatomic position of the phlebostatic axis does not change when patients are repositioned.

An experienced nurse instructs a new nurse about how to care for a patient with dyspnea caused by a pulmonary fungal infection. Which action by the new nurse indicates a need for further teaching? a. Listening to the patient's lung sounds several times during the shift b. Placing the patient on droplet precautions in a private hospital room c. Monitoring patient serology results to identify the infecting organism d. Increasing the O2 flow rate to keep the O2 saturation over 90%

ANS: B Fungal infections are not transmitted from person to person. Therefore no isolation procedures are necessary. The other actions by the new nurse are appropriate. DIF: Cognitive Level: Apply (application) REF: 512 TOP: Nursing Process: Evaluation MSC: NCLEX: Safe and Effective Care Environment

16. Gastric lavage and administration of activated charcoal are ordered for an unconscious patient who has been admitted to the emergency department (ED) after ingesting 30 lorazepam (Ativan) tablets. Which prescribed action should the nurse plan to do first? a. Insert a large-bore orogastric tube. b. Assist with intubation of the patient. c. Prepare a 60-mL syringe with saline. d. Give first dose of activated charcoal.

ANS: B In an unresponsive patient, intubation is done before gastric lavage and activated charcoal administration to prevent aspiration. The other actions will be implemented after intubation.

While caring for a patient who has been admitted with a pulmonary embolism, the nurse notes a change in the patient's oxygen saturation (SpO2) from 94% to 88%. Which action should the nurse take? a. Suction the patient's oropharynx. b. Increase the prescribed O2 flow rate. c. Instruct the patient to cough and deep breathe. d. Help the patient to sit in a more upright position.

ANS: B Increasing O2 flow rate will usually improve O2 saturation in patients with ventilation-perfusion mismatch, as occurs with pulmonary embolism. Because the problem is with perfusion, actions that improve ventilation, such as deep breathing and coughing, sitting upright, and suctioning, are not likely to improve oxygenation. DIF: Cognitive Level: Apply (application)

A patient has just been admitted with probable bacterial pneumonia and sepsis. Which order should the nurse implement first? a. Chest x-ray via stretcher b. Blood cultures from two sites c. Ciprofloxacin (Cipro) 400 mg IV d. Acetaminophen (Tylenol) rectal suppository

ANS: B Initiating antibiotic therapy rapidly is essential, but it is important that the cultures be obtained before antibiotic administration. The chest x-ray and acetaminophen administration can be done last. DIF: Cognitive Level: Analyze (analysis) REF: 501 OBJ: Special Questions: Prioritization TOP: Nursing Process: Implementation MSC: NCLEX: Physiological Integrity

A nurse is assessing a child and notes Koplik spots. In which of these communicable diseases are Koplik spots present? a. Rubella b. Measles (rubeola) c. Chickenpox (varicella) d. Exanthema subitum (roseola)

ANS: B Koplik spots are small irregular red spots with a minute, bluish white center found on the buccal mucosa 2 days before systemic rash. Rubella occurs with rash on the face, which rapidly spreads downward. Varicella appears with highly pruritic macules, followed by papules and vesicles. Roseola is seen with rose-pink macules on the trunk, spreading to face and extremities.

The nurse should implement which prescribed treatment for a child with warts? a. Vaccination b. Local destruction c. Corticosteroids d. Specific antibiotic therapy

ANS: B Local destructive therapy individualized according to location, type, and number—including surgical removal, electrocautery, curettage, cryotherapy, caustic solutions, x-ray treatment, and laser therapies—is used. Vaccination is prophylaxis for warts and is not a treatment. Corticosteroids and specific antibiotic therapy are not effective in the treatment of warts.

3. A patient with massive trauma and possible spinal cord injury is admitted to the emergency department (ED). Which assessment finding by the nurse will help confirm a diagnosis of neurogenic shock? a. Inspiratory crackles c. Cool, clammy extremities b. Heart rate 45 beats/min d. Temperature 101.2°F (38.4°C)

ANS: B Neurogenic shock is characterized by hypotension and bradycardia. The other findings would be more consistent with other types of shock. DIF: Cognitive Level: Understand (comprehension) REF: 1590 TOP: Nursing Process: Assessment MSC: NCLEX: Physiological Integrity

The nurse is caring for a newborn who was born 24 hours ago to a mother who received no prenatal care. The newborn is a poor feeder but sucks avidly on his hands. Clinical manifestations also include loose stools, tachycardia, fever, projectile vomiting, sneezing, and generalized sweating. Which should the nurse suspect? a. Seizure disorder b. Narcotic withdrawal c. Placental insufficiency d. Meconium aspiration syndrome

ANS: B Newborns exposed to drugs in utero usually show no untoward effects until 12 to 24 hours for heroin or much longer for methadone. The newborn usually has nonspecific signs that may coexist with other conditions such as hypocalcemia and hypoglycemia. In addition, these newborns may have loose stools, tachycardia, fever, projectile vomiting, sneezing, and generalized sweating, which is uncommon in newborns. Loose stools, tachycardia, fever, projectile vomiting, sneezing, and generalized sweating are manifestations not descriptive of seizure activity. Placental insufficiency usually results in a child who is small for gestational age. Meconium aspiration syndrome usually has manifestations of respiratory distress.

Phenylketonuria (PKU) is a genetic disease that results in the body's inability to correctly metabolize: a. glucose. b. phenylalanine. c. phenylketones. d. thyroxine.

ANS: B PKU is an inborn error of metabolism caused by a deficiency or absence of the enzyme needed to metabolize the essential amino acid phenylalanine. Phenylalanine hydroxylase is missing in PKU. Individuals with this disorder can metabolize glucose. Phenylketones are metabolites of phenylalanine, excreted in the urine. Thyroxine is one of the principal hormones secreted by the thyroid gland.

7. A patient with cardiogenic shock has the following vital signs: BP 102/50, pulse 128, respirations 28. The pulmonary artery wedge pressure (PAWP) is increased, and cardiac output is low. The nurse will anticipate an order for which medication? a. 5% albumin infusion c. epinephrine (Adrenalin) drip b. furosemide (Lasix) IV d. hydrocortisone (Solu-Cortef)

ANS: B The PAWP indicates that the patient's preload is elevated, and furosemide is indicated to reduce the preload and improve cardiac output. Epinephrine would further increase the heart rate and myocardial oxygen demand. 5% albumin would also increase the PAWP. Hydrocortisone might be considered for septic or anaphylactic shock. DIF: Cognitive Level: Apply (application) REF: 1600 TOP: Nursing Process: Planning MSC: NCLEX: Physiological Integrity

The nurse educator is evaluating the performance of a new registered nurse (RN) who is providing care to a patient who is receiving mechanical ventilation with 15 cm H2O of peak end-expiratory pressure (PEEP). Which action indicates that the new RN is safe? a. The RN plans to suction the patient every 1 to 2 hours. b. The RN uses a closed-suction technique to suction the patient. c. The RN tapes connection between the ventilator tubing and the ET. d. The RN changes the ventilator circuit tubing routinely every 48 hours.

ANS: B The closed-suction technique is used when patients require high levels of PEEP (>10 cm H2O) to prevent the loss of PEEP that occurs when disconnecting the patient from the ventilator. Suctioning should not be scheduled routinely, but it should be done only when patient assessment data indicate the need for suctioning. Taping connections between the ET and the ventilator tubing would restrict the ability of the tubing to swivel in response to patient repositioning. Ventilator tubing changes increase the risk for ventilator-associated pneumonia (VAP) and are not indicated routinely.

The nurse obtains the following assessment data on an older patient who has influenza. Which information will be most important for the nurse to communicate to the health care provider? a. Fever of 100.4° F (38° C) b. Diffuse crackles in the lungs c. Sore throat and frequent cough d. Myalgia and persistent headache

ANS: B The crackles indicate that the patient may be developing pneumonia, a common complication of influenza, which would require aggressive treatment. Myalgia, headache, mild temperature elevation, and sore throat with cough are typical manifestations of influenza and are treated with supportive care measures such as over-the-counter pain relievers and increased fluid intake. DIF: Cognitive Level: Analyze (analysis) REF: 481 OBJ: Special Questions: Prioritization TOP: Nursing Process: Assessment MSC: NCLEX: Physiological Integrity

. Which action should the nurse take first when a patient develops epistaxis? a. Pack the affected nare tightly with an epistaxis balloon. b. Apply squeezing pressure to the nostrils for 10 minutes. c. Obtain silver nitrate that may be needed for cauterization. d. Instill a vasoconstrictor medication into the affected nare.

ANS: B The first nursing action for epistaxis is to apply direct pressure by pinching the nostrils. Application of cold packs may decrease blood flow to the area but will not be sufficient to stop bleeding. Cauterization, nasal packing, and vasoconstrictors are medical interventions that may be needed if pressure to the nares does not stop the bleeding, but these are not the first actions to take for a nosebleed. DIF: Cognitive Level: Analyze (analysis) REF: 476 OBJ: Special Questions: Prioritization TOP: Nursing Process: Implementation MSC: NCLEX: Physiological Integrity

20. A patient who has been involved in a motor vehicle crash arrives in the emergency department (ED) with cool, clammy skin; tachycardia; and hypotension. Which intervention ordered by the health care provider should the nurse implement first? a. Insert two large-bore IV catheters. b. Provide O2 at 100% per non-rebreather mask. c. Draw blood to type and crossmatch for transfusions. d. Initiate continuous electrocardiogram (ECG) monitoring.

ANS: B The first priority in the initial management of shock is maintenance of the airway and ventilation. ECG monitoring, insertion of IV catheters, and obtaining blood for transfusions should also be rapidly accomplished but only after actions to maximize O2 delivery have been implemented. DIF: Cognitive Level: Analyze (analysis) REF: 1597 OBJ: Special Questions: Prioritization TOP: Nursing Process: Implementation MSC: NCLEX: Physiological Integrity

When evaluating a patient with a central venous catheter, the nurse observes that the insertion site is red and tender to touch and the patient's temperature is 101.8° F. What should the nurse plan to do next? a. Give analgesics and antibiotics as ordered. b. Discontinue the catheter and culture the tip. c. Change the flush system and monitor the site. d. Check the site more frequently for any swelling.

ANS: B The information indicates that the patient has a local and systemic infection caused by the catheter, and the catheter should be discontinued. Changing the flush system, giving analgesics, and continued monitoring will not help prevent or treat the infection. Administration of antibiotics is appropriate, but the line should still be discontinued to avoid further complications such as endocarditis.

2. The nurse plans to teach a patient how to manage allergic rhinitis. Which information should the nurse include in the teaching plan? a. Using oral antihistamines for 2 weeks before the allergy season may prevent reactions. b. Identifying and avoiding environmental triggers are the best way to prevent symptoms. c. Frequent hand washing is the primary way to prevent spreading the condition to others. d. Corticosteroid nasal sprays will reduce inflammation, but systemic effects limit their use.

ANS: B The most important intervention is to assist the patient in identifying and avoiding potential allergens. Intranasal corticosteroids (not oral antihistamines) should be started several weeks before the allergy season. Corticosteroid nasal sprays have minimal systemic absorption. Acute viral rhinitis (common cold) can be prevented by washing hands, but allergic rhinitis cannot. DIF: Cognitive Level: Apply (application) REF: 477 TOP: Nursing Process: Planning MSC: NCLEX: Physiological Integrity

The nurse discusses management of upper respiratory infections (URIs) with a patient who has acute sinusitis. Which statement by the patient indicates that additional teaching is needed? a. "I will drink lots of juices and other fluids to stay well hydrated." b. "I can use nasal decongestant spray until the congestion is gone." c. "I can take acetaminophen (Tylenol) to treat my sinus discomfort." d. "I will watch for changes in nasal secretions or the sputum that I cough up."

ANS: B The nurse should clarify that nasal decongestant sprays should be used for no more than 3 days to prevent rebound vasodilation and congestion. The other responses indicate that the teaching has been effective. DIF: Cognitive Level: Apply (application) REF: 480 TOP: Nursing Process: Evaluation MSC: NCLEX: Physiological Integrity

11. When assessing an older patient admitted to the emergency department (ED) with a broken arm and facial bruises, the nurse observes several additional bruises in various stages of healing. Which statement or question by the nurse should be first? a. "You should not go home." b. "Do you feel safe at home?" c. "Would you like to see a social worker?" d. "I need to report my concerns to the police."

ANS: B The nurse's initial response should be to further assess the patient's situation. Telling the patient not to return home may be an option once further assessment is done. A social worker or police report may be appropriate once further assessment is completed.

The oxygen saturation (SpO2) for a patient with left lower lobe pneumonia is 90%. The patient has wheezes, a weak cough effort, and complains of fatigue. Which action should the nurse take next? a. Position the patient on the left side. b. Assist the patient with staged coughing. c. Place a humidifier in the patient's room. d. Schedule a 4-hour rest period for the patient.

ANS: B The patient's assessment indicates that assisted coughing is needed to help remove secretions, which will improve oxygenation. A 4-hour rest period at this time may allow the O2 saturation to drop further. Humidification will not be helpful unless the secretions can be mobilized. Positioning on the left side may cause a further decrease in oxygen saturation because perfusion will be directed more toward the more poorly ventilated lung. DIF: Cognitive Level: Apply (application)

22. When assessing a patient with a sore throat, the nurse notes anterior cervical lymph node swelling, a temperature of 101.6° F (38.7° C), and yellow patches on the tonsils. Which action will the nurse anticipate taking? a. Teach the patient about the use of expectorants. b. Use a swab to obtain a sample for a rapid strep antigen test. c. Discuss the need to rinse the mouth out after using any inhalers. d. Teach the patient to avoid nonsteroidal antiinflammatory drugs (NSAIDs).

ANS: B The patient's clinical manifestations are consistent with streptococcal pharyngitis, and the nurse will anticipate the need for a rapid strep antigen test or cultures (or both). Because patients with streptococcal pharyngitis usually do not have a cough, use of expectorants will not be anticipated. Rinsing out the mouth after inhaler use may prevent fungal oral infections, but the patient's assessment data are not consistent with a fungal infection. NSAIDs are frequently prescribed for pain and fever relief with pharyngitis. DIF: Cognitive Level: Apply (application) REF: 484 TOP: Nursing Process: Planning MSC: NCLEX: Physiological Integrity

2. A nurse is caring for a patient whose hemodynamic monitoring indicates a blood pressure of 92/54 mm Hg, a pulse of 64 beats/min, and an elevated pulmonary artery wedge pressure (PAWP). Which intervention ordered by the health care provider should the nurse question? a. Elevate head of bed to 30 degrees. b. Infuse normal saline at 250 mL/hr. c. Hold nitroprusside if systolic BP is less than 90 mm Hg. d. Titrate dobutamine to keep systolic BP is greater than 90 mm Hg.

ANS: B The patient's elevated PAWP indicates volume excess in relation to cardiac pumping ability, consistent with cardiogenic shock. A saline infusion at 250 mL/hr will exacerbate the volume excess. The other actions will help to improve cardiac output, which should lower the PAWP and may raise the BP. DIF: Cognitive Level: Apply (application) REF: 1600 TOP: Nursing Process: Planning MSC: NCLEX: Physiological Integrity

An 81-year-old patient who has been in the intensive care unit (ICU) for a week is now stable and transfer to the progressive care unit is planned. On rounds, the nurse notices that the patient has new onset confusion. The nurse will plan to a. give PRN lorazepam (Ativan) and cancel the transfer. b. inform the receiving nurse and then transfer the patient. c. notify the health care provider and postpone the transfer. d. obtain an order for restraints as needed and transfer the patient.

ANS: B The patient's history and symptoms most likely indicate delirium associated with the sleep deprivation and sensory overload in the ICU environment. Informing the receiving nurse and transferring the patient is appropriate. Postponing the transfer is likely to prolong the delirium. Benzodiazepines and restraints contribute to delirium and agitation.

A patient with respiratory failure has a respiratory rate of 6 breaths/min and an oxygen saturation (SpO2) of 88%. The patient is increasingly lethargic. Which intervention will the nurse anticipate? a. Administration of 100% O2 by non-rebreather mask b. Endotracheal intubation and positive pressure ventilation c. Insertion of a mini-tracheostomy with frequent suctioning d. Initiation of continuous positive pressure ventilation (CPAP)

ANS: B The patient's lethargy, low respiratory rate, and SpO2 indicate the need for mechanical ventilation with ventilator-controlled respiratory rate. Giving high-flow O2 will not be helpful because the patient's respiratory rate is so low. Insertion of a mini-tracheostomy will facilitate removal of secretions, but it will not improve the patient's respiratory rate or oxygenation. CPAP requires that the patient initiate an adequate respiratory rate to allow adequate gas exchange. DIF: Cognitive Level: Apply (application)

The nurse reviews the electronic health record for a patient scheduled for a total hip replacement. Which assessment data shown in the accompanying figure increase the patient's risk for respiratory complications after surgery? History Lab Data Physical Assessment *Age 81 H&H 11.8/38% *LCTA * Med/Surg history: Albumin 2.7 *Mildly confused: Recent 15lb weight loss, disoriented to date, Knee arthroscopy 3 oriented to person and place months ago a. Older age and anemia b. Albumin level and weight loss c. Recent arthroscopic procedure d. Confusion and disorientation to time

ANS: B The patient's recent weight loss and low protein stores indicate possible muscle weakness, which make it more difficult for an older patient to recover from the effects of general anesthesia and immobility associated with the hip surgery. The other information will also be noted by the nurse but does not place the patient at higher risk for respiratory failure. DIF: Cognitive Level: Analyze (analysis)

A nurse is caring for a patient with ARDS who is being treated with mechanical ventilation and high levels of positive end-expiratory pressure (PEEP). Which assessment finding by the nurse indicates that the PEEP may need to be reduced? a. The patient's PaO2 is 50 mm Hg and the SaO2 is 88%. b. The patient has subcutaneous emphysema on the upper thorax. c. The patient has bronchial breath sounds in both the lung fields. d. The patient has a first-degree atrioventricular heart block with a rate of 58 beats/min.

ANS: B The subcutaneous emphysema indicates barotrauma caused by positive pressure ventilation and PEEP. Bradycardia, hypoxemia, and bronchial breath sounds are all concerns and will need to be addressed, but they are not specific indications that PEEP should be reduced. DIF: Cognitive Level: Apply (application)

A nurse is teaching parents about caring for their child with chickenpox. The nurse should let the parents know that the child is considered to be no longer contagious when which occurs? a. When fever is absent b. When lesions are crusted c. 24 hours after lesions erupt d. 8 days after onset of illness

ANS: B When the lesions are crusted, the chickenpox is no longer contagious. This may be a week after onset of disease. Chickenpox is still contagious when child has fever. Children are contagious after lesions erupt. If lesions are crusted at 8 days, the child is no longer contagious.

A patient with a pleural effusion is scheduled for a thoracentesis. Which action should the nurse take to prepare the patient for the procedure? a. Start a peripheral IV line to administer sedatives. b. Position the patient sitting up on the side of the bed. c. Obtain a collection device to hold 3 liters of pleural fluid. d. Remind the patient not to eat or drink anything for 6 hours.

ANS: B When the patient is sitting up, fluid accumulates in the pleural space at the lung bases and can more easily be located and removed. The patient does not usually require sedation for the procedure, and there are no restrictions on oral intake because the patient is not sedated or unconscious. Usually only 1000 to 1200 mL of pleural fluid is removed at one time. Rapid removal of a large volume can result in hypotension, hypoxemia, or pulmonary edema. DIF: Cognitive Level: Apply (application) REF: 527 TOP: Nursing Process: Planning MSC: NCLEX: Physiological Integrity

A nurse is reviewing acid-base laboratory data on a newborn admitted to the NICU for meconium aspiration. Which laboratory values should the nurse report to the physician? (Select all that apply.) a. pH: 7.35 b. PCO2 : 49 c. HCO3-: 30 d. PaO2: 96

ANS: B, C Normal values of pH for a newborn are: Birth: 7.11-7.36 1 day: 7.29-7.45 Child: 7.35-7.45. Normal values of PCO2 are: Newborn: 27-40 mm Hg Infant: 27-41 mm Hg Girls: 32-45 mm Hg Boys: 35-48 mm Hg. Normal values for HCO3- are: Infant: 21-28 mEq/ml Thereafter: 22-26 mEq/ml. The PaO2 is within normal limits for a newborn. Therefore, the nurse should report the PCO2 of 49 and the HCO3- of 30.

Which actions should the nurse start to reduce the risk for ventilator-associated pneumonia (VAP) (select all that apply)? a. Obtain arterial blood gases daily. b. Provide a "sedation holiday" daily. c. Give prescribed pantoprazole (Protonix). d. Elevate the head of the bed to at least 30°. e. Provide oral care with chlorhexidine (0.12%) solution daily.

ANS: B, C, D, E All of these interventions are part of the ventilator bundle that is recommended to prevent VAP. Arterial blood gases may be done daily but are not always necessary and do not help prevent VAP.

A nurse is assessing a premature newborn for the possibility of necrotizing enterocolitis (NEC). Which assessment findings should the nurse expect to find if NEC is confirmed? (Select all that apply.) a. Minimal gastric residual b. Abdominal distention c. Apnea d. Urinary output at 2 ml/kg/hr e. Unstable temperature

ANS: B, C, E The nurse should observe for indications of early development of NEC by checking the appearance of the abdomen for distention (measuring abdominal girth, measuring residual gastric contents before feedings, and listening for bowel sounds) and performing all routine assessments for high-risk neonates. The premature newborn may have apnea and unstable temperature if NEC is developing. The urinary output will be decreased and will be below the expected 2 ml/kg/hr.

A nurse is preparing to administer routine immunizations to a 4-month-old infant. The infant is currently up to date on all previously recommended immunizations. Which immunizations will the nurse prepare to administer? (Select all that apply.) a. Measles, mumps, and rubella (MMR) b. Rotavirus (RV) c. Diphtheria, tetanus, pertussis (DTaP) d. Varicella e. Haemophilus influenzae type b (HIB) f. Inactivated poliovirus (IPV)

ANS: B, C, E, F Recommended immunization schedule for a 4-month-old, up to date on immunizations, would be to administer the rotavirus (RV), diphtheria, tetanus, pertussis (DTaP), Haemophilus influenza type b (HIB), and inactivated poliovirus (IPV) vaccinations. The measles, mumps, and rubella (MMR) and varicella would not be administered until the child is at least 1 year of age.

7. A patient with hypotension and an elevated temperature after working outside on a hot day is treated in the emergency department (ED). The nurse determines that discharge teaching has been effective when the patient makes which statement? a. "I'll take salt tablets when I work outdoors in the summer." b. "I should take acetaminophen (Tylenol) if I start to feel too warm." c. "I need to drink extra fluids when working outside in hot weather." d. "I'll move to a cool environment if I notice that I'm feeling confused"

ANS: C Oral fluids and electrolyte replacement solutions such as sports drinks help replace fluid and electrolytes lost when exercising in hot weather. Salt tablets are not recommended because of the risks of gastric irritation and hypernatremia. Antipyretic drugs are not effective in lowering body temperature elevations caused by excessive exposure to heat. A patient who is confused is likely to have more severe hyperthermia and will be unable to remember to take appropriate action.

An intraaortic balloon pump (IABP) is being used for a patient who is in cardiogenic shock. Which assessment data indicate to the nurse that the goals of treatment with the IABP are being met? a. Urine output of 25 mL/hr b. Heart rate of 110 beats/minute c. Cardiac output (CO) of 5 L/min d. Stroke volume (SV) of 40 mL/beat

ANS: C A CO of 5 L/min is normal and indicates that the IABP has been successful in treating the shock. The low SV signifies continued cardiogenic shock. The tachycardia and low urine output also suggest continued cardiogenic shock.

The nurse is taking care of a 2-year-old child with a macule skin lesion. Which clinical finding should the nurse expect to assess with this type of lesion? a. Flat, nonpalpable, and irregularly shaped lesion that is greater than 1 cm in diameter b. Heaped-up keratinized cells, flaky exfoliation, irregular, thick or thin, dry or oily, varied in size c. Flat, brown mole less than 1 cm in diameter d. Elevated, flat-topped, firm, rough, superficial papule greater than 1 cm in diameter

ANS: C A macule is flat; nonpalpable; circumscribed; less than 1 cm in diameter; and brown, red, purple, white, or tan. A patch is a flat, nonpalpable, and irregularly shaped macule that is greater than 1 cm in diameter. Scale is heaped-up keratinized cells, flaky exfoliation, irregular, thick or thin, dry or oily, varied in size, and silver white or tan. A plaque is an elevated, flat-topped, firm, rough, superficial papule greater than 1 cm in diameter. It may be coalesced papules.

Which nursing consideration is important when caring for a child with impetigo contagiosa? a. Apply topical corticosteroids to decrease inflammation. b. Carefully remove dressings so as not to dislodge undermined skin, crusts, and debris. c. Carefully wash hands and maintain cleanliness when caring for an infected child. d. Examine child under a Wood lamp for possible spread of lesions.

ANS: C A major nursing consideration related to bacterial skin infections, such as impetigo contagiosa, is to prevent the spread of the infection and complications. This is done by thorough hand washing before and after contact with the affected child. Corticosteroids are not indicated in bacterial infections. Dressings are usually not indicated. The undermined skin, crusts, and debris are carefully removed after softening with moist compresses. A Wood lamp is used to detect fluorescent materials in the skin and hair. It is used in certain disease states, such as tinea capitis.

The nurse notes that a patient has incisional pain, a poor cough effort, and scattered coarse crackles after a thoracotomy. Which action should the nurse take first? a. Assist the patient to sit upright in a chair. b. Splint the patient's chest during coughing. c. Medicate the patient with prescribed morphine. d. Observe the patient use the incentive spirometer.

ANS: C A major reason for atelectasis and poor airway clearance in patients after chest surgery is incisional pain (which increases with deep breathing and coughing). The first action by the nurse should be to medicate the patient to minimize incisional pain. The other actions are all appropriate ways to improve airway clearance, but should be done after the morphine is given. DIF: Cognitive Level: Analyze (analysis) REF: 521 OBJ: Special Questions: Prioritization TOP: Nursing Process: Implementation MSC: NCLEX: Physiological Integrity

Which genetic term refers to the transfer of all or part of a chromosome to a different chromosome after chromosome breakage? a. Trisomy b. Monosomy c. Translocation d. Nondisjunction

ANS: C A translocation occurs when a part of a chromosome breaks off and attaches to another chromosome. When this occurs in the germ cells, the translocation can be transmitted to the next generation. Trisomy is the condition in which three of a specific chromosome are found rather than the usual two. Monosomy is the condition in which one of a specific chromosome is noted rather than the usual two. The term is not used for males when the normal complement of sex chromosomes (one X and one Y) is present. Nondisjunction is the failure of a chromosome to separate during cell division. Of the resultant daughter cells, one will be trisomic and one will be monosomic.

The single parent of a 3-year-old child who has just been diagnosed with chickenpox tells the nurse that she cannot afford to stay home with the child and miss work. The parent asks the nurse if some medication will shorten the course of the illness. Which is the most appropriate nursing intervention? a. Reassure the parent that it is not necessary to stay home with the child. b. Explain that no medication will shorten the course of the illness. c. Explain the advantages of the medication acyclovir (Zovirax) to treat chickenpox. d. Explain the advantages of the medication VCZ immune globulin (VariZIG) to treat chickenpox.

ANS: C Acyclovir is effective in treating the number of lesions; shortening the duration of fever; and decreasing itching, lethargy, and anorexia. It is important the parent stay with the child to monitor fever. Acyclovir lessens the severity of chickenpox. VariZIG is given only to high-risk children.

21. Family members are in the patient's room when the patient has a cardiac arrest and the staff start resuscitation measures. Which action should the nurse take next? a. Keep the family in the room and assign a staff member to explain the care given and answer questions. b. Ask the family to wait outside the patient's room with a designated staff member to provide emotional support. c. Ask the family members whether they would prefer to remain in the patient's room or wait outside the room. d. Tell the family members that patients are comforted by having family members present during resuscitation efforts.

ANS: C Although many family members and patients report benefits from family presence during resuscitation efforts, the nurse's initial action should be to determine the preference of these family members. The other actions may be appropriate, but this will depend on what is learned when assessing family preferences.

Which diagnostic test will provide the nurse with the most specific information to evaluate the effectiveness of interventions for a patient with ventilatory failure? a. Chest x-ray b. O2 saturation c. Arterial blood gas analysis d. Central venous pressure monitoring

ANS: C Arterial blood gas (ABG) analysis is most useful in this setting because ventilatory failure causes problems with CO2 retention, and ABGs provide information about the PaCO2 and pH. The other tests may also be done to help in assessing oxygenation or determining the cause of the patient's ventilatory failure. DIF: Cognitive Level: Apply (application)

A parent reports to the nurse that her child has inflamed conjunctivae of both eyes with purulent drainage and crusting of the eyelids, especially on awakening. These manifestations suggest: a. viral conjunctivitis. b. allergic conjunctivitis. c. bacterial conjunctivitis. d. conjunctivitis caused by foreign body.

ANS: C Bacterial conjunctivitis has these symptoms. Viral or allergic conjunctivitis has watery drainage. Foreign body causes tearing and pain, and usually only one eye is affected.

22. A patient who has deep human bite wounds on the left hand is being treated in the urgent care center. Which action will the nurse plan to take? a. Prepare to administer rabies immune globulin (BayRab). b. Assist the health care provider with suturing of the bite wounds. c. Teach the patient the reason for the use of prophylactic antibiotics. d. Keep the wounds dry until the health care provider can assess them.

ANS: C Because human bites of the hand frequently become infected, prophylactic antibiotics are usually prescribed to prevent infection. To minimize infection, deep bite wounds on the extremities are left open. Rabies immune globulin might be used after an animal bite. Initial treatment of bite wounds includes copious irrigation to help clean out contaminants and microorganisms.

8. A 22-yr-old patient who experienced a drowning accident in a local pool, but now is awake and breathing spontaneously, is admitted for observation. Which assessment will be most important for the nurse to take during the observation period? a. Auscultate heart sounds. c. Auscultate breath sounds. b. Palpate peripheral pulses. d. Check mental orientation.

ANS: C Because pulmonary edema is a common complication after drowning, the nurse should assess the breath sounds frequently. The other information also will be obtained by the nurse, but it is not as pertinent to the patient's admission diagnosis.

The nurse notes thick, white secretions in the endotracheal tube (ET) of a patient who is receiving mechanical ventilation. Which intervention will be most effective in addressing this problem? a. Increase suctioning to every hour. b. Reposition the patient every 1 to 2 hours. c. Add additional water to the patient's enteral feedings. d. Instill 5 mL of sterile saline into the ET before suctioning.

ANS: C Because the patient's secretions are thick, better hydration is indicated. Suctioning every hour without any specific evidence for the need will increase the incidence of mucosal trauma and would not address the etiology of the ineffective airway clearance. Instillation of saline does not liquefy secretions and may decrease the SpO2. Repositioning the patient is appropriate but will not decrease the thickness of secretions.

The nurse notes premature ventricular contractions (PVCs) while suctioning a patient's endotracheal tube. Which action by the nurse is a priority? a. Decrease the suction pressure to 80 mm Hg. b. Document the dysrhythmia in the patient's chart. c. Stop and ventilate the patient with 100% oxygen. d. Give antidysrhythmic medications per protocol.

ANS: C Dysrhythmias during suctioning may indicate hypoxemia or sympathetic nervous system stimulation. The nurse should stop suctioning and ventilate the patient with 100% oxygen. Lowering the suction pressure will decrease the effectiveness of suctioning without improving the hypoxemia. Because the PVCs occurred during suctioning, there is no need for antidysrhythmic medications (which may have adverse effects) unless they recur when the suctioning is stopped and patient is well oxygenated.

Vitamin A supplementation may be recommended for the young child who has which disease? a. Mumps b. Rubella c. Measles (rubeola) d. Erythema infectiosum

ANS: C Evidence shows vitamin A decreases morbidity and mortality in measles. Mumps is treated with analgesics for pain and antipyretics for fever. Rubella is treated similarly to mumps. Erythema infectiosum is treated similarly to mumps and rubella.

0. A patient who had a total laryngectomy has a nursing diagnosis of hopelessness related to loss of control of personal care. Which information obtained by the nurse indicates that this identified problem is resolving? a. The patient allows the nurse to suction the tracheostomy. b. The patient's spouse provides the daily tracheostomy care. c. The patient asks how to clean the tracheostomy stoma and tube. d. The patient uses a communication board to request "No Visitors."

ANS: C Independently caring for the laryngectomy tube indicates that the patient has regained control of personal care and hopelessness is at least partially resolved. Letting the nurse and spouse provide care and requesting no visitors may indicate that the patient is still experiencing hopelessness. DIF: Cognitive Level: Apply (application) REF: 495 TOP: Nursing Process: Evaluation MSC: NCLEX: Psychosocial Integrity

The family members of a patient who has just been admitted to the intensive care unit (ICU) with multiple traumatic injuries have just arrived in the ICU waiting room. Which action should the nurse take next? a. Explain ICU visitation policies and encourage family visits. b. Immediately take the family members to the patient's bedside. c. Describe the patient's injuries and the care that is being provided. d. Invite the family to participate in a multidisciplinary care conference.

ANS: C Lack of information is a major source of anxiety for family members and should be addressed first. Family members should be prepared for the patient's appearance and the ICU environment before visiting the patient for the first time. ICU visiting should be individualized to each patient and family rather than being dictated by rigid visitation policies. Inviting the family to participate in a multidisciplinary conference is appropriate but should not be the initial action by the nurse.

The school nurse is conducting a class for school-age children on Lyme disease. Which is characteristic of Lyme disease? a. Difficult to prevent b. Treated with oral antibiotics in stages 1, 2, and 3 c. Caused by a spirochete that enters the skin through a tick bite d. Common in geographic areas where the soil contains the mycotic spores that cause the disease

ANS: C Lyme disease is caused by Borrelia burgdorferi, a spirochete spread by ticks. The early characteristic rash is erythema migrans. Tick bites should be avoided by entering tick-infested areas with caution. Light-colored clothing should be worn to identify ticks easily. Long-sleeved shirts and long pants tucked into socks should be the attire. Early treatment of the erythema migrans (stage 1) can prevent the development of Lyme disease. Lyme disease is caused by a spirochete, not mycotic spores.

The nurse is caring for a patient who has an intraaortic balloon pump in place. Which action should be included in the plan of care? a. Position the patient supine at all times. b. Avoid the use of anticoagulant medications. c. Measure the patient's urinary output every hour. d. Provide passive range of motion for all extremities.

ANS: C Monitoring urine output will help determine whether the patient's cardiac output has improved and also help monitor for balloon displacement. The head of the bed can be elevated up to 30 degrees. Heparin is used to prevent thrombus formation. Limited movement is allowed for the extremity with the balloon insertion site to prevent displacement of the balloon.

4. An older patient with cardiogenic shock is cool and clammy. Hemodynamic monitoring indicates a high systemic vascular resistance (SVR). Which intervention should the nurse anticipate? a. Increase the rate for the dopamine infusion. b. Decrease the rate for the nitroglycerin infusion. c. Increase the rate for the sodium nitroprusside infusion. d. Decrease the rate for the 5% dextrose in normal saline (D5/.9 NS) infusion.

ANS: C Nitroprusside is an arterial vasodilator and will decrease the SVR and afterload, which will improve cardiac output. Changes in the D5/.9 NS and nitroglycerin infusions will not directly decrease SVR. Increasing the dopamine will tend to increase SVR. DIF: Cognitive Level: Apply (application) REF: 1599 TOP: Nursing Process: Planning MSC: NCLEX: Physiological Integrity

The nurse is taking care of a 7-year-old child with herpes simplex virus (type 1 or 2). Which prescribed medication should the nurse expect to be included in the treatment plan? a. Corticosteroids b. Oral griseofulvin c. Oral antiviral agent d. Topical and/or systemic antibiotic

ANS: C Oral antiviral agents are effective for viral infections such as herpes simplex. Corticosteroids are not effective for viral infections. Griseofulvin is an antifungal agent and not effective for viral infections. Antibiotics are not effective in viral diseases.

When caring for a patient with pulmonary hypertension, which parameter is most appropriate for the nurse to monitor to evaluate the effectiveness of the treatment? a. Central venous pressure (CVP) b. Systemic vascular resistance (SVR) c. Pulmonary vascular resistance (PVR) d. Pulmonary artery wedge pressure (PAWP)

ANS: C PVR is a major contributor to pulmonary hypertension, and a decrease would indicate that pulmonary hypertension was improving. The other parameters also may be monitored but do not directly assess for pulmonary hypertension.

16. When the nurse educator is evaluating the skills of a new registered nurse (RN) caring for patients experiencing shock, which action by the new RN indicates a need for more education? a. Placing the pulse oximeter on the ear for a patient with septic shock b. Keeping the head of the bed flat for a patient with hypovolemic shock c. Maintaining a cool room temperature for a patient with neurogenic shock d. Increasing the nitroprusside infusion rate for a patient with a very high SVR

ANS: C Patients with neurogenic shock have poikilothermia. The room temperature should be kept warm to avoid hypothermia. The other actions by the new RN are appropriate. DIF: Cognitive Level: Apply (application) REF: 1590 OBJ: Special Questions: Delegation TOP: Nursing Process: Evaluation MSC: NCLEX: Safe and Effective Care Environment

6. To evaluate the effectiveness of the pantoprazole (Protonix) ordered for a patient with systemic inflammatory response syndrome (SIRS), which assessment will the nurse perform? a. Auscultate bowel sounds. c. Check stools for occult blood. b. Ask the patient about nausea. d. Palpate for abdominal tenderness.

ANS: C Proton pump inhibitors are given to decrease the risk for stress ulcers in critically ill patients. The other assessments will also be done, but these will not help in determining the effectiveness of the pantoprazole administration. DIF: Cognitive Level: Apply (application) REF: 1606 TOP: Nursing Process: Evaluation MSC: NCLEX: Physiological Integrity

The nurse is taking care of a child with scabies. Which primary clinical manifestation should the nurse expect to assess with this disease? a. Edema b. Redness c. Pruritus d. Maceration

ANS: C Scabies is caused by the scabies mite. The inflammatory response and intense itching occur after the host has become sensitized to the mite. This occurs approximately 30 to 60 days after initial contact. Edema, redness, and maceration are not observed in scabies.

A nurse is weaning a 68-kg male patient who has chronic obstructive pulmonary disease (COPD) from mechanical ventilation. Which patient assessment finding indicates that the weaning protocol should be stopped? a. The patient's heart rate is 97 beats/min. b. The patient's oxygen saturation is 93%. c. The patient respiratory rate is 32 breaths/min. d. The patient's spontaneous tidal volume is 450 mL.

ANS: C Tachypnea is a sign that the patient's work of breathing is too high to allow weaning to proceed. The patient's heart rate is within normal limits, although the nurse should continue to monitor it. An oxygen saturation of 93% is acceptable for a patient with COPD. A spontaneous tidal volume of 450 mL is within the acceptable range.

The commonly used Guthrie blood test is performed on newborns to diagnose: a. Down syndrome. b. isoimmunization. c. PKU. d. congenital hypothyroidism (CH).

ANS: C The Guthrie blood test is an assay commonly used to diagnosis PKU. The test should be performed after the newborn has received postnatal feedings. Down syndrome is diagnosed through chromosomal analysis. Isoimmunization is detected by analysis of blood for unexpected antibodies. CH is diagnosed by analysis of a filter paper blood spot for thyroxine (T4).

21. A patient who has neurogenic shock is receiving a phenylephrine infusion through a right forearm IV. Which assessment finding obtained by the nurse indicates a need for immediate action? a. The patient's heart rate is 58 beats/min. b. The patient's extremities are warm and dry. c. The patient's IV infusion site is cool and pale. d. The patient's urine output is 28 mL over the past hour.

ANS: C The coldness and pallor at the infusion site suggest extravasation of the phenylephrine. The nurse should discontinue the IV and, if possible, infuse the drug into a central line. An apical pulse of 58 beats/min is typical for neurogenic shock but does not indicate an immediate need for nursing intervention. A 28-mL urinary output over 1 hour would require the nurse to monitor the output over the next hour, but an immediate change in therapy is not indicated. Warm, dry skin is consistent with early neurogenic shock, but it does not indicate a need for a change in therapy or immediate action. DIF: Cognitive Level: Analyze (analysis) REF: 1599 OBJ: Special Questions: Prioritization TOP: Nursing Process: Assessment MSC: NCLEX: Physiological Integrity

After a laryngectomy, a patient coughs violently during suctioning and dislodges the tracheostomy tube. Which action should the nurse take first? a. Arrange for arterial blood gases to be drawn immediately. b. Cover stoma with sterile gauze and ventilate through stoma. c. Attempt to reinsert the tracheostomy tube with the obturator in place. d. Assess the patient's oxygen saturation and notify the health care provider.

ANS: C The first action should be to attempt to reinsert the tracheostomy tube to maintain the patient's airway. Covering the stoma with a dressing and manually ventilating the patient may be an appropriate action if the nurse is unable to reinsert the tracheostomy tube. Assessing the patient's oxygenation is an important action, but it is not as appropriate until there is an established airway. DIF: Cognitive Level: Analyze (analysis) REF: 488 OBJ: Special Questions: Prioritization TOP: Nursing Process: Implementation MSC: NCLEX: Physiological Integrity

The nurse is caring for a patient with idiopathic pulmonary arterial hypertension (IPAH). Which assessment information requires the most immediate action by the nurse? a. The O2 saturation is 90%. b. The blood pressure is 98/56 mm Hg. c. The epoprostenol (Flolan) infusion is disconnected. d. The international normalized ratio (INR) is prolonged.

ANS: C The half-life of this drug is 6 minutes, so the nurse will need to restart the infusion as soon as possible to prevent rapid clinical deterioration. The other data also indicate a need for ongoing monitoring or intervention, but the priority action is to reconnect the infusion. DIF: Cognitive Level: Analyze (analysis) REF: 532 OBJ: Special Questions: Prioritization TOP: Nursing Process: Assessment MSC: NCLEX: Physiological Integrity

Herpes zoster is caused by the varicella virus and has an affinity for: a. sympathetic nerve fibers. b. parasympathetic nerve fibers. c. posterior root ganglia and posterior horn of the spinal cord. d. lateral and dorsal columns of the spinal cord.

ANS: C The herpes zoster virus has an affinity for posterior root ganglia, the posterior horn of the spinal cord, and skin. The zoster virus does not involve sympathetic or parasympathetic nerve fibers and the lateral and dorsal columns of the spinal cord.

Which assessment finding obtained by the nurse when caring for a patient receiving mechanical ventilation indicates the need for suctioning? a. The patient's oxygen saturation is 93%. b. The patient was last suctioned 6 hours ago. c. The patient's respiratory rate is 32 breaths/minute. d. The patient has occasional audible expiratory wheezes.

ANS: C The increase in respiratory rate indicates that the patient may have decreased airway clearance and requires suctioning. Suctioning is done when patient assessment data indicate that it is needed, not on a scheduled basis. Occasional expiratory wheezes do not indicate poor airway clearance, and suctioning the patient may induce bronchospasm and increase wheezing. An oxygen saturation of 93% is acceptable and does not suggest that immediate suctioning is needed.

44. Which action by the nurse will be most effective in decreasing the spread of pertussis in a community setting? a. Providing supportive care to patients diagnosed with pertussis b. Teaching family members about the need for careful hand washing c. Teaching patients about the need for adult pertussis immunizations d. Encouraging patients to complete the prescribed course of antibiotics

ANS: C The increased rate of pertussis in adults is thought to be caused by decreasing immunity after childhood immunization. Immunization is the most effective method of protecting communities from infectious diseases. Hand washing should be taught, but pertussis is spread by droplets and contact with secretions. Supportive care does not shorten the course of the disease or the risk for transmission. Taking antibiotics as prescribed does assist with decreased transmission, but patients are likely to have already transmitted the disease by the time the diagnosis is made. DIF: Cognitive Level: Analyze (analysis) REF: 500 TOP: Nursing Process: Implementation MSC: NCLEX: Health Promotion and Maintenance

18. A patient is admitted to the emergency department (ED) for shock of unknown etiology. The first action by the nurse should be to a. obtain the blood pressure. b. check the level of orientation. c. administer supplemental oxygen. d. obtain a 12-lead electrocardiogram.

ANS: C The initial actions of the nurse are focused on the ABCs—airway, breathing, and circulation—and administration of O2 should be done first. The other actions should be accomplished as rapidly as possible after providing O2. DIF: Cognitive Level: Analyze (analysis) REF: 1597 OBJ: Special Questions: Prioritization TOP: Nursing Process: Implementation MSC: NCLEX: Physiological Integrity

25. After reviewing the information shown in the accompanying figure for a patient with pneumonia and sepsis, which information is most important to report to the health care provider? Physical Assessment • Petechiae noted on chest and legs • Crackles heard bilaterally in lung bases • No redness or swelling at central line IV site Laboratory Data • Blood urea nitrogen (BUN) 34 mg/Dl • Hematocrit 30% • Platelets 50,000/µL Vital Signs • Temperature 100°F (37.8°C) • Pulse 102/min • Respirations 26/min • BP 110/60 mm Hg • O2 saturation 93% on 2L O2 via nasal cannula a. Temperature and IV site appearance b. Oxygen saturation and breath sounds c. Platelet count and presence of petechiae d. Blood pressure, pulse rate, respiratory rate.

ANS: C The low platelet count and presence of petechiae suggest that the patient may have disseminated intravascular coagulation and that multiple organ dysfunction syndrome is developing. The other information will also be discussed with the health care provider but does not indicate that the patient's condition is deteriorating or that a change in therapy is needed immediately. DIF: Cognitive Level: Analyze (analysis) REF: 1606 OBJ: Special Questions: Prioritization TOP: Nursing Process: Assessment MSC: NCLEX: Physiological Integrity

13. A nurse is caring for a patient who has had a total laryngectomy and radical neck dissection. During the first 24 hours after surgery what is the priority nursing action? a. Monitor the incision for bleeding. b. Maintain adequate IV fluid intake. c. Keep the patient in semi-Fowler's position. d. Teach the patient to suction the tracheostomy.

ANS: C The most important goals after a laryngectomy and radical neck dissection are to maintain the airway and ensure adequate oxygenation. Keeping the patient in a semi-Fowler's position will decrease edema and limit tension on the suture lines to help ensure an open airway. Maintenance of IV fluids and monitoring for bleeding are important, but maintaining an open airway is the priority. During the immediate postoperative period, the patient with a laryngectomy requires frequent suctioning of the tracheostomy tube. The patient may be taught to suction after the tracheostomy is stable, if needed, but not during the immediate postoperative period. DIF: Cognitive Level: Analyze (analysis) REF: 488 OBJ: Special Questions: Prioritization TOP: Nursing Process: Implementation MSC: NCLEX: Physiological Integrity

The nurse notes that a patient's endotracheal tube (ET), which was at the 22-cm mark, is now at the 25-cm mark and the patient is anxious and restless. Which action should the nurse take next? a. Offer reassurance to the patient. b. Bag the patient at an FIO2 of 100%. c. Listen to the patient's breath sounds. d. Notify the patient's health care provider.

ANS: C The nurse should first determine whether the ET tube has been displaced into the right mainstem bronchus by listening for unilateral breath sounds. If so, assistance will be needed to reposition the tube immediately. The other actions are also appropriate, but detection and correction of tube malposition are the most critical actions.

The nurse assesses vital signs for a patient admitted 2 days ago with gram-negative sepsis: temperature of 101.2° F, blood pressure of 90/56 mm Hg, pulse of 92 beats/min, and respirations of 34 breaths/min. Which action should the nurse take next? a. Give the scheduled IV antibiotic. b. Give the PRN acetaminophen (Tylenol). c. Obtain oxygen saturation using pulse oximetry. d. Notify the health care provider of the patient's vital signs.

ANS: C The patient's increased respiratory rate in combination with the admission diagnosis of gram-negative sepsis indicates that acute respiratory distress syndrome (ARDS) may be developing. The nurse should check for hypoxemia, a hallmark of ARDS. The health care provider should be notified after further assessment of the patient. Giving the scheduled antibiotic and the PRN acetaminophen will also be done, but they are not the highest priority for a patient who may be developing ARDS. DIF: Cognitive Level: Analyze (analysis)

The nurse is caring for an older patient who was hospitalized 2 days earlier with community-acquired pneumonia. Which assessment information is most important to communicate to the health care provider? a. Persistent cough of blood-tinged sputum. b. Scattered crackles in the posterior lung bases. c. Oxygen saturation 90% on 100% O2 by nonrebreather mask. d. Temperature 101.5° F (38.6° C) after 2 days of IV antibiotics.

ANS: C The patient's low SpO2 despite receiving a high fraction of inspired oxygen (FIO2) indicates the possibility of acute respiratory distress syndrome (ARDS). The patient's blood-tinged sputum and scattered crackles are not unusual in a patient with pneumonia, although they do require continued monitoring. The continued temperature elevation indicates a possible need to change antibiotics, but this is not as urgent a concern as the progression toward hypoxemia despite an increase in O2 flow rate. DIF: Cognitive Level: Analyze (analysis)

The nurse has just auscultated coarse crackles bilaterally on a patient with a tracheostomy tube in place. If the patient is unsuccessful in coughing up secretions, what action should the nurse take? a. Encourage increased incentive spirometer use. b. Encourage the patient to increase oral fluid intake. c. Put on sterile gloves and use a sterile catheter to suction. d. Preoxygenate the patient for 3 minutes before suctioning.

ANS: C This patient needs suctioning now to secure a patent airway. Sterile gloves and a sterile catheter are used when suctioning a tracheostomy. Preoxygenation for 3 minutes is not necessary; 30 seconds is recommended. Incentive spirometer use opens alveoli and can induce coughing, which can mobilize secretions. However, the patient with a tracheostomy may not be able to use an incentive spirometer. Increasing oral fluid intake would not moisten and help mobilize secretions in a timely manner. DIF: Cognitive Level: Apply (application) REF: 488 TOP: Nursing Process: Implementation MSC: NCLEX: Safe and Effective Care Environment

The nurse is caring for a patient receiving a continuous norepinephrine (Levophed) IV infusion. Which patient assessment finding indicates that the infusion rate may need to be adjusted? a. Heart rate is 58 beats/minute. b. Mean arterial pressure (MAP) is 56 mm Hg. c. Systemic vascular resistance (SVR) is elevated. d. Pulmonary artery wedge pressure (PAWP) is low.

ANS: C Vasoconstrictors such as norepinephrine (Levophed) will increase SVR, and this will increase the work of the heart and decrease peripheral perfusion. The infusion rate may need to be decreased. Bradycardia, hypotension (MAP of 56 mm Hg), and low PAWP are not associated with norepinephrine infusion.

A patient scheduled for a total laryngectomy and radical neck dissection for cancer of the larynx asks the nurse, "Will I be able to talk normally after surgery?" What is the most accurate response by the nurse? a. "You will breathe through a permanent opening in your neck, but you will not be able to communicate orally." b. "You won't be able to talk right after surgery, but you will be able to speak again after the tracheostomy tube is removed." c. "You will have a permanent opening into your neck, and you will need rehabilitation for some type of voice restoration." d. "You won't be able to speak as you used to, but there are artificial voice devices that will give you the ability to speak normally."

ANS: C Voice rehabilitation is planned after a total laryngectomy, and a variety of assistive devices are available to restore communication. Although the ability to communicate orally is changed, it would not be lost. Artificial voice devices do not permit normal-sounding speech. In a total laryngectomy, the vocal cords are removed, so normal speech is impossible. DIF: Cognitive Level: Apply (application) REF: 493 TOP: Nursing Process: Implementation MSC: NCLEX: Physiological Integrity

12. A nurse is assessing a patient who is receiving a nitroprusside infusion to treat cardiogenic shock. Which finding indicates that the drug is effective? a. No new heart murmurs b. Decreased troponin level c. Warm, pink, and dry skin d. Blood pressure of 92/40 mm Hg

ANS: C Warm, pink, and dry skin indicates that perfusion to tissues is improved. Because nitroprusside is a vasodilator, the blood pressure may be low even if the drug is effective. Absence of a heart murmur and a decrease in troponin level are not indicators of improvement in shock. DIF: Cognitive Level: Apply (application) REF: 1599 TOP: Nursing Process: Evaluation MSC: NCLEX: Physiological Integrity

5. After receiving 2 L of normal saline, the central venous pressure for a patient who has septic shock is 10 mm Hg, but the blood pressure is still 82/40 mm Hg. The nurse will anticipate an order for a. furosemide . c. norepinephrine . b. nitroglycerin . d. sodium nitroprusside .

ANS: C When fluid resuscitation is unsuccessful, vasopressor drugs are given to increase the systemic vascular resistance (SVR) and blood pressure and improve tissue perfusion. Furosemide would cause diuresis and further decrease the BP. Nitroglycerin would decrease the preload and further drop cardiac output and BP. Nitroprusside is an arterial vasodilator and would further decrease SVR. DIF: Cognitive Level: Apply (application) REF: 1599 TOP: Nursing Process: Planning MSC: NCLEX: Physiological Integrity

2. The emergency department (ED) nurse is starting therapeutic hypothermia in a patient who has been resuscitated after a cardiac arrest. Which actions in the hypothermia protocol can be delegated to an experienced licensed practical/vocational nurse (LPN/LVN) (select all that apply)? a. Continuously monitor heart rhythm. b. Assess neurologic status every 2 hours. c. Give acetaminophen (Tylenol) 650 mg. d. Place cooling blankets above and below patient. e. Attach rectal temperature probe to cooling blanket control panel.

ANS: C, D, E Experienced LPN/LVNs have the education and scope of practice to implement hypothermia measures (e.g., cooling blanket, temperature probe) and administer medications under the supervision of a registered nurse (RN). Assessment of neurologic status and monitoring the heart rhythm require RN-level education and scope of practice and should be done by the RN.

10. When rewarming a patient who arrived in the emergency department (ED) with a temperature of 87° F (30.6° C), which finding indicates that the nurse should discontinue active rewarming? a. The patient begins to shiver. b. The BP decreases to 86/42 mm Hg. c. The patient develops atrial fibrillation. d. The core temperature is 94° F (34.4° C).

ANS: D A core temperature of at least 89.6° F to 93.2° F (32° C to 34° C) indicates that sufficient rewarming has occurred. Dysrhythmias, hypotension, and shivering may occur during rewarming, and should be treated but are not an indication to stop rewarming the patient.

A patient with chronic obstructive pulmonary disease (COPD) arrives in the emergency department complaining of shortness of breath and dyspnea on minimal exertion. Which assessment finding by the nurse is most important to report to the health care provider? a. The patient has bibasilar lung crackles. b. The patient is sitting in the tripod position. c. The patient's pulse oximetry indicates a 91% O2 saturation. d. The patient's respirations have dropped to 10 breaths/minute.

ANS: D A drop in respiratory rate in a patient with respiratory distress suggests the onset of fatigue and a high risk for respiratory arrest. Therefore immediate action such as positive-pressure ventilation is needed. Patients who are experiencing respiratory distress frequently sit in the tripod position because it decreases the work of breathing. Crackles in the lung bases may be the baseline for a patient with COPD. An O2 saturation of 91% is common in patients with COPD and will provide adequate gas exchange and tissue oxygenation.

8. The emergency department (ED) nurse receives report that a seriously injured patient involved in a motor vehicle crash is being transported to the facility with an estimated arrival in 5 minutes. In preparation for the patient's arrival, the nurse will obtain a. a dopamine infusion. b. a hypothermia blanket. c. lactated Ringer's solution. d. two 16-gauge IV catheters.

ANS: D A patient with multiple trauma may require fluid resuscitation to prevent or treat hypovolemic shock, so the nurse will anticipate the need for 2 large-bore IV lines to administer normal saline. Lactated Ringer's solution should be used cautiously and will not be ordered until the patient has been assessed for possible liver abnormalities. Vasopressor infusion is not used as the initial therapy for hypovolemic shock. Patients in shock need to be kept warm not cool. DIF: Cognitive Level: Apply (application) REF: 1600 TOP: Nursing Process: Planning MSC: NCLEX: Physiological Integrity

A patient with pneumonia has a fever of 101.4° F (38.6° C), a nonproductive cough, and an O2 saturation of 88%. The patient complains of weakness, fatigue, and needs assistance to get out of bed. Which nursing diagnosis should the nurse assign as the priority? a. Hyperthermia related to infectious illness b. Impaired transfer ability related to weakness c. Ineffective airway clearance related to thick secretions d. Impaired gas exchange related to respiratory congestion

ANS: D All of these nursing diagnoses are appropriate for the patient, but the patient's O2 saturation indicates that all body tissues are at risk for hypoxia unless the gas exchange is improved. DIF: Cognitive Level: Analyze (analysis) REF: 505 OBJ: Special Questions: Prioritization TOP: Nursing Process: Diagnosis MSC: NCLEX: Physiological Integrity

When assessing a patient with chronic obstructive pulmonary disease (COPD), the nurse finds a new onset of agitation and confusion. Which action should the nurse take first? a. Observe for facial symmetry. b. Notify the health care provider. c. Attempt to calm and reorient the patient. d. Assess oxygenation using pulse oximetry.

ANS: D Because agitation and confusion are frequently the initial indicators of hypoxemia, the nurse's initial action should be to assess O2 saturation. The other actions are also appropriate, but assessment of oxygenation takes priority over other assessments and notification of the health care provider. DIF: Cognitive Level: Analyze (analysis)

A patient with acute respiratory distress syndrome (ARDS) who is intubated and receiving mechanical ventilation develops a right pneumothorax. Which collaborative action will the nurse anticipate next? a. Increase the tidal volume and respiratory rate. b. Decrease the fraction of inspired oxygen (FIO2). c. Perform endotracheal suctioning more frequently. d. Lower the positive end-expiratory pressure (PEEP).

ANS: D Because barotrauma is associated with high airway pressures, the level of PEEP should be decreased. The other actions will not decrease the risk for another pneumothorax. DIF: Cognitive Level: Apply (application)

13. Which assessment information is most important for the nurse to obtain when evaluating whether treatment of a patient with anaphylactic shock has been effective? a. Heart rate b. Orientation c. Blood pressure d. Oxygen saturation

ANS: D Because the airway edema that is associated with anaphylaxis can affect airway and breathing, the O2 saturation is the most critical assessment. Improvements in the other assessments will also be expected with effective treatment of anaphylactic shock. DIF: Cognitive Level: Analyze (analysis) REF: 1600 TOP: Nursing Process: Evaluation MSC: NCLEX: Physiological Integrity

A patient arrives in the ear, nose, and throat clinic complaining of a piece of tissue being "stuck up my nose" and with foul-smelling nasal drainage from the right nare. Which action should the nurse take first? a. Notify the clinic health care provider. b. Obtain aerobic culture specimens of the drainage. c. Ask the patient about how the cotton got into the nose. d. Have the patient occlude the left nare and blow the nose.

ANS: D Because the highest priority action is to remove the foreign object from the nare, the nurse's first action should be to assist the patient to remove the object. The other actions are also appropriate but should be done after attempting to clear the nose. DIF: Cognitive Level: Analyze (analysis) REF: 482 OBJ: Special Questions: Prioritization TOP: Nursing Process: Implementation MSC: NCLEX: Physiological Integrity

A patient is diagnosed with both human immunodeficiency virus (HIV) and active tuberculosis (TB) disease. Which information obtained by the nurse is most important to communicate to the health care provider? a. The Mantoux test had an induration of 7 mm. b. The chest-x-ray showed infiltrates in the lower lobes. c. The patient has a cough that is productive of blood-tinged mucus. d. The patient is being treated with antiretrovirals for HIV infection.

ANS: D Drug interactions can occur between the antiretrovirals used to treat HIV infection and the medications used to treat TB. The other data are expected in a patient with HIV and TB. DIF: Cognitive Level: Analyze (analysis) REF: 509 OBJ: Special Questions: Prioritization TOP: Nursing Process: Assessment MSC: NCLEX: Physiological Integrity

Which action will the nurse need to do when preparing to assist with the insertion of a pulmonary artery catheter? a. Determine if the cardiac troponin level is elevated. b. Auscultate heart and breath sounds during insertion. c. Place the patient on NPO status before the procedure. d. Attach cardiac monitoring leads before the procedure.

ANS: D Dysrhythmias can occur as the catheter is floated through the right atrium and ventricle, and it is important for the nurse to monitor for these during insertion. Pulmonary artery catheter insertion does not require anesthesia, and the patient will not need to be NPO. Changes in cardiac troponin or heart and breath sounds are not expected during pulmonary artery catheter insertion.

Trisomy 13, trisomy 18, and trisomy 21 have which in common? a. Viability is rare. b. They are considered deletion syndromes. c. Diagnosis is difficult, time-consuming, and expensive. d. Diagnosis can be made early, based on physical characteristics.

ANS: D Each of these disorders, trisomy 13, 18, and 21, has unique physical characteristics. A presumptive diagnosis can often be made soon after birth and later confirmed by chromosomal analysis. Children with trisomy 13 and 18 usually have short life expectancies. Trisomy 21 has a variable life expectancy, with 80% of individuals living to age 30 years. Trisomy 13, trisomy 18, and trisomy 21 are not deletion syndromes.

Which nursing interventions included in the care of a mechanically ventilated patient with acute respiratory failure can the registered nurse (RN) delegate to an experienced licensed practical/vocational nurse (LPN/LVN) working in the intensive care unit? a. Assess breath sounds every hour. b. Monitor central venous pressures. c. Place patient in the prone position. d. Insert an indwelling urinary catheter.

ANS: D Insertion of indwelling urinary catheters is included in LPN/LVN education and scope of practice and can be safely delegated to an LPN/LVN who is experienced in caring for critically ill patients. Placing a patient who is on a ventilator in the prone position requires multiple staff, and should be supervised by an RN. Assessment of breath sounds and obtaining central venous pressures require advanced assessment skills and should be done by the RN caring for a critically ill patient. DIF: Cognitive Level: Apply (application)

A parent has asked the nurse about how her child can be tested for pinworms. The nurse responds by stating that which is the most common test for diagnosing pinworms in a child? a. Lower gastrointestinal (GI) series b. Three stool specimens, at intervals of 4 days c. Observation for presence of worms after child defecates d. Laboratory examination of a fecal smear

ANS: D Laboratory examination of substances containing the worm, its larvae, or ova can identify the organism. Most are identified by examining fecal smears from the stools of persons suspected of harboring the parasite. Fresh specimens are best for revealing parasites or larvae. Lower GI series is not helpful for diagnosing enterobiasis. Stool specimens are not necessary to diagnose pinworms. Worms will not be visible after child defecates.

. Which intervention will the nurse include in the plan of care for a patient who is diagnosed with a lung abscess? a. Teach the patient to avoid the use of over-the-counter expectorants. b. Assist the patient with chest physiotherapy and postural drainage. c. Notify the health care provider immediately about any bloody or foul-smelling sputum. d. Teach about the need for prolonged antibiotic therapy after discharge from the hospital.

ANS: D Long-term antibiotic therapy is needed for effective eradication of the infecting organisms in lung abscess. Chest physiotherapy and postural drainage are not recommended for lung abscess because they may lead to spread of the infection. Foul-smelling and bloody sputum are common clinical manifestations in lung abscess. Expectorants may be used because the patient is encouraged to cough. DIF: Cognitive Level: Apply (application) REF: 512 TOP: Nursing Process: Planning MSC: NCLEX: Physiological Integrity

The nurse completes discharge teaching for a patient who has had a lung transplant. Which patient statement indicates to the nurse that the teaching has been effective? a. "I will make an appointment to see the doctor every year." b. "I will stop taking the prednisone if I experience a dry cough." c. "I will not worry if I feel a little short of breath with exercise." d. "I will call the health care provider right away if I develop a fever."

ANS: D Low-grade fever may indicate infection or acute rejection so the patient should notify the health care provider immediately if the temperature is elevated. Patients require frequent follow-up visits with the transplant team. Annual health care provider visits would not be sufficient. Home O2 use is not an expectation after lung transplant. Shortness of breath should be reported. Low-grade fever, fatigue, dyspnea, dry cough, and O2 desaturation are signs of rejection. Immunosuppressive therapy, including prednisone, needs to be continued to prevent rejection. DIF: Cognitive Level: Apply (application) REF: 534 TOP: Nursing Process: Evaluation MSC: NCLEX: Physiological Integrity

1. The nurse teaches a patient about discharge instructions after a rhinoplasty. Which statement, if made by the patient, indicates that the teaching was successful? a. "My nose will look normal after 24 to 48 hours." b. "I can take 800 mg ibuprofen every 6 hours for pain." c. "I will remove and reapply the nasal packing every day." d. "I will elevate my head for 48 hours to minimize swelling."

ANS: D Maintaining the head in an elevated position will decrease the amount of nasal swelling. Nonsteroidal antiinflammatory drugs, such as ibuprofen, increase the risk for postoperative bleeding and should not be used postoperatively. The patient would not remove or reapply nasal packing, which is usually removed by the surgeon on the day after surgery. Although return to a preinjury appearance is the goal of the surgery, it is not always possible to achieve this result, especially in the first few weeks after surgery. DIF: Cognitive Level: Apply (application) REF: 476 TOP: Nursing Process: Implementation MSC: NCLEX: Physiological Integrity

14. The emergency department (ED) triage nurse is assessing four victims involved in a motor vehicle collision. Which patient has the highest priority for treatment? a. A patient with no pedal pulses b. A patient with an open femur fracture c. A patient with bleeding facial lacerations d. A patient with paradoxical chest movement

ANS: D Most immediate deaths from trauma occur because of problems with ventilation, so the patient with paradoxical chest movements should be treated first. Face and head fractures can obstruct the airway, but the patient with facial injuries only has lacerations. The other two patients also need rapid intervention but do not have airway or breathing problems.

Which is characteristic of newborns whose mothers smoked during pregnancy? a. Large for gestational age b. Preterm, but size appropriate for gestational age c. Growth retardation in weight only d. Growth retardation in weight, length, and head circumference

ANS: D Newborns born to mothers who smoke had growth failure in weight, length, and chest circumference when compared with newborns of mothers who did not smoke. A dose-effect relation exists. Newborns have significant growth failure, which is related to the number of cigarettes smoked.

The nurse is caring for a patient who has acute pharyngitis caused by Candida albicans. Which action is appropriate for the nurse to include in the plan of care? a. Assess patient for allergies to penicillin antibiotics. b. Teach the patient to sleep in a warm, dry environment. c. Avoid giving the patient warm food or warm liquids to drink. d. Teach patient to "swish and swallow" prescribed oral nystatin

ANS: D Oral or pharyngeal fungal infections are treated with nystatin solution. The goal of the "swish and swallow" technique is to expose all of the oral mucosa to the antifungal agent. Warm liquids may be soothing to a sore throat. The patient should be taught to use a cool mist humidifier. There is no need to assess for penicillin and cephalosporin allergies because Candida albicans infection is treated with antifungals. DIF: Cognitive Level: Apply (application) REF: 484 TOP: Nursing Process: Planning MSC: NCLEX: Physiological Integrity

When monitoring for the effectiveness of treatment for a patient with a large anterior wall myocardial infarction, the most important information for the nurse to obtain is a. central venous pressure (CVP). b. systemic vascular resistance (SVR). c. pulmonary vascular resistance (PVR). d. pulmonary artery wedge pressure (PAWP).

ANS: D PAWP reflects left ventricular end diastolic pressure (or left ventricular preload) and is a sensitive indicator of cardiac function. Because the patient is high risk for left ventricular failure, the PAWP must be monitored. An increase will indicate left ventricular failure. The other values would also provide useful information, but the most definitive measurement of changes in cardiac function is the PAWP.

The nurse is performing tuberculosis (TB) skin tests in a clinic that has many patients who have immigrated to the United States. Which question is most important for the nurse to ask before the skin test? a. "Do you take any over-the-counter (OTC) medications?" b. "Do you have any family members with a history of TB?" c. "How long has it been since you moved to the United States?" d. "Did you receive the bacille Calmette-Guérin (BCG) vaccine for TB?"

ANS: D Patients who have received the BCG vaccine will have a positive Mantoux test. Another method for screening (e.g., chest x-ray) will need to be used in determining whether the patient has a TB infection. The other information also may be valuable but is not as pertinent to the decision about doing TB skin testing. DIF: Cognitive Level: Analyze (analysis) REF: 508 TOP: Nursing Process: Assessment MSC: NCLEX: Physiological Integrity

Parents tell the nurse that their child keeps scratching the areas where he has poison ivy. The nurse's response should be based on which knowledge? a. Poison ivy does not itch and needs further investigation. b. Scratching the lesions will not cause a problem. c. Scratching the lesions will cause the poison ivy to spread. d. Scratching the lesions may cause them to become secondarily infected.

ANS: D Poison ivy is a contact dermatitis that results from exposure to the oil urushiol in the plant. Every effort is made to prevent the child from scratching because the lesions can become secondarily infected. The poison ivy produces localized, streaked or spotty, oozing, and painful impetiginous lesions. Itching is a common response. Scratching the lesions can result in secondary infections. The lesions do not spread by contact with the blister serum or by scratching.

The nurse supervises unlicensed assistive personnel (UAP) who are providing care for a patient with right lower lobe pneumonia. The nurse should intervene if which action by UAP is observed? a. UAP assist the patient to ambulate to the bathroom. b. UAP help splint the patient's chest during coughing. c. UAP transfer the patient to a bedside chair for meals. d. UAP lower the head of the patient's bed to 15 degrees.

ANS: D Positioning the patient with the head of the bed lowered will decrease ventilation. The other actions are appropriate for a patient with pneumonia. DIF: Cognitive Level: Apply (application) REF: 505 OBJ: Special Questions: Delegation TOP: Nursing Process: Implementation MSC: NCLEX: Safe and Effective Care Environment

47. The nurse provides discharge teaching for a patient who has two fractured ribs from an automobile accident. Which statement, if made by the patient, would indicate that teaching has been effective? a. "I am going to buy a rib binder to wear during the day." b. "I can take shallow breaths to prevent my chest from hurting." c. "I should plan on taking the pain pills only at bedtime so I can sleep." d. "I will use the incentive spirometer every hour or two during the day."

ANS: D Prevention of the complications of atelectasis and pneumonia is a priority after rib fracture. This can be ensured by deep breathing and coughing. Use of a rib binder, shallow breathing, and taking pain medications only at night are likely to result in atelectasis. DIF: Cognitive Level: Apply (application) REF: 521 TOP: Nursing Process: Evaluation MSC: NCLEX: Physiological Integrity

A patient develops increasing dyspnea and hypoxemia 2 days after heart surgery. To determine whether the patient has acute respiratory distress syndrome (ARDS) or pulmonary edema caused by heart failure, the nurse will plan to assist with a. obtaining a ventilation-perfusion scan. b. drawing blood for arterial blood gases. c. positioning the patient for a chest x-ray. d. insertion of a pulmonary artery catheter.

ANS: D Pulmonary artery wedge pressures are normal in the patient with ARDS because the fluid in the alveoli is caused by increased permeability of the alveolar-capillary membrane rather than by the backup of fluid from the lungs (as occurs in cardiogenic pulmonary edema). The other tests will not help in differentiating cardiogenic from noncardiogenic pulmonary edema. DIF: Cognitive Level: Apply (application)

Which is a common childhood communicable disease that may cause severe defects in the fetus when it occurs in its congenital form? a. Erythema infectiosum b. Roseola c. Rubeola d. Rubella

ANS: D Rubella causes teratogenic effects on the fetus. There is a low risk of fetal death to those in contact with children affected with fifth disease. Roseola and rubeola are not dangerous to the fetus.

A mother tells the nurse that she does not want her infant immunized because of the discomfort associated with injections. What should the nurse explain? a. This cannot be prevented. b. Infants do not feel pain as adults do. c. This is not a good reason for refusing immunizations. d. A topical anesthetic, EMLA, can be applied before injections are given.

ANS: D Several topical anesthetic agents can be used to minimize the discomfort associated with immunization injections. These include EMLA (eutectic mixture of local anesthetic) and vapor coolant sprays. Pain associated with many procedures can be prevented and minimized by using the principles of atraumatic care. With preparation, the injection site can be properly anesthetized to decrease the amount of pain felt by the infant. Infants have the neural pathways to feel pain. Numerous research studies have indicated that infants perceive and react to pain in the same manner as do children and adults. The mother should be allowed to discuss her concerns and the alternatives available. This is part of the informed consent process.

What is cellulitis often caused by? a. Herpes zoster b. Candida albicans c. Human papillomavirus d. Streptococcus or Staphylococcus organisms

ANS: D Streptococci, staphylococci, and Haemophilus influenzae are the organisms usually responsible for cellulitis. Herpes zoster is the virus associated with varicella and shingles. C. albicans is associated with candidiasis, or thrush. Human papillomavirus is associated with various types of human warts.

2. During the primary survey of a patient with severe leg trauma, the nurse observes that the patient's left pedal and posterior tibial pulses are absent and the entire leg is swollen. Which action will the nurse take next? a. Send blood to the lab for a complete blood count. b. Assess further for a cause of the decreased circulation. c. Finish the airway, breathing, circulation, disability survey. d. Start normal saline fluid infusion with a large-bore IV line.

ANS: D The assessment data indicate that the patient may have arterial trauma and hemorrhage. When a possibly life-threatening injury is found during the primary survey, the nurse should immediately start interventions before proceeding with the survey. Although a complete blood count is indicated, administration of IV fluids should be started first. Completion of the primary survey and further assessment should be completed after the IV fluids are initiated.

After change-of-shift report on a ventilator weaning unit, which patient should the nurse assess first? a. Patient who failed a spontaneous breathing trial and has been placed in a rest mode on the ventilator b. Patient who is intubated and has continuous partial pressure end-tidal CO2 (PETCO2) monitoring c. Patient with a central venous oxygen saturation (ScvO2) of 69% while on bilevel positive airway pressure (BiPAP) d. Patient who was successfully weaned and extubated 4 hours ago and now has no urine output for the last 6 hours

ANS: D The decreased urine output may indicate acute kidney injury or that the patient's cardiac output and perfusion of vital organs have decreased. Any of these causes would require rapid action. The data about the other patients indicate that their conditions are stable and do not require immediate assessment or changes in their care. Continuous PETCO2 monitoring is frequently used when patients are intubated. The rest mode should be used to allow patient recovery after a failed SBT, and an ScvO2 of 69% is within normal limits.

. A patient is admitted to the emergency department with an open stab wound to the left chest. What action should the nurse take? a. Keep the head of the patient's bed positioned flat. b. Cover the wound tightly with an occlusive dressing. c. Position the patient so that the left chest is dependent. d. Tape a nonporous dressing on three sides over the wound.

ANS: D The dressing taped on three sides will allow air to escape when intrapleural pressure increases during expiration, but it will prevent air from moving into the pleural space during inspiration. Placing the patient on the left side or covering the chest wound with an occlusive dressing will allow trapped air in the pleural space and cause tension pneumothorax. The head of the bed should be elevated to 30 to 45 degrees to facilitate breathing. DIF: Cognitive Level: Apply (application) REF: 519 TOP: Nursing Process: Implementation MSC: NCLEX: Physiological Integrity

After change-of-shift report, which patient should the progressive care nurse assess first? a. Patient who was extubated in the morning and has a temperature of 101.4° F (38.6° C) b. Patient with bilevel positive airway pressure (BiPAP) for sleep apnea whose respiratory rate is 16 c. Patient with arterial pressure monitoring who is 2 hours post-percutaneous coronary intervention who needs to void d. Patient who is receiving IV heparin for a venous thromboembolism and has a partial thromboplastin time (PTT) of 98 sec

ANS: D The findings for this patient indicate high risk for bleeding from an elevated (nontherapeutic) PTT. The nurse needs to adjust the rate of the infusion (dose) per the health care provider's parameters. The patient with BiPAP for sleep apnea has a normal respiratory rate. The patient recovering from the percutaneous coronary intervention will need to be assisted with voiding and this task could be delegated to unlicensed assistive personnel. The patient with a fever may be developing ventilator-associated pneumonia, but addressing the bleeding risk is a higher priority.

The nurse educator is evaluating the care that a new registered nurse (RN) provides to a patient receiving mechanical ventilation. Which action by the new RN indicates the need for more education? a. The RN increases the FIO2 to 100% before suctioning. b. The RN secures a bite block in place using adhesive tape. c. The RN asks for assistance to reposition the endotracheal tube. d. The RN positions the patient with the head of bed at 10 degrees.

ANS: D The head of the patient's bed should be positioned at 30 to 45 degrees to prevent ventilator-associated pneumonia. The other actions by the new RN are appropriate.

17. A patient arrives in the emergency department (ED) after topical exposure to powdered lime at work. Which action should the nurse take first? a. Obtain the patient's vital signs. b. Obtain a baseline complete blood count. c. Decontaminate the patient by showering with water. d. Brush off any visible powder on the skin and clothing.

ANS: D The initial action should be to protect staff members and decrease the patient's exposure to the toxin by decontamination. Patients exposed to powdered lime should not be showered; instead, any and all visible powder should be brushed off. The other actions can be done after the decontamination is completed.

While waiting for cardiac transplantation, a patient with severe cardiomyopathy has a ventricular assist device (VAD) implanted. When planning care for this patient, the nurse should anticipate a. giving immunosuppressive medications. b. preparing the patient for a permanent VAD. c. teaching the patient the reason for complete bed rest. d. monitoring the surgical incision for signs of infection.

ANS: D The insertion site for the VAD provides a source for transmission of infection to the circulatory system and requires frequent monitoring. Patient's with VADs are able to have some mobility and may not be on bed rest. The VAD is a bridge to transplantation, not a permanent device. Immunosuppression is not necessary for nonbiologic devices like the VAD.

To maintain proper cuff pressure of an endotracheal tube (ET) when the patient is on mechanical ventilation, the nurse should a. inflate the cuff with a minimum of 10 mL of air. b. inflate the cuff until the pilot balloon is firm on palpation. c. inject air into the cuff until a manometer shows 15 mm Hg pressure. d. inject air into the cuff until a slight leak is heard only at peak inflation.

ANS: D The minimal occluding volume technique involves injecting air into the cuff until an air leak is present only at peak inflation. The volume to inflate the cuff varies with the ET and the patient's size. Cuff pressure should be maintained at 20 to 25 mm Hg. An accurate assessment of cuff pressure cannot be obtained by palpating the pilot balloon.

Which should the nurse anticipate in the newborn whose mother used cocaine during pregnancy? a. Seizures b. Hyperglycemia c. Cardiac and respiratory problems d. Neurobehavioral depression or excitability

ANS: D The nurse should anticipate neurobehavioral depression or excitability and implement care directed at the newborn's manifestations. Few or no neurologic sequelae appear in newborns born to mothers who use cocaine during pregnancy. The newborn is usually a poor feeder, so hypoglycemia would be a more likely occurrence. Cardiac and respiratory problems are usually not evident in these newborns.

The nurse responds to a ventilator alarm and finds the patient lying in bed holding the endotracheal tube (ET). Which action should the nurse take next? a. Activate the rapid response team. b. Provide reassurance to the patient. c. Call the health care provider to reinsert the tube. d. Manually ventilate the patient with 100% oxygen.

ANS: D The nurse should ensure maximal patient oxygenation by manually ventilating with a bag-valve-mask system. Offering reassurance to the patient, notifying the health care provider about the need to reinsert the tube, and activating the rapid response team are also appropriate after the nurse has stabilized the patient's oxygenation.

36. A patient with a possible pulmonary embolism complains of chest pain and difficulty breathing. The nurse finds a heart rate of 142 beats/min, blood pressure of 100/60 mm Hg, and respirations of 42 breaths/min. Which action should the nurse take first? a. Administer anticoagulant drug therapy. b. Notify the patient's health care provider. c. Prepare patient for a spiral computed tomography (CT). d. Elevate the head of the bed to a semi-Fowler's position.

ANS: D The patient has symptoms consistent with a pulmonary embolism (PE). Elevating the head of the bed will improve ventilation and gas exchange. The other actions can be accomplished after the head is elevated (and O2 is started). A spiral CT may be ordered by the health care provider to identify PE. Anticoagulants may be ordered after confirmation of the diagnosis of PE. DIF: Cognitive Level: Analyze (analysis) REF: 531 OBJ: Special Questions: Prioritization TOP: Nursing Process: Implementation MSC: NCLEX: Physiological Integrity

Four hours after mechanical ventilation is initiated for a patient with chronic obstructive pulmonary disease (COPD), the patient's arterial blood gas (ABG) results include a pH of 7.51, PaO2 of 82 mm Hg, PaCO2 of 26 mm Hg, and HCO3- of 23 mEq/L (23 mmol/L). The nurse will anticipate the need to a. increase the FIO2. b. increase the tidal volume. c. increase the respiratory rate. d. decrease the respiratory rate.

ANS: D The patient's PaCO2 and pH indicate respiratory alkalosis caused by too high a respiratory rate. The PaO2 is appropriate for a patient with COPD and increasing the respiratory rate and tidal volume would further lower the PaCO2.

49. After change-of-shift report, which patient should the nurse assess first? a. A 72-yr-old with cor pulmonale who has 4+ bilateral edema in his legs and feet b. A 28-yr-old with a history of a lung transplant and a temperature of 101° F (38.3° C) c. A 40-yr-old with a pleural effusion who is complaining of severe stabbing chest pain d. A 64-yr-old with lung cancer and tracheal deviation after subclavian catheter insertion

ANS: D The patient's history and symptoms suggest possible tension pneumothorax, a medical emergency. The other patients also require assessment as soon as possible, but tension pneumothorax will require immediate treatment to avoid death from inadequate cardiac output or hypoxemia. DIF: Cognitive Level: Analyze (analysis) REF: 520 OBJ: Special Questions: Prioritization | Special Questions: Multiple Patients TOP: Nursing Process: Assessment MSC: NCLEX: Safe and Effective Care Environment

When assisting with the placement of a pulmonary artery (PA) catheter, the nurse notes that the catheter is correctly placed when the monitor shows a a. typical PA pressure waveform. b. tracing of the systemic arterial pressure. c. tracing of the systemic vascular resistance. d. typical PA wedge pressure (PAWP) tracing.

ANS: D The purpose of a PA line is to measure PAWP, so the catheter is floated through the pulmonary artery until the dilated balloon wedges in a distal branch of the pulmonary artery, and the PAWP readings are available. After insertion, the balloon is deflated and the PA waveform will be observed. Systemic arterial pressures are obtained using an arterial line and the systemic vascular resistance is a calculated value, not a waveform.

After receiving change-of-shift report on a medical unit, which patient should the nurse assess first? a. A patient with cystic fibrosis who has thick, green-colored sputum b. A patient with pneumonia who has crackles bilaterally in the lung bases c. A patient with emphysema who has an oxygen saturation of 90% to 92% d. A patient with septicemia who has intercostal and suprasternal retractions

ANS: D This patient's history of septicemia and labored breathing suggest the onset of ARDS, which will require rapid interventions such as administration of O2 and use of positive-pressure ventilation. The other patients should also be assessed, but their assessment data are typical of their disease processes and do not suggest deterioration in their status. DIF: Cognitive Level: Analyze (analysis)

The nurse is talking to the parents of a child with pediculosis capitis. Which should the nurse include when explaining how to manage pediculosis capitis? a. "You will need to cut the hair shorter if infestation and nits are severe." b. "You can distinguish viable from nonviable nits, and remove all viable ones." c. "You can wash all nits out of hair with a regular shampoo." d. "You will need to remove nits with an extra-fine-tooth comb or tweezers."

ANS: D Treatment consists of the application of pediculicide and manual removal of nit cases. An extra-fine-tooth comb facilitates manual removal. Parents should be cautioned against cutting the child's hair short; lice infest short hair as well as long. It increases the child's distress and serves as a continual reminder to peers who are prone to tease children with a different appearance. It is not possible to differentiate between viable and nonviable eggs. Regular shampoo is not effective; a pediculicide is necessary.

Which may be given to high-risk children after exposure to chickenpox to prevent varicella? a. Acyclovir (Zovirax) b. Varicella globulin c. Diphenhydramine hydrochloride (Benadryl) d. VCZ immune globulin (VariZIG)

ANS: D VariZIG is given to high-risk children to prevent the development of chickenpox. Acyclovir decreases the severity, not the development, of chickenpox. Varicella globulin is not effective because it is not the immune globulin. Diphenhydramine may help pruritus but not the actual chickenpox.

When caring for the patient with a pulmonary artery (PA) pressure catheter, the nurse observes that the PA waveform indicates that the catheter is in the wedged position. Which action should the nurse take next? a. Zero balance the transducer. b. Activate the fast flush system. c. Notify the health care provider. d. Deflate and reinflate the PA balloon.

ANS: D When the catheter is in the wedge position, blood flow past the catheter is obstructed, placing the patient at risk for pulmonary infarction. A health care provider or advanced practice nurse should be called to reposition the catheter. The other actions will not correct the wedging of the PA catheter.

Which are clinical manifestations of postmaturity in the newborn? (Select all that apply.) a. Excessive lanugo b. Increased subcutaneous fat c. Absence of scalp hair d. Parchment-like skin e. Minimal vernix caseosa f. Long fingernails

ANS: D, E, F In postmature newborns, the skin is often cracked, parchment-like, and desquamating; there is little to no vernix caseosa; and fingernails are long. Lanugo is usually absent in postmature newborns. Subcutaneous fat is usually depleted, giving the child a thin, elongated appearance. Scalp hair is usually abundant.

The nurse is preparing to care for a newborn receiving phototherapy. Which interventions are appropriate? (Select all that apply.) a. Avoid stimulation. b. Decrease fluid intake. c. Expose all the newborn's skin. d. Monitor skin temperature closely. e. Reposition the newborn every 2 hours. f. Cover the newborn's eyes with eye shields or patches.

ANS: D, E, F Several nursing interventions are instituted to protect the newborn during phototherapy. Temperature is closely monitored to prevent hyperthermia or hypothermia. The newborn is repositioned every 2 hours to maximize exposure to the phototherapy and to prevent skin breakdown. The infant's eyes are shielded by an opaque mask to prevent exposure to the light. The newborn is clothed in a diaper because a side effect of phototherapy includes loose, greenish stools. Other side effects include increased metabolic rate; dehydration; electrolyte disturbances, such as hypocalcemia; and priapism. Infants receiving phototherapy may require additional fluid volume to compensate for insensible and intestinal fluid loss. The infant should receive adequate stimulation, which includes feeding and touching.

A patient in the clinic with cystic fibrosis (CF) reports increased sweating and weakness during the summer months. Which action by the nurse would be most appropriate? a. Teach the patient signs of hypoglycemia. b. Have the patient add dietary salt to meals. c. Suggest decreasing intake of dietary fat and calories. d. Instruct the patient about pancreatic enzyme replacements.

B Added dietary salt is indicated whenever sweating is excessive, such as during hot weather, when fever is present, or from intense physical activity. The management of pancreatic insufficiency includes pancreatic enzyme replacement of lipase, protease, and amylase (e.g., Pancreaze, Creon, Ultresa, Zenpep) administered before each meal and snack. This patient is at risk for hyponatremia based on reported symptoms. Adequate intake of fat, calories, protein, and vitamins is important. Fat-soluble vitamins (vitamins A, D, E, and K) must be supplemented because they are malabsorbed. Use of caloric supplements improves nutritional status. Hyperglycemia caused by pancreatic insufficiency is more likely to occur than hypoglycemia.

The nurse provides dietary teaching for a patient with chronic obstructive pulmonary disease (COPD) who has a low body mass index (BMI). Which patient statement indicates that the teaching has been effective? a. "I will drink lots of fluids with my meals." b. "I can have ice cream as a snack every day." c. "I will exercise for 15 minutes before meals." d. "I will decrease my intake of meat and poultry."

B High-calorie foods such as ice cream are an appropriate snack for patients with COPD. Fluid intake of 3 L/day is recommended, but fluids should be taken between meals rather than with meals to improve oral intake of solid foods. The patient should avoid exercise for an hour before meals to prevent fatigue while eating. Meat and dairy products are high in protein and are good choices for the patient with COPD.

The emergency department nurse is evaluating the effectiveness of therapy for a patient who has received treatment during an asthma attack. Which assessment finding is the best indicator that the therapy has been effective? a. No wheezes are audible. b. O2saturation is >90%. c. Accessory muscle use has decreased. d. Respiratory rate is 16 breaths/minute.

B The goal for treatment of an asthma attack is to keep the O2 saturation above 90%. The other patient data may occur when the patient is too fatigued to continue with the increased work of breathing required in an asthma attack.

A young adult female patient with cystic fibrosis (CF) tells the nurse that she is considering getting married and wondering about having children. Which initial response by the nurse is best? a. "Are you aware of the normal lifespan for patients with CF?" b. "Would like more information to help you with that decision?" c. "Many women with CF do not have difficulty conceiving children." d. "You will need to have genetic counseling before making a decision."

B The nurse's initial response should be to assess the patient's knowledge level and need for information. Although the lifespan for patients with CF is likely to be shorter than normal, it would not be appropriate for the nurse to address this as the initial response to the patient's comments. The other responses have accurate information, but the nurse should first assess the patient's understanding about the issues surrounding pregnancy.

The nurse teaches a patient with chronic bronchitis about a new prescription for Advair Diskus (combined fluticasone and salmeterol). Which action by the patient would indicate to the nurse that teaching about medication administration has been successful? a. The patient shakes the device before use. b. The patient rapidly inhales the medication. c. The patient attaches a spacer to the Diskus. d. The patient performs huff coughing after inhalation.

B The patient should inhale the medication rapidly. Otherwise the dry particles will stick to the tongue and oral mucosa and not get inhaled into the lungs. Advair Diskus is a dry powder inhaler; shaking is not recommended. Spacers are not used with dry powder inhalers. Huff coughing is a technique to move mucus into larger airways to expectorate. The patient should not huff cough or exhale forcefully after taking Advair in order to keep the medication in the lungs.

The nurse teaches a patient about pursed-lip breathing. Which action by the patient would indicate to the nurse that further teaching is needed? a. The patient inhales slowly through the nose. b. The patient puffs up the cheeks while exhaling. c. The patient practices by blowing through a straw. d. The patient's ratio of inhalation to exhalation is 1:3.

B The patient should relax the facial muscles without puffing the cheeks while doing pursed-lip breathing. The other actions by the patient indicate a good understanding of pursed-lip breathing.

The clinic nurse makes a follow-up telephone call to a patient with asthma. The patient reports having a baseline peak flow reading of 600 L/min, and the current peak flow is 420 L/min. Which action should the nurse take first? a. Tell the patient to go to the hospital emergency department. b. Instruct the patient to use the prescribed albuterol (Ventolin HFA). c. Ask about recent exposure to any new allergens or asthma triggers. d. Question the patient about use of the prescribed inhaled corticosteroids.

B The patient's peak flow is 70% of normal, indicating a need for immediate use of short-acting b2 -adrenergic SABA medications. Assessing for correct use of medications or exposure to allergens is also appropriate, but would not address the current decrease in peak flow. Because the patient is currently in the yellow zone, hospitalization is not needed.

A patient with severe chronic obstructive pulmonary disease (COPD) tells the nurse, "I wish I were dead! I'm just a burden on everybody." Based on this information, which nursing diagnosis is most appropriate? a. Complicated grieving related to expectation of death b. Chronic low self-esteem related to physical dependence c. Ineffective coping related to unknown outcome of illness d. Deficient knowledge related to lack of education about COPD

B The patient's statement about not being able to do anything for himself or herself supports this diagnosis. Although deficient knowledge, complicated grieving, and ineffective coping may also be appropriate diagnoses for patients with COPD, the data for this patient do not support these diagnoses.

Postural drainage with percussion and vibration is ordered twice daily for a patient with chronic bronchitis. Which intervention should the nurse include in the plan of care? a. Schedule the procedure 1 hour after the patient eats. b. Maintain the patient in the lateral position for 20 minutes. c. Give the prescribed albuterol (Ventolin HFA) before the therapy. d. Perform percussion before assisting the patient to the drainage position.

C Bronchodilators are administered before chest physiotherapy. Postural drainage, percussion, and vibration should be done 1 hour before or 3 hours after meals. Patients remain in each postural drainage position for 5 minutes. Percussion is done while the patient is in the postural drainage position.

It is generally recommended that a child with acute streptococcal pharyngitis can return to school: a. when sore throat is better. b. if no complications develop. c. after taking antibiotics for 24 hours. d. after taking antibiotics for 3 days.

C

The nurse interviews a patient with a new diagnosis of chronic obstructive pulmonary disease (COPD). Which information is most specific in confirming a diagnosis of chronic bronchitis? a. The patient tells the nurse about a family history of bronchitis. b. The patient indicates a 30 pack-year cigarette smoking history. c. The patient reports a productive cough for 3 months every winter. d. The patient denies having respiratory problems until the past 12 months.

C A diagnosis of chronic bronchitis is based on a history of having a productive cough for 3 months for at least 2 consecutive years. There is no family tendency for chronic bronchitis. Although smoking is the major risk factor for chronic bronchitis, a smoking history does not confirm the diagnosis.

The nurse teaches a patient how to administer formoterol (Perforomist) through a nebulizer. Which action by the patient indicates good understanding of the teaching? a. The patient attaches a spacer before using the inhaler. b. The patient coughs vigorously after using the inhaler. c. The patient removes the facial mask when misting stops. d. The patient activates the inhaler at the onset of expiration.

C A nebulizer is used to administer aerosolized medication. A mist is seen when the medication is aerosolized, and when all of the medication has been used, the misting stops. The other options refer to inhaler use. Coughing vigorously after inhaling and activating the inhaler at the onset of expiration are both incorrect techniques when using an inhaler.

It is generally recommended that a child with acute streptococcal pharyngitis can return to school: a. when sore throat is better. b. if no complications develop. c. after taking antibiotics for 24 hours. d. after taking antibiotics for 3 days

C After children have taken antibiotics for 24 hours, they are no longer contagious to other children. Sore throat may persist longer than 24 hours after beginning antibiotic therapy, but the child is no longer considered contagious. Complications may take days to weeks to develop.

A child is diagnosed with influenza, probably type A disease. Management includes which recommendation? a. Clear liquid diet for hydration b. Aspirin to control fever c. Amantadine hydrochloride (Symmetrel) to reduce symptoms d. Antibiotics to prevent bacterial infection

C Amantadine hydrochloride may reduce symptoms related to influenza A if administered within 24 to 48 hours of onset. It is ineffective against type B or C. A clear liquid diet is not necessary for influenza, but maintaining hydration is important. Aspirin is not recommended in children because of increased risk of Reye syndrome. Acetaminophen or ibuprofen is a better choice. Preventive antibiotics are not indicated for influenza unless there is evidence of a secondary bacterial infection.

A patient is scheduled for spirometry. Which action should the nurse take to prepare the patient for this procedure? a. Give the rescue medication immediately before testing. b. Administer oral corticosteroids 2 hours before the procedure. c. Withhold bronchodilators for 6 to 12 hours before the examination. d. Ensure that the patient has been NPO for several hours before the test.

C Bronchodilators are held before spirometry so that a baseline assessment of airway function can be determined. Testing is repeated after bronchodilator use to determine whether the decrease in lung function is reversible. There is no need for the patient to be NPO. Oral corticosteroids should be held before spirometry. Rescue medications (which are bronchodilators) would not be given until after the baseline pulmonary function was assessed.

The nurse is caring for a patient with chronic obstructive pulmonary disease (COPD). Which information obtained from the patient would prompt the nurse to consult with the health care provider before administering the prescribed theophylline? a. The patient reports a recent 15-lb weight gain. b. The patient denies shortness of breath at present. c. The patient takes cimetidine (Tagamet HB) daily. d. The patient complains of coughing up green mucus.

C Cimetidine interferes with the metabolism of theophylline, and concomitant administration may lead rapidly to theophylline toxicity. The other patient information would not affect whether the theophylline should be administered or not.

The nurse is caring for a patient with cor pulmonale. The nurse should monitor the patient for which expected finding? a. Chest pain c. Peripheral edema b. Finger clubbing d. Elevated temperature

C Cor pulmonale causes clinical manifestations of right ventricular failure, such as peripheral edema. The other clinical manifestations may occur in the patient with other complications of chronic obstructive pulmonary disease but are not indicators of cor pulmonale.

A patient with chronic obstructive pulmonary disease (COPD) has a nursing diagnosis of imbalanced nutrition: less than body requirements. Which intervention would be most appropriate for the nurse to include in the plan of care? a. Encourage increased intake of whole grains. b. Increase the patient's intake of fruits and fruit juices. c. Offer high-calorie protein snacks between meals and at bedtime. d. Assist the patient in choosing foods with high vegetable content.

C Eating small amounts more frequently (as occurs with snacking) will increase caloric intake by decreasing the fatigue and feelings of fullness associated with large meals. Patients with COPD should rest before meals. Foods that have a lot of texture such as whole grains may take more energy to eat and get absorbed and lead to decreased intake. Although fruits, juices, and minerals are not contraindicated, foods high in protein are a better choice.

A patient newly diagnosed with asthma is being discharged. The nurse anticipates including which topic in the discharge teaching? a. Use of long-acting b-adrenergic medications b. Side effects of sustained-release theophylline c. Self-administration of inhaled corticosteroids d. Complications associated with O2 therapy

C Inhaled corticosteroids are more effective in improving asthma than any other drug and are indicated for all patients with persistent asthma. The other therapies would not typically be first-line treatments for newly diagnosed asthma.

A 55-yr-old patient with increasing dyspnea is being evaluated for a possible diagnosis of chronic obstructive pulmonary disease (COPD). When teaching a patient about pulmonary spirometry for this condition, what is the most important question the nurse should ask? a. "Are you claustrophobic?" b. "Are you allergic to shellfish?" c. "Have you taken any bronchodilators today?" d. "Do you have any metal implants or prostheses?"

C Spirometry will help establish the COPD diagnosis. Bronchodilators should be avoided at least 6 hours before the test. Spirometry does not involve being placed in an enclosed area such as for magnetic resonance imaging (MRI). Contrast dye is not used for spirometry. The patient may still have spirometry done if metal implants or prostheses are present because they are contraindications for an MRI.

A patient seen in the asthma clinic has recorded daily peak flow rates that are 75% of the baseline. Which action will the nurse plan to take next? a. Increase the dose of the leukotriene inhibitor. b. Teach the patient about the use of oral corticosteroids. c. Administer a bronchodilator and recheck the peak flow. d. Instruct the patient to keep the scheduled follow-up appointment.

C The patient's peak flow reading indicates that the condition is worsening (yellow zone). The patient should take the bronchodilator and recheck the peak flow. Depending on whether the patient returns to the green zone, indicating well-controlled symptoms, the patient may be prescribed oral corticosteroids or a change in dosing of other medications. Keeping the next appointment is appropriate, but the patient also needs to be taught how to control symptoms now and use the bronchodilator.

A patient hospitalized with chronic obstructive pulmonary disease (COPD) is being discharged home on O2 therapy. Which instruction should the nurse include in the discharge teaching? a. Travel is not possible with the use of O2 devices. b. O2 flow should be increased if the patient has more dyspnea. c. O2 use can improve the patient's prognosis and quality of life. d. Storage of O2 requires large metals tanks that each last 4 to 6 hours.

C The use of home O2 improves quality of life and prognosis. Because increased dyspnea may be a symptom of an acute process such as pneumonia, the patient should notify the health care provider rather than increasing the O2 flow rate if dyspnea becomes worse. O2 can be supplied using liquid, storage tanks, or concentrators, depending on individual patient circumstances. Travel is possible using portable O2 concentrators.

Which instruction should the nurse include in an exercise teaching plan for a patient with chronic obstructive pulmonary disease (COPD)? a. "Avoid upper body exercise to prevent dyspnea." b. "Stop exercising if you start to feel short of breath." c. "Use the bronchodilator before you start to exercise." d. "Breathe in and out through the mouth while you exercise."

C Use of a bronchodilator before exercise improves airflow for some patients and is recommended. Shortness of breath is normal with exercise and not a reason to stop. Patients should be taught to breathe in through the nose and out through the mouth (using a pursed-lip technique). Upper-body exercise can improve the mechanics of breathing in patients with COPD.

The nurse assesses a patient with a history of asthma. Which assessment finding indicates that the nurse should take immediate action? a. Pulse oximetry reading of 91% b. Respiratory rate of 26 breaths/min c. Use of accessory muscles in breathing d. Peak expiratory flow rate of 240 L/min

C Use of accessory muscle indicates that the patient is experiencing respiratory distress, and rapid intervention is needed. The other data indicate the need for ongoing monitoring and assessment but do not suggest that immediate treatment is required.

The nurse takes an admission history on a patient with possible asthma who has new- onset wheezing and shortness of breath. Which information may indicate a need for a change in therapy? a. The patient has chronic inflammatory bowel disease. b. The patient has a history of pneumonia 6 months ago. c. The patient takes propranolol (Inderal) for hypertension. d. The patient uses acetaminophen (Tylenol) for headaches.

C b-Blockers such as propranolol can cause bronchospasm in some patients with asthma. The other information will be documented in the health history but does not indicate a need for a change in therapy.

The nurse teaches a patient who has asthma about peak flow meter use. Which action by the patient indicates that teaching was successful? a. The patient inhales rapidly through the peak flow meter mouthpiece. b. The patient takes montelukast (Singulair) for peak flows in the red zone. c. The patient calls the health care provider when the peak flow is in the green zone. d. The patient uses an albuterol (Ventolin HFA) inhaler for peak flows in the yellow zone.

D Readings in the yellow zone indicate a decrease in peak flow. The patient should use short-acting b2-adrenergic (SABA) medications. Readings in the green zone indicate good asthma control. The patient should exhale quickly and forcefully through the peak flow meter mouthpiece to obtain the readings. Readings in the red zone do not indicate good peak flow, and the patient should take a fast-acting bronchodilator and call the health care provider for further instructions. Singulair is not indicated for acute attacks but rather is used for maintenance therapy.

Cystic fibrosis (CF) is suspected in a toddler. Which test is essential in establishing this diagnosis? a. Bronchoscopy b. Serum calcium c. Urine creatinine d. Sweat chloride test

D A sweat chloride test result greater than 60 mEq/L is diagnostic of CF. Bronchoscopy, although helpful for identifying bacterial infection in children with CF, is not diagnostic. Serum calcium is normal in children with CF. Urine creatinine is not diagnostic of CF

The nurse develops a teaching plan to help increase activity tolerance at home for an older adult with severe chronic obstructive pulmonary disease (COPD). Which instructions would be appropriate for the nurse to include in the plan of care? a. Stop exercising when you feel short of breath. b. Walk until pulse rate exceeds 130 beats/minute. c. Limit exercise to activities of daily living (ADLs). d. Walk 15 to 20 minutes a day at least 3 times/week.

D Encourage the patient to walk 15 to 20 minutes a day at least three times a week with gradual increases. Shortness of breath is normal with exercise and not an indication that the patient should stop. Limiting exercise to ADLs will not improve the patient's exercise tolerance. A 70-yr-old patient should have a pulse rate of 120 beats/min or less with exercise (80% of the maximal heart rate of 150 beats/min ).

The nurse completes an admission assessment on a patient with asthma. Which information given by patient is indicates a need for a change in therapy? a. The patient uses albuterol (Ventolin HFA) before aerobic exercise. b. The patient says that the asthma symptoms are worse every spring. c. The patient's heart rate increases after using the albuterol (Ventolin HFA) inhaler. d. The patient's only medications are albuterol (Ventolin HFAl) and salmeterol (Serevent).

D Long-acting b2-agonists should be used only in patients who also are using an inhaled corticosteroid for long-term control. Salmeterol should not be used as the first-line therapy for long-term control. Using a bronchodilator before exercise is appropriate. The other information given by the patient requires further assessment by the nurse but is not unusual for a patient with asthma.

The nurse is caring for a 10-month-old infant with respiratory syncytial virus (RSV) bronchiolitis. Which intervention should be included in the child's care? (Select all that apply.) a. Place in a mist tent. b. Administer antibiotics. c. Administer cough syrup. d. Encourage to drink 8 ounces of formula every 4 hours. e. Cluster care to encourage adequate rest. f. Place on noninvasive oxygen monitoring

D, E, F Hydration is important in children with RSV bronchiolitis to loosen secretions and prevent shock. Clustering of care promotes periods of rest. The use of noninvasive oxygen monitoring is recommended. Mist tents are no longer used. Antibiotics do not treat illnesses with viral causes. Cough syrup suppresses clearing of respiratory secretions and is not indicated for young children.

The nurse is caring for a 10-month-old infant with respiratory syncytial virus (RSV) bronchiolitis. Which intervention should be included in the child's care? (Select all that apply.) a. Place in a mist tent. b. Administer antibiotics. c. Administer cough syrup. d. Encourage to drink 8 ounces of formula every 4 hours. e. Cluster care to encourage adequate rest. f. Place on noninvasive oxygen monitoring.

D,E,F

The nurse receives change-of-shift report on the following four patients. Which patient should the nurse assess first? a. A 23-yr-old patient with cystic fibrosis who has pulmonary function testing scheduled b. A 46-yr-old patient on bed rest who is complaining of sudden onset of shortness of breath c. A 77-yr-old patient with tuberculosis (TB) who has four medications due in 15 minutes d. A 35-yr-old patient who was admitted with pneumonia and has a temperature of 100.2° F (37.8° C)

NS: B Patients on bed rest who are immobile are at high risk for deep vein thrombosis (DVT). Sudden onset of shortness of breath in a patient with a DVT suggests a pulmonary embolism and requires immediate assessment and action such as O2 administration. The other patients should also be assessed as soon as possible, but there is no indication that they may need immediate action to prevent clinical deterioration. DIF: Cognitive Level: Analyze (analysis) REF: 529 OBJ: Special Questions: Prioritization | Special Questions: Multiple Patients TOP: Nursing Process: Planning MSC: NCLEX: Safe and Effective Care Environment

A patient with a tracheostomy has a new order for a fenestrated tracheostomy tube. Which action should the nurse include in the plan of care in collaboration with the speech therapist? a. Leave the tracheostomy inner cannula inserted at all times. b. Place the decannulation cap in the tube before cuff deflation. c. Assess the ability to swallow before using the fenestrated tube. d. Inflate the tracheostomy cuff during use of the fenestrated tube.

c. Assess the ability to swallow before using the fenestrated tube. ANS: C Because the cuff is deflated when using a fenestrated tube, the patient's risk for aspiration should be assessed before changing to a fenestrated tracheostomy tube. The decannulation cap is never inserted before cuff deflation because to do so would obstruct the patient's airway. The cuff is deflated and the inner cannula removed to allow air to flow across the patient's vocal cords when using a fenestrated tube. DIF: Cognitive Level: Apply (application) REF: 485 TOP: Nursing Process: Planning MSC: NCLEX: Physiological Integrity

A child is being admitted to the hospital to be tested for cystic fibrosis (CF). Which tests should the nurse expect? a. Sweat chloride test, stool for fat, chest radiograph films b. Stool test for fat, gastric contents for hydrochloride, chest radiograph films c. Sweat chloride test, bronchoscopy, duodenal fluid analysis d. Sweat chloride test, stool for trypsin, biopsy of intestinal mucosa

A A sweat test result of greater than 60 mEq/L is diagnostic of CF, a high level of fecal fat is a gastrointestinal (GI) manifestation of CF, and a chest radiograph showing patchy atelectasis and obstructive emphysema indicates CF. Gastric contents contain hydrochloride normally; it is not diagnostic. Bronchoscopy and duodenal fluid are not diagnostic. Stool test for trypsin and intestinal biopsy are not helpful in diagnosing CF.

An 18-month-old child is seen in the clinic with AOM. Trimethoprim-sulfamethoxazole (Bactrim) is prescribed. Which statement made by the parent indicates a correct understanding of the instructions? a. "I should administer all the prescribed medication." b. "I should continue medication until the symptoms subside." c. "I will immediately stop giving medication if I notice a change in hearing." d. "I will stop giving medication if fever is still present in 24 hours."

A Antibiotics should be given for their full course to prevent recurrence of infection with resistant bacteria. Symptoms may subside before the full course is given. Hearing loss is a complication of AOM. Antibiotics should continue to be given. Medication may take 24 to 48 hours to make symptoms subside. It should be continued

A child is admitted to the hospital with asthma. Which assessment findings support this diagnosis? a. Nonproductive cough, wheezing b. Fever, general malaise c. Productive cough, rales d. Stridor, substernal retractions

A Asthma presents with a nonproductive cough and wheezing. Pneumonia appears with an acute onset, fever, and general malaise. A productive cough and rales would be indicative of pneumonia. Stridor and substernal retractions are indicative of croup

A school-age child had an upper respiratory tract infection for several days and then began having a persistent dry, hacking cough that was worse at night. The cough has become productive in the past 24 hours. This is most suggestive of which diagnosis? a. Bronchitis b. Bronchiolitis c. Viral-induced asthma d. Acute spasmodic laryngitis

A Bronchitis is characterized by these symptoms and occurs in children older than 6 years. Bronchiolitis is rare in children older than 2 years. Asthma is a chronic inflammation of the airways that may be exacerbated by a virus. Acute spasmodic laryngitis occurs in children between 3 months and 3 years.

A child with cystic fibrosis (CF) receives aerosolized bronchodilator medication. When should this medication be administered? a. Before chest physiotherapy (CPT) b. After CPT c. Before receiving 100% oxygen d. After receiving 100% oxygen

A Bronchodilators should be given before CPT to open bronchi and make expectoration easier. Aerosolized bronchodilator medications are not helpful when used after CPT. Oxygen administration is necessary only in acute episodes with caution because of chronic carbon dioxide retention.

Parents of a child with cystic fibrosis ask the nurse about genetic implications of the disorder. Which statement, made by the nurse, expresses accurately the genetic implications? a. If it is present in a child, both parents are carriers of this defective gene. b. It is inherited as an autosomal dominant trait. c. It is a genetic defect found primarily in non-Caucasian population groups. d. There is a 50% chance that siblings of an affected child also will be affected

A CF is an autosomal recessive gene inherited from both parents and is inherited as an autosomal recessive, not autosomal dominant, trait. CF is found primarily in Caucasian populations. An autosomal recessive inheritance pattern means that there is a 25% chance a sibling will be infected but a 50% chance a sibling will be a carrier.

In providing nourishment for a child with cystic fibrosis (CF), which factor should the nurse keep in mind? a. Diet should be high in carbohydrates and protein. b. Diet should be high in easily digested carbohydrates and fats. c. Most fruits and vegetables are not well tolerated. d. Fats and proteins must be greatly curtailed

A Children with CF require a well-balanced, high-protein, high-calorie diet because of impaired intestinal absorption. Enzyme supplementation helps digest foods; other modifications are not necessary. A well-balanced diet containing fruits and vegetables is important. Fats and proteins are a necessary part of a well-balanced diet.

A nurse is interpreting the results of a tuberculin skin test (TST) on an adolescent who is HIV positive. Which induration size indicates a positive result for this child 48-72 hours after the test? a. 5 mm b. 10 mm c. 15 mm d. 20 mm

A Clinical evidence of a positive TST in children receiving immunosuppressive therapy, including immunosuppressive doses of steroids or who have immunosuppressive conditions, including HIV infection is an induration of 5 mm. Children younger than 4 years of age with: (a) other medical risk conditions, including Hodgkin disease, lymphoma, diabetes mellitus, chronic renal failure, or malnutrition; (b) born or whose parents were born in high-prevalence (TB) regions of the world; (c) frequently exposed to adults who are HIV infected, homeless, users of illicit drugs, residents of nursing homes, incarcerated or institutionalized, or migrant farm workers; and (d) who travel to high-prevalence (TB) regions of the world are positive when the induration is 10 mm. Children 4 years of age or older without any risk factors are positive when the induration is 20 mm.

Which explains why cool-mist vaporizers rather than steam vaporizers are recommended in home treatment of childhood respiratory tract infections? a. They are safer. b. They are less expensive. c. Respiratory secretions are dried. d. A more comfortable environment is produced.

A Cool-mist vaporizers are safer than steam vaporizers, and little evidence exists to show any advantages to steam. The cost of cool-mist and steam vaporizers is comparable. Steam loosens secretions, not dries them. Both may promote a more comfortable environment, but cool-mist vaporizers present decreased risk for burns and growth of organisms.

A nurse is charting that a hospitalized child has labored breathing. Which describes labored breathing? a. Dyspnea b. Tachypnea c. Hypopnea d. Orthopnea

A Dyspnea is labored breathing. Tachypnea is rapid breathing. Hypopnea is breathing that is too shallow. Orthopnea is difficulty breathing except in upright position.

An infant's parents ask the nurse about preventing OM. Which should be recommended? a. Avoid tobacco smoke. b. Use nasal decongestant. c. Avoid children with OM. d. Bottle-feed or breastfeed in supine position.

A Eliminating tobacco smoke from the child's environment is essential for preventing OM and other common childhood illnesses. Nasal decongestants are not useful in preventing OM. Children with uncomplicated OM are not contagious unless they show other upper respiratory tract infection (URI) symptoms. Children should be fed in an upright position to prevent OM.

A school-age child has been admitted with an acute asthma episode. The child is receiving oxygen by nasal prongs at 2 liters. How often should the nurse plan to monitor the child's pulse oximetry status? a. Continuous b. Every 30 minutes c. Every hour d. Every 2 hours

A The child on supplemental oxygen requires intermittent or continuous oxygenation monitoring, depending on severity of respiratory compromise and initial oxygenation status. The child in status asthmaticus should be placed on continuous cardiorespiratory (including blood pressure) and pulse oximetry monitoring.

A nurse is teaching nursing students about clinical manifestations of cystic fibrosis (CF). Which is/are the earliest recognizable clinical manifestation(s) of CF? a. Meconium ileus b. History of poor intestinal absorption c. Foul-smelling, frothy, greasy stools d. Recurrent pneumonia and lung infections

A The earliest clinical manifestation of CF is a meconium ileus, which is found in about 10% of children with CF. Clinical manifestations include abdominal distention, vomiting, failure to pass stools, and rapid development of dehydration. History of malabsorption is a later sign that manifests as failure to thrive. Foul-smelling stools are a later manifestation of CF. Recurrent respiratory tract infections are a later sign of CF.

Decongestant nose drops are recommended for a 10-month-old infant with an upper respiratory tract infection. Instructions for nose drops should include which action? a. Avoid using for more than 3 days. b. Keep drops to use again for nasal congestion. c. Administer drops until nasal congestion subsides. d. Administer drops after feedings and at bedtime.

A Vasoconstrictive nose drops such as phenylephrine (Neo-Synephrine) should not be used for more than 3 days to avoid rebound congestion. Drops should be discarded after one illness because they may become contaminated with bacteria. Vasoconstrictive nose drops can have a rebound effect after 3 days of use. Drops administered before feedings are more helpful.

An infant has developed staphylococcal pneumonia. Nursing care of the child with pneumonia includes which interventions? (Select all that apply.) a. Cluster care to conserve energy b. Round-the-clock administration of antitussive agents c. Strict intake and output to avoid congestive heart failure d. Administration of antibiotics

A, D Antibiotics are indicated for a bacterial pneumonia. Often the child will have decreased pulmonary reserve, and the clustering of care is essential. Antitussive agents are used sparingly. It is desirable for the child to cough up some of the secretions. Fluids are essential to kept secretions as liquefied as possible.

Which is defined as a vaguely outlined area of edematous tissue situated over the portion of the scalp that presents in a vertex delivery? a. Caput succedaneum b. Hydrocephalus c. Cephalhematoma d. Subdural hematoma

ANS: A A vaguely outlined area of edematous tissue situated over the portion of the scalp that presents in a vertex delivery is the definition of a caput succedaneum. The swelling consists of serum and/or blood accumulated in the tissues above the bone, and it may extend beyond the bone margin. Hydrocephalus is caused by an imbalance in production and absorption of cerebrospinal fluid. When production exceeds absorption, fluid accumulates within the ventricular system, causing dilation of the ventricles. A cephalhematoma has sharply demarcated boundaries that do not extend beyond the limits of the (bone) suture line. A subdural hematoma is located between the dura and the cerebrum. It would not be visible on the scalp.

The nurse is caring for a 3-week-old preterm newborn born at 29 weeks of gestation. While taking vital signs and changing the newborn's diaper, the nurse observes the newborn's color is pink but slightly mottled, arms and legs are limp and extended, hiccups are present, and heart rate is regular and rapid. The nurse should recognize these behaviors as manifestations of: a. stress. b. subtle seizures. c. preterm behavior. d. onset of respiratory distress.

ANS: A Color pink but slightly mottled, arms and legs limp and extended, hiccups, respiratory pauses and gasping, and an irregular, rapid heart rate are signs of stress or fatigue in a newborn. Neonatal seizures usually have some type of repetitive movement from twitching to rhythmic jerking movements. The behavior of a preterm newborn may be inactive and listless. Respiratory distress is exhibited by retractions and nasal flaring.

The nurse is planning care for a newborn receiving IV calcium gluconate for treatment of hypocalcemia. Which intervention is the most appropriate during the acute phase? a. Allow newborn to sleep with pacifier to decrease stimuli. b. Keep newborn awake to monitor central nervous system changes. c. Encourage parents to hold and feed newborn to facilitate attachment during illness. d. Awaken newborn periodically to assess level of consciousness.

ANS: A For newborns with hypocalcemia, the nurse should manipulate the environment to reduce stimuli that might precipitate a seizure or tremors. A quiet, nonstimulating environment should be maintained for the newborn until calcium levels are normalized. Care should be provided without sudden jarring. Parents can be involved in observations and care when the child is awake.

Hemolytic disease is suspected in a mother's second newborn. Which factor is important in understanding how this could develop? a. The mother's first child was Rh positive. b. The mother is Rh positive. c. Both parents have type O blood. d. RhIG (RhoGAM) was given to the mother during her first pregnancy.

ANS: A Hemolytic disease of the newborn results from an abnormally rapid rate of red blood cell (RBC) destruction. The major causes of this are Rh and maternal-fetal ABO incompatibility. If an Rh-negative mother has previously been exposed to Rh-positive blood through pregnancy or blood transfusion, antibodies to this blood group antigen may develop so that she is isoimmunized. With further exposure to Rh, the maternal antibodies will agglutinate with the red cells of the fetus who has the antigen and destroy the cells. Hemolytic disease is also caused by ABO incompatibilities. Blood type is the important consideration. If both parents are type O blood, ABO incompatibility would not be a possibility. The mother should have received Rho(D) immune globulin to prevent antibody development after the first pregnancy.

A premature newborn requires oxygen and mechanical ventilation. Which complications should the nurse assess for? a. Bronchopulmonary dysplasia, pneumothorax b. Anemia, necrotizing enterocolitis c. Cerebral palsy, persistent patent ductus d. Congestive heart failure, cerebral edema

ANS: A Oxygen therapy, although lifesaving, is not without hazards. The positive pressure created by mechanical ventilation creates an increase in the number of ruptured alveoli and subsequent pneumothorax and bronchopulmonary dysplasia. Anemia, necrotizing enterocolitis, cerebral palsy, persistent patent ductus, congestive heart failure, and cerebral edema are complications not primarily due to oxygen therapy and mechanical ventilation.

A preterm newborn, after spending 8 weeks in the NICU, is being discharged. The parents of the newborn express apprehension and worry that the newborn may still be in danger. The nurse should recognize that this is: a. normal. b. a reason to postpone discharge. c. suggestive of maladaptation. d. suggestive of inadequate bonding.

ANS: A Parents become apprehensive and excited as the time for discharge approaches. They have many concerns and insecurities regarding the care of their newborn. A major concern is that they may be unable to recognize signs of illness or distress in their newborn. Preparation for discharge should begin early and include helping the parent acquire the skills necessary for care. Apprehension and worry are normal adaptive responses. The NICU nurses should facilitate discharge by involving parents in care as soon as possible.

The nurse is caring for a preterm newborn who requires mechanical ventilation for the treatment of respiratory distress syndrome. The nurse should recognize that, because of the mechanical ventilation, there is an increased risk of: a. alveolar rupture. b. meconium aspiration. c. transient tachypnea. d. retractions and nasal flaring.

ANS: A Positive pressure introduced by mechanical apparatus has created an increase in the incidence of ruptured alveoli and subsequent pneumothorax and bronchopulmonary dysplasia. Meconium aspiration is not associated with mechanical ventilation. Tachypnea may be an indication of a pneumothorax, but it would not be transient. Retractions and nasal flaring are indications of the use of accessory muscles when the newborn cannot obtain sufficient oxygen. The use of mechanical ventilation bypasses the newborn's need to use these muscles.

To prevent Rh isoimmunization, RhIG (RhoGAM) is administered to all: a. Rh-negative women who deliver an Rh-positive newborn. b. Rh-positive women who deliver an Rh-negative newborn. c. Rh-negative newborns whose mothers are Rh positive. d. Rh-positive fathers before conception of second newborn when first newborn was Rh positive.

ANS: A Rh IG human gamma globulin concentrate of anti-D is administered to all unsensitized Rh-negative women after delivery or abortion of an Rh-positive newborn or fetus. Administering RhIG to an individual who is Rh positive will result in agglutination of red cells and hemolysis. It will not alter the person's genetic makeup. The anti-D antibody contained in RhIG will have no effect on Rh-negative newborns because the D antibody is not present.

The nurse is caring for a newborn with respiratory distress syndrome. The newborn has an endotracheal tube. Which statement describes nursing considerations related to suctioning? a. Suctioning should not be carried out routinely. b. Newborn should be in Trendelenburg position for suctioning. c. Routine suctioning, usually every 15 minutes, is necessary. d. Frequent suctioning is necessary to maintain patency of bronchi.

ANS: A Suctioning is not an innocuous procedure and can cause bronchospasm, bradycardia, hypoxia, and increased ICP. It should never be carried out routinely. The Trendelenburg position should be avoided. This position can contribute to increased ICP and reduced lung capacity from gravity pushing organs against diaphragm. Routine suctioning is avoided because of the potential complications of bronchospasm, bradycardia, hypoxia, and increased ICP.

Which is the most appropriate nursing action when intermittently gavage-feeding a preterm newborn? a. Allow formula to flow by gravity. b. Insert tube through nares rather than mouth. c. Avoid letting newborn suck on tube. d. Apply steady pressure to syringe to deliver formula to stomach in a timely manner.

ANS: A The formula is allowed to flow by gravity. The length of time to complete the feeding will vary. Preferably, the tube is inserted through the mouth. Newborns are obligatory nose breathers, and the presence of the tube in the nose irritates the nasal mucosa. Passage of the tube through the mouth allows the nurse to observe and evaluate the sucking response. The feeding should not be done under pressure. This procedure is not used as a timesaver for the nurse.

Which is the central factor responsible for respiratory distress syndrome? a. Deficient surfactant production b. Overproduction of surfactant c. Overdeveloped alveoli d. Absence of alveoli

ANS: A The successful adaptation to extrauterine breathing requires numerous factors, which most term newborns successfully accomplish. Preterm newborns with respiratory distress are not able to adjust. The most likely central cause is the abnormal development of the surfactant system. The deficient production of surfactant results in unequal inflation of alveoli on inspiration and the collapse of the alveoli on end expiration. The number and state of development of the alveoli is not a central factor in respiratory distress syndrome. The instability of the alveoli related to the lack of surfactant is the causative issue.

The newborn with severe jaundice is at risk for developing: a. encephalopathy. b. bullous impetigo. c. respiratory distress. d. blood incompatibility.

ANS: A Unconjugated bilirubin, which can cross the blood-brain barrier, is highly toxic to neurons. A newborn with severe jaundice is at risk for developing kernicterus or bilirubin encephalopathy. Encephalopathy is a highly infectious bacterial infection of the skin. It has no relation to severe jaundice and is the most likely complication of severe jaundice. A blood incompatibility may be the causative factor for the severe jaundice.

A blood sample for measurement of bilirubin is required from a newborn receiving phototherapy. In what environment should this blood sample be drawn? a. While phototherapy lights are turned off b. While newborn remains under phototherapy lights c. When newborn is covered with a blanket d. When newborn has been off phototherapy for 30 to 60 minutes

ANS: A When blood is drawn, phototherapy lights are turned off, and the blood is transported in a covered tube to avoid a false reading as a result of bilirubin destruction in the test tube. The lights will cause a degradation of the bilirubin in the sample, resulting in a falsely lowered result. The newborn does not need to be covered with a blanket. The phototherapy lights must be off. There is no reason to delay obtaining the blood sample. It can be drawn as soon as the lights are turned off.

Which finding on a newborn assessment should the nurse recognize as suggestive of a clavicle fracture? a. Negative scarf sign b. Asymmetric Moro reflex c. Swelling of fingers on affected side d. Paralysis of affected extremity and muscles

ANS: B A newborn with a broken clavicle may have no symptoms. The Moro reflex, which results in sudden extension and abduction of the extremities followed by flexion and adduction of the extremities, will most likely be asymmetric. The scarf sign that is used to determine gestational age should not be performed if a broken clavicle is suspected. Swelling of fingers on affected side and paralysis of affected extremity and muscles are not indicative of a fractured clavicle.

Which term refers to a newborn born before completion of week 37 of gestation, regardless of birth weight? a. Postterm b. Premature c. Low birth weight d. Small for gestational age

ANS: B A premature newborn is any child born before 37 weeks of gestation, regardless of birth weight. A postterm or postmature newborn is any child born after 42 weeks of gestational age, regardless of birth weight. A low-birth-weight newborn is a child whose birth weight is less than 2500 g, regardless of gestational age. A small-for-gestational-age (or small-for-date) newborn is any child whose rate of intrauterine growth was slowed and whose birth weight falls below the 10th percentile on intrauterine growth curves.

A mother is upset because her newborn has erythema toxicum neonatorum. The nurse should reassure her that this is: a. easily treated. b. benign and transient. c. usually not contagious. d. usually not disfiguring.

ANS: B Erythema toxicum neonatorum, or newborn rash, is a benign, self-limiting eruption of unknown cause that usually appears within the first 2 days of life. The rash usually lasts about 5 to 7 days. No treatment is indicated. Erythema toxicum neonatorum is not contagious. Successive crops of lesions heal without pigmentation.

Which is the primary treatment for hypoglycemia in newborns with feeding intolerance? a. Oral glucose feedings b. Intravenous (IV) infusion of glucose c. Short-term insulin therapy d. Feedings (formula or breast milk) at least every 2 hours

ANS: B IV infusions of glucose are indicated when the glucose level is very low and when feedings are not tolerated. Early feedings in the normoglycemic newborn are preventive. When the newborn is unable to tolerate feedings or the blood glucose level has become extremely low, then IV infusions are indicated. Insulin administration will further depress the blood glucose level. Feedings can be preventive. The child may not be able to tolerate this frequency.

When should the nurse expect jaundice to be present in a newborn with hemolytic disease? a. At birth b. During first 24 hours after birth c. 24 to 48 hours after birth d. 48 to 72 hours after birth

ANS: B In hemolytic disease of the newborn, jaundice is usually evident within the first 24 hours of life. Newborns with hemolytic disease are usually not jaundiced at birth, although some degree of hepatosplenomegaly, pallor, and hypovolemic shock may occur when the most severe form, hydrops fetalis, is present; 24 to 72 hours is too late for hemolytic disease of the newborn. Jaundice at these ages is most likely due to physiologic or early-onset breastfeeding jaundice.

The parents of a newborn ask the nurse what caused the baby's facial nerve paralysis. The nurse's response is based on knowledge that this is caused by a(n): a. genetic defect. b. birth injury. c. spinal cord injury. d. inborn error of metabolism.

ANS: B Pressure on the facial nerve during delivery may result in injury to cranial nerve VII, which can occur with birth injury. A genetic defect, spinal cord injury, or inborn error of metabolism would not cause facial paralysis.

Which statement best describes the clinical manifestations of the preterm newborn? a. Head is proportionately small in relation to the body. b. Sucking reflex is absent, weak, or ineffectual. c. Thermostability is well established. d. Extremities remain in attitude of flexion.

ANS: B Reflex activity is only partially developed. Sucking is absent, weak, or ineffectual. The preterm newborn's head is proportionately larger than the body. Thermoregulation is poorly developed, and the preterm newborn needs a neutral thermal environment to be provided. The preterm newborn may be listless and inactive compared with the overall attitude of flexion and activity of a full-term newborn.

A healthy, stable, preterm newborn will soon be discharged. The nurse should recommend which position for sleep? a. Prone b. Supine c. Side lying d. Position of comfort

ANS: B The American Academy of Pediatrics recommends that healthy newborns be placed to sleep in a supine position. Other positions are associated with sudden infant death syndrome. The prone position can be used for supervised play.

The nurse is preparing a parent of a newborn for home phototherapy. Which statement made by the parent would indicate a need for further teaching? a. "I should change the baby's position many times during the day." b. "I can dress the baby in lightweight clothing while under phototherapy." c. "I should be sure that the baby's eyelids are closed before applying patches." d. "I can take the patches off the baby during feedings and other caregiving activities."

ANS: B The baby should be placed nude under the lights. The newborn should be repositioned frequently to expose all body surfaces to the lights. The newborn's eyelids must be closed before the patches are applied because the corneas may become excoriated if in contact with the dressing. The eye patches should be removed so the newborn can have visual and sensory stimulation.

Which intervention should the nurse implement to maintain the skin integrity of the premature newborn? a. Cleanse skin with a gentle alkaline-based soap and water. b. Cleanse skin with a neutral pH solution only when necessary. c. Thoroughly rinse skin with plain water after bathing in a mild hexachlorophene solution. d. Avoid cleaning skin.

ANS: B The premature newborn should be given baths no more than two or three times per week with a neutral pH solution. The eyes, oral and diaper areas, and pressure points should be cleansed daily. Alkaline-based soaps might destroy the acid mantle of the skin. They should not be used. The increased permeability of the skin facilitates absorption of the chemical ingredients. The newborn's skin must be cleaned to remove stool and urine, which are irritating to the skin.

The nurse is caring for a newborn receiving an exchange transfusion for hemolytic disease. Assessment of the newborn reveals slight respiratory distress and tachycardia. Which should the nurse's first action be? a. Notify practitioner. b. Stop the transfusion. c. Administer calcium gluconate. d. Monitor vital signs electronicall

ANS: B When signs of cardiac or respiratory problems occur, the procedure is stopped, and the newborn's cardiorespiratory status is allowed to stabilize. The practitioner is usually performing the exchange transfusion with the nurse's assistance. The procedure must be stopped so the newborn can stabilize. Respiratory distress and tachycardia are signs of cardiorespiratory problems, not hypocalcemia. Calcium gluconate is not indicated. The vital signs should be monitored electronically throughout the entire procedure

The nurse is planning care for a low-birth-weight newborn. Which is an appropriate nursing intervention to promote adequate oxygenation? a. Place in Trendelenburg position periodically. b. Suction at least every 2 to 3 hours. c. Maintain neutral thermal environment. d. Hyperextend neck with nose pointing to ceiling

ANS: C A neutral thermal environment is one that permits the newborn to maintain a normal core temperature with minimal oxygen consumption and caloric expenditure. The Trendelenburg position should be avoided. This position can contribute to increased intracranial pressure (ICP) and reduced lung capacity from gravity pushing organs against diaphragm. Suctioning should be done only as necessary. Routine suctioning may cause bronchospasm, bradycardia due to vagal nerve stimulation, hypoxia, and increased ICP. Neck hyperextension is avoided because it reduces diameter of trachea

A nurse is assessing for jaundice in a dark-skinned newborn. Where is the best place to assess for jaundice in this newborn? a. Buttocks b. Tip of nose and sclera c. Sclera, conjunctiva, and oral mucosa d. Palms of hands and soles of feet

ANS: C Assessing for jaundice is part of the routine physical assessment in newborns. In dark-skinned newborns, the sclera, conjunctiva, and oral mucosa are the best place to observe jaundice because of the lack of skin pigmentation in these areas. The skin pigmentation in the buttocks, tip of nose and sclera, and palms of hands and soles of feet can mask the appearance of jaundice.

When should the nurse expect breastfeeding-associated jaundice to first appear in a normal newborn? a. 0 to 12 hours b. 12 to 24 hours c. 2 to 4 days d. 4 to 5 days

ANS: C Breastfeeding-associated jaundice is caused by decreased milk intake related to decreased caloric and fluid intake by the newborn before the mother's milk is well established. Fasting is associated with decreased hepatic clearance of bilirubin; 0 to 24 hours is too soon. Jaundice within the first 24 hours is associated with hemolytic disease of the newborn; 4 to 5 days is too late. Jaundice at this time may be due to breast-milk jaundice.

Early clinical manifestations of bilirubin encephalopathy in the newborn include: a. mental retardation. b. absence of stooling. c. lethargy or irritability. d. increased or decreased temperature.

ANS: C Clinical manifestations of bilirubin encephalopathy are those of nervous system depression or excitation. Prodromal symptoms consist of decreased activity, lethargy, irritability, hypotonia, and seizures. Newborns who survive may have evidence of mental retardation. Absence of stooling and increased/decreased temperature are not manifestations of bilirubin encephalopathy.

Several types of seizures can occur in the newborn. Which is characteristic of clonic seizures? a. Apnea b. Tremors c. Rhythmic jerking movements d. Extensions of all four limbs

ANS: C Clonic seizures are characterized by slow rhythmic jerking movements that occur approximately 1 to 3 per second. Apnea is a common manifestation of subtle seizures. Tremors are not characteristic of seizure activity. They may be indicative of hypoglycemia or hypocalcemia. A clonic seizure would have extension and contraction of the extremities, not just extension.

A newborn is diagnosed with retinopathy of prematurity. The nurse should know that: a. blindness cannot be prevented. b. no treatment is currently available. c. cryotherapy and laser therapy are effective treatments. d. long-term administration of oxygen will be necessary.

ANS: C Cryotherapy and laser photocoagulation therapy can be used to minimize the vascular proliferation process that causes the retinal damage. Blindness can be prevented with early recognition and treatment. Cryotherapy and laser therapy can be used to stop the process. Surgical intervention can be used to repair a detached retina if necessary. Long-term administration of oxygen is one of the causes. Oxygen should be used judiciously.

The nurse is caring for a very low-birth-weight (VLBW) newborn with a peripheral intravenous infusion. Which statement describes nursing considerations regarding infiltration? a. Infiltration occurs infrequently because VLBW newborns are inactive. b. Continuous infusion pumps stop automatically when infiltration occurs. c. Hypertonic solutions can cause severe tissue damage if infiltration occurs. d. Infusion site should be checked for infiltration at least once per 8-hour shift.

ANS: C Hypertonic fluids can damage cells if the fluid leaks from the vein. Careful monitoring is required to prevent severe tissue damage. Infiltrations occur for many reasons, not only activity. The vein, catheter, and fluid used all contribute to the possibility of infiltration. The continuous infusion pump may alarm when the pressure increases, but this does not alert the nurse to all infiltrations. Infusion rates and sites should be checked hourly to prevent tissue damage from extravasations, fluid overload, and dehydration.

A preterm newborn has been receiving orogastric feedings of breast milk. The nurse initiates nipple feedings, but the newborn tires easily and has weak sucking and swallowing reflexes. The most appropriate nursing intervention is to: a. encourage mother to breastfeed. b. try nipple-feeding preterm newborn formula. c. resume orogastric feedings of breast milk. d. resume orogastric feedings of formula.

ANS: C If a preterm newborn tires easily or has weak sucking when nipple feedings are initiated, the nurse should resume orogastric feedings with the milk of mother's choice. When nipple feeding is unsuccessful, it is unlikely that the newborn will be able to breastfeed. Breast milk should be continued as long as the mother desires.

The nurse is caring for a newborn with hyperbilirubinemia who is receiving phototherapy. Which is an appropriate nursing intervention for this newborn? a. Apply lotion as prescribed to moisturize skin. b. Maintain nothing-by-mouth (NPO) status to prevent nausea and vomiting. c. Monitor temperature to prevent hypothermia or hyperthermia. d. Keep eye patches on for at least 8 to 12 of every 24 hours.

ANS: C Newborns who are receiving phototherapy are at risk for thermoregulation issues. The nurse must monitor the newborn's temperature closely to rapidly detect either hypothermia or hyperthermia. Lotions are not used. They may predispose the newborn to increased tanning or "frying" effect. Newborns receiving phototherapy require additional fluid to compensate for increased fluid losses caused by the lights. The eye patches must be in place whenever the child is under the phototherapy lights.

What is oral candidiasis (thrush) in the newborn? a. Bacterial infection that is life threatening in the neonatal period b. Bacterial infection of mucous membranes that responds readily to treatment c. Yeastlike fungal infection of mucous membranes that is relatively common d. Benign disorder that is transmitted from mother to newborn during the birth process only

ANS: C Oral candidiasis, characterized by white adherent patches on the tongue, palate, and inner aspects of the cheeks, is not uncommon in newborns. Candida albicans is the usual causative organism. Oral candidiasis is usually a benign disorder in the newborn, often confined to the oral and diaper regions. It is caused by a yeastlike organism and is treated with good hygiene, application of a fungicide, and correction of any underlying disorder. Thrush can be transmitted in several ways, including by maternal transmission during delivery; person-to-person transmission; and contaminated bottles, hands, or other objects.

Physiologic jaundice in a newborn can be caused by: a. fetal-maternal blood incompatibility. b. destruction of red blood cells as a result of antibody reaction. c. liver's inability to bind bilirubin adequately for excretion. d. immature kidneys' inability to hydrolyze and excrete bilirubin.

ANS: C Physiologic jaundice is caused by the immature hepatic function of the newborn's liver coupled with the increased load from red blood cell hemolysis. The excess bilirubin from the destroyed red blood cells cannot be excreted from the body. The fetal-maternal blood incompatibility and the associated red cell destruction by antibodies are the causes of hemolytic disease of the newborn. The kidneys are not involved in the excretion of bilirubin.

The parents of a newborn with a strawberry hemangioma ask the nurse what the treatment will be. The nurse's response should be based on knowledge that: a. excision of the lesion will be necessary. b. injections of prednisone into the lesion will reduce it. c. no treatment is usually necessary because of the high rate of spontaneous involution. d. pulsed dye laser treatments will be necessary immediately to prevent permanent disability.

ANS: C There is a high rate of spontaneous resolution, so treatment is usually not indicated for hemangiomas. Surgical removal would not be indicated. If steroids are indicated, then systemic prednisone is administered for 2 to 3 weeks. The pulse dye laser is used in the uncommon situation of potential visual or respiratory impairment.

Which is the most appropriate nursing intervention for the newborn who is jittery and twitching and has a high-pitched cry? a. Monitor blood pressure closely. b. Obtain urine sample to detect glycosuria. c. Obtain serum glucose and serum calcium levels. d. Administer oral glucose or, if newborn refuses to suck, IV dextrose.

ANS: C These are signs and symptoms of hypocalcemia and hypoglycemia. A blood test is useful to determine the treatment. Laboratory analysis for calcium and blood glucose should be the priority intervention. Monitoring vital signs is important, but recognition of the possible hypocalcemia and hypoglycemia is imperative. A finding of glycosuria would not facilitate the diagnosis of hypoglycemia. A determination must be made between the hypocalcemia and hypoglycemia before treatment can be initiated.

Which is an important nursing action related to the use of tape and/or adhesives on premature newborns? a. Avoid using tape and adhesives until skin is more mature. b. Use solvents to remove tape and adhesives instead of pulling on skin. c. Remove adhesives with warm water or mineral oil. d. Use scissors carefully to remove tape instead of pulling tape off

ANS: C Warm water, mineral oil, or petrolatum can be used to facilitate the removal of adhesive. In the premature newborn, often it is impossible to avoid using adhesives and tape. The smallest amount of adhesive necessary should be used. Solvents should be avoided because they tend to dry and burn the delicate skin. Scissors should not be used to remove dressings or tape from the extremities of very small and immature newborns because it is easy to snip off tiny extremities or nick loosely attached skin.

The nurse is caring for a high-risk newborn with an umbilical catheter in a radiant warmer. The nurse notes blanching of the feet. Which is the most appropriate nursing action? a. Elevate feet 15 degrees. b. Place socks on newborn. c. Wrap feet loosely in prewarmed blanket. d. Report findings immediately to the practitioner.

ANS: D Blanching of the feet, in a newborn with an umbilical catheter, is an indication of vasospasm. Vasoconstriction of the peripheral vessels, triggered by the vasospasm, can seriously impair circulation. It is an emergency situation and must be reported immediately.

A preterm newborn of 36 weeks of gestation is admitted to the NICU. Approximately 2 hours after birth, the newborn begins having difficulty breathing, with grunting, tachypnea, and nasal flaring. Which is important for the nurse to recognize? a. This is a normal finding. b. This is not significant unless cyanosis is present. c. Improvement should occur within 24 hours. d. Further evaluation is needed.

ANS: D Difficulty breathing, with grunting, tachypnea, and nasal flaring are clinical manifestations of respiratory distress syndrome and require further evaluation. This is not a normal finding and requires further evaluation. Cyanosis may be present, but these are significant findings indicative of respiratory distress without cyanosis. The child's condition will most likely worsen for approximately 48 hours without intervention. Improvement may begin at 72 hours.

Meconium aspiration syndrome is caused by: a. hypoglycemia. b. carbon dioxide retention. c. bowel obstruction with meconium. d. aspiration of meconium in utero or at birth.

ANS: D Meconium aspiration syndrome is caused by the aspiration of amniotic fluid containing meconium into the fetal or newborn trachea in utero or at first breath. Hypoglycemia and carbon dioxide retention are not related to meconium aspiration. Bowel obstruction with meconium may be an indication of cystic fibrosis or Hirschsprung disease, not meconium aspiration.

The nurse is planning care for a family expecting their newborn to die. The nurse's interventions should be based on which statement? a. Tangible remembrances of the newborn (e.g., lock of hair, picture) prolong grief. b. Photographs of newborns should not be taken after the death has occurred. c. Funerals are not recommended because mother is still recovering from childbirth. d. Parents should be encouraged to name their newborn if they have not done so already.

ANS: D Naming the deceased newborn is an important step in the grieving process. It gives the parents a tangible person for whom to grieve, which is a key component of the grieving process. Tangible remembrances and photographs can make the newborn seem more real to the parents. Many NICUs will make bereavement memory packets, which may include a lock of hair, handprint, footprints, bedside name card, and other individualized objects. Families need to be informed of their options. The ritual of a funeral provides an opportunity for the parents to be supported by relatives and friends.

When is the best time for the neonatal intensive care unit (NICU) nurse to initiate an individualized stimulation program for the preterm newborn? a. As soon as possible after newborn is born b. As soon as parent is available to provide stimulation c. When newborn is over 38 weeks of gestation d. When developmental organization and stability are sufficient

ANS: D Newborn stimulation is essential for growth and development. The appropriate time for the introduction of an individualized program is when developmental organization and stability are achieved at approximately 34 and 36 weeks of gestation. The newborn needs to be developmentally ready for a stimulation program. The newborn must be assessed to determine the readiness and appropriateness of the stimulation program. The program should be designed and implemented by the nursing staff. The family can be involved, as the nurses help teach the parents to be responsive to the child's cues, but the stimulation should not depend on the family's availability. An individualized stimulation program should be started when the child is developmentally ready.

The mother of a preterm newborn asks the nurse when she can start breastfeeding. The nurse should explain that breastfeeding can be initiated when her newborn: a. achieves a weight of at least 3 pounds. b. indicates an interest in breastfeeding. c. does not require supplemental oxygen. d. has adequate sucking and swallowing reflexes.

ANS: D Research supports that human milk is the best source of nutrition for term and preterm newborns. Preterm newborns should be breastfed as soon as they have adequate sucking and swallowing reflexes and no other complications such as respiratory complications or concurrent illnesses. Weight is not an issue. Interest in breastfeeding can be evaluated by having nonnutritive sucking at the breast during skin-to-skin kangaroo care so the mother and child may become accustomed to each other. Supplemental oxygen can be provided during breastfeeding by using a nasal cannula.

Which is a bright red, rubbery nodule with a rough surface and a well-defined margin that may be present at birth? a. Port-wine stain b. Juvenile melanoma c. Cavernous hemangioma d. Strawberry hemangioma

ANS: D Strawberry hemangiomas or capillary hemangiomas are benign cutaneous tumors that involve capillaries only. They are bright red, rubbery nodules with rough surfaces and well-defined margin. They may or may not be apparent at birth but enlarge during the first year of life and tend to resolve spontaneously by age 2 to 3 years. Port-wine stain is a vascular stain that is a permanent lesion and is present at birth. Initially it is a pink, red, or, rarely, purple stain of the skin that is flat at birth and thickens, darkens, and proportionately enlarges as the child grows. Melanoma is not differentiated into juvenile and adult forms. A cavernous hemangioma involves deeper vessels in the dermis and has a bluish red color and poorly defined margins.

Which is most descriptive of the clinical manifestations observed in neonatal sepsis? a. Seizures and sunken fontanels b. Sudden hyperthermia and profuse sweating c. Decreased urinary output and frequent stools d. Nonspecific physical signs with hypothermia

ANS: D The clinical manifestations of neonatal sepsis are usually characterized by the newborn generally "not doing well." Poor temperature control, usually with hypothermia, lethargy, poor feeding, pallor, cyanosis or mottling, and jaundice, may be evident. Seizures and sunken fontanels are not manifestations of the sepsis. Severe neurologic sequelae may occur in low-birth-weight children with sepsis. Hyperthermia is rare in neonatal sepsis. Urinary output is not affected by sepsis.

The parents of a newborn who has just died decide they want to hold their deceased infant. The most appropriate nursing intervention at this time is to: a. explain gently that this is no longer possible. b. encourage parents to accept the loss of their newborn. c. offer to take a photograph of their newborn because they cannot hold newborn. d. get the newborn, wrap in a blanket, and rewarm in a radiant warmer so parents can hold their deceased infant.

ANS: D The parents should be allowed to hold their newborn in the hospital setting. The newborn's body should be retrieved and rewarmed in a radiant warmer. The nurse should provide a private place where the parents can hold their child for a final time. A photograph is an excellent idea, but it does not replace the parents' need to hold the child.

The nurse is teaching nursing students about normal physiologic changes in the respiratory system of toddlers. Which best describes why toddlers have fewer respiratory tract infections as they grow older? a. The amount of lymphoid tissue decreases. b. Repeated exposure to organisms causes increased immunity. c. Viral organisms are less prevalent in the population. d. Secondary infections rarely occur after viral illnesses.

B Children have increased immunity after exposure to a virus. The amount of lymphoid tissue increases as children grow older. Viral organisms are not less prevalent, but older children have the ability to resist invading organisms. Secondary infections after viral illnesses include Mycoplasma pneumoniae and group A b-hemolytic streptococcal infections.

Which type of croup is always considered a medical emergency? a. Laryngitis b. Epiglottitis c. Spasmodic croup d. Laryngotracheobronchitis (LTB)

B Epiglottitis is always a medical emergency needing antibiotics and airway support for treatment. Laryngitis is a common viral illness in older children and adolescents, with hoarseness and URI symptoms. Spasmodic croup is treated with humidity. LTB may progress to a medical emergency in some children.

Cardiopulmonary resuscitation (CPR) is begun on a toddler. Which pulse is usually palpated because it is the most central and accessible? a. Radial b. Carotid c. Femoral d. Brachial

B In a toddler, the carotid pulse is palpated. The radial pulse is not considered a central pulse. The femoral pulse is not the most central and accessible. Brachial pulse is felt in infants younger than 1 year.

The nurse is caring for a child with acute respiratory distress syndrome (ARDS) associated with sepsis. Nursing actions should include: a. forcing fluids. b. monitoring pulse oximetry. c. instituting seizure precautions. d. encouraging a high-protein diet.

B Monitoring cardiopulmonary status is an important evaluation tool in the care of the child with ARDS. Maintenance of vascular volume and hydration is important and should be done parenterally. Seizures are not a side effect of ARDS. Adequate nutrition is necessary, but a high-protein diet is not helpful.

A child with cystic fibrosis (CF) is receiving recombinant human deoxyribonuclease (DNase). Which is an adverse effect of this medication? a. Mucus thickens b. Voice alters c. Tachycardia d. Jitteriness

B One of the only adverse effects of DNase is voice alterations and laryngitis. DNase decreases viscosity of mucus, is given in an aerosolized form, and is safe for children younger than 12 years. b2 agonists can cause tachycardia and jitteriness.

The nurse encourages the mother of a toddler with acute laryngotracheobronchitis (LTB) to stay at the bedside as much as possible. The nurse's rationale for this action is described primarily in which statement? a. Mothers of hospitalized toddlers often experience guilt. b. The mother's presence will reduce anxiety and ease child's respiratory efforts. c. Separation from mother is a major developmental threat at this age. d. The mother can provide constant observations of the child's respiratory efforts.

B The family's presence will decrease the child's distress. It is true that mothers of hospitalized toddlers often experience guilt but this is not the best answer. The main reason to keep parents at the child's bedside is to ease anxiety and therefore respiratory effort. The child should have constant monitoring by cardiorespiratory monitor and noninvasive oxygen saturation monitoring, but the parent should not play this role in the hospital.

A nurse is admitting an infant with asthma. The nurse understands that asthma in infants is usually triggered by: a. medications. b. a viral infection. c. exposure to cold air. d. allergy to dust or dust mites.

B Viral illnesses cause inflammation that causes increased airway reactivity in asthma. Medications such as aspirin, nonsteroidal anti-inflammatory drugs (NSAIDs), and antibiotics may aggravate asthma, but not frequently in infants. Exposure to cold air may exacerbate already existing asthma. Allergy is associated with asthma, but 20% to 40% of children with asthma have no evidence of allergic disease.

b-Adrenergic agonists and methylxanthines are often prescribed for a child with an asthma attack. Which describes their action? a. Liquefy secretions. b. Dilate the bronchioles. c. Reduce inflammation of the lungs. d. Reduce infection.

B b-Adrenergic agonists and methylxanthines work to dilate the bronchioles in acute exacerbations. These medications do not liquefy secretions or reduce infection. Corticosteroids and mast cell stabilizers reduce inflammation in the lungs.

A nurse is caring for a school-age child with left unilateral pneumonia and pleural effusion. A chest tube has been inserted to promote continuous closed chest drainage. Which interventions should the nurse implement when caring for this child? (Select all that apply.) a. Positioning child on the right side b. Assessing the chest tube and drainage device for correct settings c. Administering prescribed doses of analgesia d. Clamping the chest tube when child ambulates e. Monitoring for need of supplemental oxygen

B, C, E Nursing care of the child with a chest tube requires close attention to respiratory status; the chest tube and drainage device used are monitored for proper function (i.e., drainage is not impeded, vacuum setting is correct, tubing is free of kinks, dressing covering chest tube insertion site is intact, water seal is maintained, and chest tube remains in place). Movement in bed and ambulation with a chest tube are encouraged according to the child's respiratory status, but children require frequent doses of analgesia. Supplemental oxygen may be required in the acute phase of the illness and may be administered by nasal cannula, face mask, flow-by, or face tent. The child should be positioned on the left side, not the right. Lying on the affected side if the pneumonia is unilateral ("good lung up") splints the chest on that side and reduces the pleural rubbing that often causes discomfort. The chest tube should never be clamped; this can cause a pneumothorax. The chest tube should be maintained to the underwater seal at all times.

Which drug is considered the most useful in treating childhood cardiac arrest? a. Bretylium tosylate (Bretylium) b. Lidocaine hydrochloride (Lidocaine) c. Epinephrine hydrochloride (Adrenaline) d. Naloxone (Narcan)

C Epinephrine works on alpha and beta receptors in the heart and is the most useful drug in childhood cardiac arrest. Bretylium is no longer used in pediatric cardiac arrest management. Lidocaine is used for ventricular arrhythmias only. Naloxone is useful only to reverse effects of opioids.

A nurse is teaching an adolescent how to use the peak expiratory flowmeter. The adolescent has understood the teaching if which statement is made? a. "I will record the average of the readings." b. "I should be sitting comfortably when I perform the readings." c. "I will record the readings at the same time every day." d. "I will repeat the routine two times."

C Instructions for use of a peak flowmeter include standing up straight before performing the reading, recording the highest of the three readings (not the average), measuring the peak expiratory flow rate (PEFR) close to the same time each day, and repeating the entire routine three times, waiting 30 seconds between each routine.

A nurse is caring for a child in acute respiratory failure. Which blood gas analysis indicates the child is still in respiratory acidosis? a. pH 7.50, CO2 48 b. pH 7.30, CO2 30 c. pH 7.32, CO2 50 d. pH 7.48, CO2 33

C Respiratory failure is a process that involves pulmonary dysfunction generally resulting in impaired alveolar gas exchange, which can lead to hypoxemia or hypercapnia. Acidosis indicates the pH is less than 7.35 and the CO2 is greater than 45. If the pH is less than 7.35 but the CO2 is low, it is metabolic acidosis. Alkalosis is when the pH is greater than 7.45. If the pH is high and the CO2 is high, it is metabolic alkalosis. When the pH is high and the CO2 is low, it is respiratory alkalosis.

Parents of two school-age children with asthma ask the nurse, "What sports can our children participate in?" The nurse should recommend which sport? a. Soccer b. Running c. Swimming d. Basketball

C Swimming is well tolerated in children with asthma because they are breathing air fully saturated with moisture and because of the type of breathing required in swimming. Exercise-induced bronchospasm is more common in sports that involve endurance, such as soccer. Prophylaxis with medications may be necessary.

Which frequency is recommended for childhood skin testing for tuberculosis (TB) using the Mantoux test? a. Every year for all children older than 2 years b. Every year for all children older than 10 years c. Every 2 years for all children starting at age 1 year d. Periodically for children who reside in high-prevalence regions

D Children who reside in high-prevalence regions for TB should be tested every 2 to 3 years. Annual testing is not necessary. Testing is not necessary unless exposure is likely or an underlying medical risk factor is present.

Pancreatic enzymes are administered to the child with cystic fibrosis (CF). Nursing considerations should include to: a. not administer pancreatic enzymes if child is receiving antibiotics. b. decrease dose of pancreatic enzymes if child is having frequent, bulky stools. c. administer pancreatic enzymes between meals if at all possible. d. pancreatic enzymes can be swallowed whole or sprinkled on a small amount of food taken at the beginning of a meal.

D Enzymes may be administered in a small amount of cereal or fruit at the beginning of a meal or swallowed whole. Pancreatic enzymes are not a contraindication for antibiotics. The dosage of enzymes should be increased if child is having frequent, bulky stools. Enzymes should be given just before meals and snacks.

The parent of an infant with nasopharyngitis should be instructed to notify the health professional if the infant displays which clinical manifestation? a. Fussiness b. Coughing c. A fever over 99° F d. Signs of an earache

D If an infant with nasopharyngitis shows signs of an earache, it may mean a secondary bacterial infection is present and the infant should be referred to a practitioner for evaluation. Irritability is common in an infant with a viral illness. Cough can be a sign of nasopharyngitis. Fever is common in viral illnesses.

Chronic otitis media with effusion (OME) is differentiated from acute otitis media (AOM) because it is usually characterized by: a. a fever as high as 40° C (104° F). b. severe pain in the ear. c. nausea and vomiting. d. a feeling of fullness in the ear.

D OME is characterized by feeling of fullness in the ear or other nonspecific complaints. Fever is a sign of AOM. OME does not cause severe pain. This may be a sign of AOM. Nausea and vomiting are associated with otitis media.

Parents have understood teaching about prevention of childhood otitis media if they make which statement? a. "We will only prop the bottle during the daytime feedings." b. "Breastfeeding will be discontinued after 4 months of age." c. "We will place the child flat right after feedings." d. "We will be sure to keep immunizations up to date."

D Parents have understood the teaching about preventing childhood otitis media if they respond they will keep childhood immunizations up to date. The child should be maintained upright during feedings and after. Otitis media can be prevented by exclusively breastfeeding until at least 6 months of age. Propping bottles is discouraged to avoid pooling of milk while the child is in the supine position.

Which is an appropriate nursing intervention when caring for an infant with an upper respiratory tract infection and elevated temperature? a. Give tepid water baths to reduce fever. b. Encourage food intake to maintain caloric needs. c. Have child wear heavy clothing to prevent chilling. d. Give small amounts of favorite fluids frequently to prevent dehydration.

D Preventing dehydration by small frequent feedings is an important intervention in the febrile child. Tepid water baths may induce shivering, which raises temperature. Food should not be forced; it may result in the child vomiting. The febrile child should be dressed in light, loose clothing.

Which drug is usually given first in the emergency treatment of an acute, severe asthma episode in a young child? a. Ephedrine b. Theophylline c. Aminophylline d. Short-acting b2 agonists

D Short-acting b2 agonists are the first treatment in an acute asthma exacerbation. Ephedrine is not helpful in acute asthma exacerbations. Theophylline is unnecessary for treating asthma exacerbations. Aminophylline is not helpful for acute asthma exacerbation

The mother of a toddler yells to the nurse, "Help! He is choking to death on his food." The nurse determines that lifesaving measures are necessary based on which symptom? a. Gagging b. Coughing c. Pulse over 100 beats/min d. Inability to speak

D The inability to speak is indicative of a foreign-body airway obstruction of the larynx. Abdominal thrusts are needed for treatment of the choking child. Gagging indicates irritation at the back of the throat, not obstruction. Coughing does not indicate a complete airway obstruction. Tachycardia may be present for many reasons.

Newborns are highly susceptible to infection as a result of: a. excessive levels of immunoglobulin A (IgA) and immunoglobulin M (IgM). b. diminished nonspecific and specific immunity. c. increased humoral immunity. d. overwhelming anti-inflammatory response.

NS: B Newborns have diminished inflammatory (nonspecific) and humoral (specific) immunity. They are unable to mount a local inflammatory reaction at the portal of entry to signal infection, and the resulting symptoms are vague and nonspecific, delaying diagnosis and treatment. Newborns have diminished or absent IgA and IgM. Humoral and anti-inflammatory immune responses are diminished in newborns.

The nurse has been caring for a newborn who just died. The parents are present but say they are "afraid" to hold the dead newborn. Which is the most appropriate nursing intervention? a. Tell them there is nothing to fear. b. Insist that they hold newborn "one last time." c. Respect their wishes and release body to morgue. d. Keep newborn's body available for a few hours in case they change their minds.

d. Keep newborn's body available for a few hours in case they change their minds.


Set pelajaran terkait

Health Care Team: Where the LPN Fits

View Set

BUS 207 Exam 2 Practice Problems

View Set

ITN 170 (Linux) Cengage review questions

View Set

Науки, які вивчає біологія

View Set

Week 3: Check Your Understanding

View Set

Supp. Reading Ch. 5 Test Questions

View Set